Sunteți pe pagina 1din 96

Facultatea de Matematică

Calcul Integral şi Aplicaţii, Semestrul I


Lector dr. Lucian MATICIUC

Seminar recapitulativ
Integrala definită. Primitive

1. Să se arate că


 Z a
a  2 f (x) dx , dacă f este funcţie pară,
Z 
f (x) dx = 0
−a 
0, dacă f este funcţie impară.

Rezolvare:

Astfel avem conform proprietăţii de aditivitate că


Z a Z 0 Z a
f (x) dx = f (x) dx + f (x) dx.
−a −a 0

Acum dacă f este pară, adică


f (−x) = f (x) , ∀ x ∈ [−a, a] ,
atunci ı̂n prima integrală fac schimbarea de variabilă

x = −y ⇔ y = −x

De aici obţinem că dx = −dy precum şi noile limite de integrare: dacă x = −a atunci y = a şi dacă
x = 0 atunci y = 0.
Deci integrala devine, conform schimbării de variabilă,
Z 0 Z 0 Z 0 Z 0
f (x) dx = f (−y) (−dy) = f (y) (−dy) = − f (y) dy
−a a a a

Dar, conform unei convenţii


Z b Z a
f (x) dx = − f (x) dx
a b

deci Z 0 Z a Z a
f (x) dx = f (y) dy = f (x) dx
−a 0 0

de unde obţinem că


Z a Z 0 Z a Z a
f (x) dx = f (x) dx + f (x) dx = 2 f (x) dx.
−a −a 0 0

Dacă f este impară, adică

f (−x) = −f (x) , ∀ x ∈ [−a, a] ,


atunci ı̂n prima integrală fac aceeaşi schimbarea de variabilă

x = −y ⇔ y = −x

De aici obţinem dx = −dy şi limitele de integrare devin: dacă x = −a atunci y = a şi dacă x = 0
atunci y = 0.

1
Deci integrala devine, conform schimbării de variabilă,
Z 0 Z 0 Z 0 Z 0 Z a Z a
f (x) dx = f (−y) (−dy) = −f (y) (−dy) = f (y) dy = − f (y) dy = − f (x) dx
−a a a a 0 0

deci Z 0 Z a Z a
f (x) dx = − f (y) dy = − f (x) dx.
−a 0 0

Obţinem deci că


Z a Z 0 Z a
f (x) dx = f (x) dx + f (x) dx = 0
−a −a 0

2. Arătaţi, folosind paritatea funcţiei de sub integrală, că


Z 1 Z 1/2
arctgx 1+x
a) dx = 0 , b) (cos x) ln dx = 0 ,
−1 e + e−x
x
−1/2 1−x

π/4 1
x3
Z Z
c) sin x·tg2 x = 0 , d) dx = 0
−π/4 −1 1 + x2

Rezolvare:

Aplic exerciţiul anterior. Astfel vom arăta că funcţiile care se integrează sunt impare.
Pentru aceasta folosim paritatea funcţiilor trigonometrice:

sin (−x) = − sin x , cos (−x) = cos x


tg (−x) = −tgx , arctg (−x) = −arctgx
arctgx
a) Notăm cu f : [−1, 1] → R, f (x) = ex +e−x .

arctg (−x) −arctgx


f (−x) = = −x = −f (x) .
e−x + e−(−x) e + ex

3. Fie f : R → R, o funcţie continuă şi periodică de perioadă T > 0. Să se arate că are loc
Z a+T Z T
f (x) dx = f (x) dx , ∀ a ∈ R
a 0

şi apoi să se calculeze:


Z 2nπ Z 2nπ
a) |sin x| dx , n ∈ N , b) |cos x| dx , n ∈ N
0 0
Rezolvare:

Funcţia f periodică ı̂nseamnă că f (x + T ) = f (x) , ∀ x ∈ R. Avem conform proprietăţii de aditivi-


tate că Z a+T Z 0 Z T Z a+T
f (x) dx = f (x) dx + f (x) dx + f (x) dx
a a 0 T

În ultima integrală fac schimbarea de variabilă

x=y+T ⇔y =x−T

De aici obţinem dx = dy şi limitele de integrare devin: dacă x = T atunci y = 0 şi dacă x = a + T
atunci y = a. Deci integrala devine, conform schimbării de variabilă,
Z a+T Z a Z a Z 0
f (x) dx = f (y + T ) dy = f (y) dy = − f (y) dy
T 0 0 a

2
deci Z a+T Z 0 Z T Z a+T Z T
f (x) dx = f (x) dx + f (x) dx + f (x) dx = f (x) dx
a a 0 T 0

a) Ştim că sin şi cos sunt periodice de periodice de perioadă 2π

sin (x + 2π) = sin x , ∀ x ∈ R


cos (x + 2π) = cos x , ∀ x ∈ R

deci evident şi funcţiile |sin x|, |cos x| .


Conform celor de mai sus, avem că
Z 2nπ Z 2π Z 4π Z 2nπ
|sin x| dx = |sin x| dx + |sin x| dx + . . . + |sin x| dx
0 0 2π 2(n−1)π
Z 2π Z 2π Z 2π Z 2π
= |sin x| dx + |sin x| dx + . . . + |sin x| dx = n |sin x| dx
0 0 0 0

Iar
Z 2π Z π Z 2π Z π Z 2π
|sin x| dx = |sin x| dx + |sin x| dx = sin xdx − sin xdx
0 0 π 0 π
π 2π
= (− cos x)|0 − (− cos x)|π = 4

deci Z 2nπ Z 2π
|sin x| dx = n |sin x| dx = 4n.
0 0

4. Să se calculeze următoarele integrale (folosind tabelul):


Z Z p Z Z Z Z
dx dx dx dx dx
a) √ , b) 2pxdx , c) √ , d) 2
, e) √ , f) √ ,
8−x 2 4+x 2 5 + x x x x

Z Z Z Z Z Z
dx dx dx dx xdx
g) √ , h) , i) 2
, j) 2
, k) 2 − 3xdx , l) √ ,
5x − 2 a−x 3x + 5 7x − 8 x2 + 1
Z Z
dx 3
m) √ , n) 2+7
dx.
7 − 5x 2 5x
5. Să se calculeze următoarele integrale (folosind metoda de integrare prin părţi):
Z Z Z Z Z
x + 5x e dx , b) e sin (bx) dx , c) x sin x dx , d) ln x dx , e) x3 ln2 x dx
2 3
 2x ax 2
a)

0
x−2
Z p Z p Z Z Z 
ln x −3
f) x2 + adx , a ∈ R, g) a2 − x2 dx , h) dx = x ln xdx = ln xdx.
x3 −2
Rezolvare:

Dacă f şi g sunt funcţii cu derivatele continue pe domeniul de definiţie I atunci are loc formula de
integrare prin părţi: Z Z
f (x) g 0 (x) dx = f (x) g (x) − f 0 (x) g (x) dx.

3
0
a) Folosim e2x = 21 e2x :
Z Z Z
 1 2x 0 1 1 0
x2 + 5x e2x dx = x2 + 5x dx = x2 + 5x e2x − x2 + 5x e2x dx

e
2 2 2
Z
1 2 1
x + 5x e2x − (2x + 5) e2x dx = aplicăm ı̂ncă o dată

=
2 2
Z
1 2 1 1 2x 0
x + 5x e2x −

= (2x + 5) e dx
2 2 2
 Z 
1 2  2x 1 1 2x 1 0 2x
= x + 5x e − (2x + 5) e − (2x + 5) e dx
2 2 2 2
 Z 
1 2 1 1 1
x + 5x e2x − (2x + 5) e2x − 2e2x dx

=
2 2 2 2
1 2 1 1 1
x + 5x e2x − (2x + 5) e2x + · e2x + C , C ∈ R

=
2 4 2 2
0
b) Folosim eax = a1 (eax ) :
Z Z Z
1 ax 0 1 1 ax 0
eax sin (bx) dx = (e ) sin (bx) dx = eax sin (bx) − e (sin (bx)) dx
a a a
Z
1 ax b
= e sin (bx) − eax cos (bx) dx = aplicăm ı̂ncă o dată
a a
Z  Z 
1 b 1 ax 0 1 b 0
= eax sin (bx) − (e ) cos (bx) dx = eax sin (bx) − 2 eax cos (bx) − eax (cos (bx)) dx
a a a a a
 Z 
1 b
= eax sin (bx) − 2 eax cos (bx) + beax sin (bx) dx
a a
b2
Z
1 b
= eax sin (bx) − 2 eax cos (bx) − 2 eax sin (bx) dx
a a a
Deci
Z Z
b 1 b
eax sin (bx) dx + 2 eax sin (bx) dx = eax sin (bx) − 2 eax cos (bx) + C , C ∈ R
a a a 
a2
Z 
ax 1 ax b ax
⇔ e sin (bx) dx = e sin (bx) − 2 e cos (bx) + C, C ∈ R
a2 + b a a
0
−1
Observaţie: putem pleca şi de la sin (bx) = b (cos (bx)) .
d) Z Z Z
0 0
ln3 x dx = x ln3 x dx = x ln3 x −
x ln3 x dx
Z Z
1
= x ln3 x − 3 x ln2 x dx = x ln3 x − 3 ln2 x dx = aplicăm ı̂ncă o dată
x
Z  Z 
0 0
= x ln3 x − 3 x ln2 x dx = x ln3 x − 3 x ln2 x − x ln2 x dx
 Z   Z 
3 2 1 3 2
= x ln x − 3 x ln x − 2 x ln x dx = x ln x − 3 x ln x − 2 ln xdx
x
  Z 
0
= x ln3 x − 3 x ln2 x − 2 x ln x − x (ln x) dx

= x ln3 x − 3 x ln2 x − 2 (x ln x − x) + C , C ∈ R.


4
f)
x2 + a
Z p Z
I= x2 + adx = (raţionalizare) = √ dx
x2 + a
Z
x2
Z
a
Z p 0 p
= √ dx + √ dx = x x2 + a dx + a ln x + x2 + a

x2 + a x2 + a
p  p Z p 
2
= a ln x + x + a + x x + a −
2 2
x + adx
p p
= a ln x + x2 + a + x x2 + a − I

Deci Z p
1 p 2 a p
I= x2 + adx = x x + a + ln x + x2 + a + C , C ∈ R.

2 2

6. Să se calculeze următoarele integrale folosind metoda de integrare prin părţi:


Z Z Z Z
x3 + 2x e5x dx , b) eax sin (bx) dx , c) eax cos (bx) dx , d) e3x sin (4x) dx ,

a)
Z Z Z Z
e) e4x cos (3x) dx , f ) x3 cos x dx , g) x3 sin (5x) dx , h) x3 cos (5x) dx ,
Z Z Z p Z p Z p
i) ln xdx , j) x2 ln3 x dx , k) x2 + 5dx , l) x2 − 5dx , m) 5 − x2 dx

Z Z Z
ln (ln x) 2
x2 − 2x + 3 ln xdx.
 
n) dx, o) x + 5x + 6 cos (2x) dx, p)
x
7. Folosind prima metodă de schimbare de variabilă să se calculeze:
Z Z Z
x + arccos x 1 1
a) √ dx , b) 2 dx , c) dx ,
1−x 2 x ln x x ln5 x
Z π/2 Z 3p Z p
cos x 2
d) dx , e) 6x − x − 5 dx , f ) x2 + x + 1dx
0 1 + sin2 x 2
Rezolvare:

Aplic prima metodă de schimbare de variabilă:


Z
0
f (u (x)) u (x) dx = F (u (x)) + C, C ∈ R,

unde F este o primitivă a lui funcţiei f.


De asemenea are loc şi ı̂n cazul integralei definite:
Z b Z u(b)
0 y=u(b)
f (u (x)) u (x) dx = f (y) dy = F (y)|y=u(a) = F (u (b)) − F (u (a)) .
a u(a)

0
a) Observ că √ 1 = (− arccos x) , deci
1−x2
Z
x + arccos x
Z
x
Z
arccos x
Z p 0 Z
0
√ dx = √ dx + √ dx = − 2
1 − x dx + arccos x (− arccos x) dx
1−x 2 1−x 2 1−x 2
Z Z
0
p p
2 2
= − 1 − x − arccos x (arccos x) dx = − 1 − x − arccos x d (arccos x) .

Acum dacă notăm


not 0
y = arccos x ⇒ dy = (arccos x) dx

5
deci integrala devine
Z Z
x + arccos x p
√ dx = − 1 − x2 − ydy
1 − x2
2
p y2 p (arccos x)
= − 1 − x2 − + C = − 1 − x2 − +C , C ∈R
2 2
0 not
1 0
b) Observ că x = (ln x) şi voi nota y = ln x ⇒ dy = (ln x) dx
Z Z Z Z
1 1 0 1
dx = (ln x) dx = dy = y −2 dy
x ln2 x ln2 x y2
y −3 (ln x)−3
= −3 +C = −3 +C , C ∈ R.
0 not 0
d) Observ că cos x = (sin x) şi voi nota y = sin x ⇒ dy = (sin x) dx şi limitele de integrare devin:
dacă x = 0 atunci y = sin 0 = 0 şi dacă x = π/2 atunci y = sin π/2 = 1
Z π/2 Z π/2 Z 1
cos x 1 0 1 1
2 dx = 2 (sin x) dx = 2
dy = arctgy|0 = arctg1−arctg0 = π/4−0
0 1 + sin x 0 1 + sin x 0 1+y

e) Folosim forma canonică a trinomului de gradul 2


 2
b −∆
ax2 + bx + c = a x + +
2a 4a
unde ∆ = b2 − 4ac.
Deci  2
6 −(36−20)
6x − x2 − 5 = −x2 + 6x − 5 = − x + −2 + −4
2 2
= − (x − 3) + 4 = 4 − (x − 3)
şi
Z 3 p Z 3 q Z 3 q
0
2 2
6x − x2 − 5dx = 4 − (x − 3) dx = 4 − (x − 3) (x − 3) dx
2 2 2
not 0
Notez y = x − 3 ⇒ dy = (x − 3) dx şi limitele de integrare devin: dacă x = 2 atunci y = −1 şi dacă
x = 3 atunci y = 0. Z 3p Z 0p
6x − x2 − 5dx = 4 − y 2 dy
2 −1
Pentru a calcula ultima integrală vezi exerciţiile precedente.
8. Folosind prima metodă de schimbare de variabilă să se calculeze:
Z Z Z Z
cos x sin x sin x cos x
a) dx , b) dx , c) √ dx , d) dx
sin x cos x cos x sin3 x
3 − 2x
Z Z Z
xdx 3
e) √ , f ) x2 ex dx, g) dx.
1+x 4 5x2 + 7
9. (i) Să se aducă la forma canonică următoarele trinoame de gradul al doilea:

a) f (x) = 4x − x2 + 5 , b) f (x) = x2 + 3x − 5 , c) f (x) = x2 + 2x + 3, d) f (x) = 2x2 − 3x + 5

e) f (x) = 2x2 − 5x + 3, f ) f (x) = x2 − x − 1, g) f (x) = 2 + 3x − 2x2 , h) f (x) = x2 + 2x + 2,

i) f (x) = x2 + 2x + 5, j) f (x) = 3x2 − x + 1, k) f (x) = 2 + 3x − 2x2 , l) f (x) = x2 − 4x + 5

(ii) Să se calculeze diferenţialele df (x) ale următoarelor funcţii de o variabilă:



a) f (x) = sin2 x , b) f (x) = ln x , c) f (x) = ln2 x, d) f (x) = x3 , e) f (x) = x,
√ √
f ) f (x) = cos x, g) f (x) = e3x , h) f (x) = x2 + a2 , i) f (x) = 4 − x2 , j) f (x) =
tgx .

6
10. Folosind a doua metodă de schimbare de variabilă să se calculeze integralele:
Z

Z 1 p Z p Z p
2 2 2 2 2
a) cos xdx , b) x 4 − x dx , c) a − x dx , d) x2 + a2 dx.
0
Rezolvare:

Aplic a doua metodă de schimbare de variabilă: Dacă facem schimbarea de variabilă

x = u (y)

atunci
dx = u0 (y) dy , y = u−1 (x)
unde u−1 este inversa funcţiei u, şi integrala devine
Z b Z u−1 (b)
f (x) dx = f (u (y)) u0 (y) dy
a u−1 (a)

a) Vom nota x = y ⇔ x = y 2 deci dx = 2ydy şi integrala devine

Z Z Z
cos2 xdx = cos2 y 2ydy = 2 y cos2 y dy


Pentru calculul acestei integrale vezi metoda de integrare prin părţi. La sfârşit se va ı̂nlocui y = x.
b) Avem substituţiile trigonometrice:

1. Dacă integrala conţine termenul a2 − x2 atunci este utilă substituţia x = a sin y sau x = a cos y

2. Dacă integrala conţine termenul x2 − a2 atunci este utilă substituţia x = a chy

3. Dacă integrala conţine termenul x2 + a2 atunci este utilă substituţia x = a shy

unde
def ex − e−x def ex + e−x
shx = , chx =
2 2
şi evident avem
0 0
ch2 x − sh2 x = 1 , (shx) = chx , (chx) = shx.
În cazul nostru este utilă substituţia x = 2 sin y (de asemenea e utilă şi substituţia x = 2 cos y). Deci
0
dx = (2 sin y) dy = 2 cos ydy

şi
x = 2 sin y ⇔ y = arcsin x/2.
Limitele de integrare devin: dacă x = 0 atunci y = arcsin 0 = 0 şi dacă x = 1 atunci y = arcsin 1/2 =
π/6. Atunci integrala devine
Z 1 p Z π/6 q
x2 4 − x2 dx = 4 sin2 y 4 − 4 sin2 y2 cos ydy
0 0
Z π/6 q Z π/6
= 16 sin2 y 1 − sin2 y cos ydy = 16 sin2 y cos2 ydy
0 0

Avem formulele
sin2 y + cos2 y = 1
sin 2y
sin (2y) = 2 sin y cos y ⇒ sin y cos y = 2
1+cos 2y
cos (2y) = 2 cos2 y − 1 ⇒ cos2 y = 2
1−cos 2y
cos (2y) = 1 − 2 sin2 y ⇒ sin2 y = 2

7
Deci
Z 1 p Z π/6 Z π/6
2
x2 4 − x2 dx = 16 (sin y cos y) dy = 4 sin2 2ydy
0 0 0
π/6 π/6 π/6   π/6 √
1 − cos 4y
Z Z Z
sin 4y π 3
=4 dy = 2 dy − 2 cos 4ydy = 2y − 2 = − .
0 2 0 0 4
0 3 4

ey − e−y
d) În acest caz este utilă substituţia x = . Deci
2
0
e − e−y ey + e−y
 y
dx = a dy = a dy
2 2

şi
2 2
e − e−y e + e−2y − 2 + e−2y + 2 e + e−y
 y  2y  2y
 y
2 2 2 2 2e
x +a = a +a =a +1 =a = a .
2 4 4 2

Deci
ey + e−y ey + e−y a2 a2
Z p Z Z Z
2
ey + e−y e2y + e−2y + 2 dy

x2 + a2 dx = a a dy = dy =
2 2 4 4
a2 e2y e−2y
 
= + + 2y + C , C ∈ R
4 2 −2

11. Să se calculeze următoarele integrale din funcţii ce conţin un trinom de gradul al doilea:
Z Z Z Z p
dx dx x+3
a) , b) √ , c) √ dx , d) a2 − x2 dx ,
2x2 − 5x + 7 2 + 3x − 2x2 x2 + 2x + 3
3x − 2
Z p Z p Z Z
2 2
dx
e) a + x dx , f ) 1 − 2x − x dx , g) 2
, h) 2
dx.
3x − x + 1 x − 4x + 5
12. Să se calculeze următoarele integrale din funcţii raţionale:
Z Z Z Z
1 1 1 1
a) dx , b) α dx , α 6= 1, c) dx , d) α dx,
x−a (x − a) ax − b (ax + b)

4x − 5
Z Z Z
1 1
e) dx, f ) dx, g) dx,
2x2 − 4x + 8 2x2 − 5x + 3 x2 − 2x + 10

−x + 5 x2 − 3x + 3
Z Z Z
a b c 
h) 2
dx , i) 3 2
dx = + + 2 dx ,
x +x−2 x − 2x + x x x − 1 (x − 1)

3x2 + x − 4
Z Z
a bx + c 
j) 3 2
dx = + 2 dx.
x + 5x + 9x + 5 x + 1 x + 4x + 5
13. Să se calculeze următoarele integrale din funcţii raţionale:

x4
Z Z Z Z
1 1 1
a) dx , b) dx , c) dx , d) dx
x4 − 1 x3 + 1 x3 − 2x2 + x x3 + 6x2 + 11x + 6

Rezolvare:
Dacă integrala este dintr-o funcţie raţională atunci:
Pasul I: dacă gradul numărătorului este mai mare decât gradul numitorului atunci mai ı̂ntâi se
ı̂mpart polinoamele până se ajunge ca gradul numărătorului să fie mai mic strict decât gradul
numitorului.
Pasul II: apoi se vor căuta divizorii numitorului şi se va descompune fracţia ı̂n fracţii simple.

8
a)
x4 x4 −1+1 x4 −1 1 1
x4 −1 = x4 −1 = x4 −1 + x4 −1 =1+ (x2 −1)(x2 +1)
1 1
=1+ (x2 −1)(x2 +1) =1+ (x−1)(x+1)(x2 +1)

Descompunerea ı̂n fracţii simple ı̂nseamnă să căută constantele a, b, c, d a.ı̂. să aibă loc
1 a b cx+d
(x−1)(x+1)(x2 +1) = x−1 + x+1 + x2 +1

Aducând la acelaşi numitor obţin

1 a(x+1)(x2 +1)+b(x−1)(x2 +1)+(cx+d)(x−1)(x+1)


(x−1)(x+1)(x2 +1) = (x−1)(x+1)(x2 +1)
 
⇔ 1 = a (x + 1) x2 + 1 + b (x − 1) x2 + 1 + (cx + d) (x − 1) (x + 1)
⇔ 1= (a + b + c) x3 + (a − b + d) x2 + (a + b − c) x + (a − b − d)

 a+b+c=0
a−b+d=0



 a+b−c=0
a−b−d=1

Rezolvând sistemul obţin a = 1/4, b = −1/4, c = 0, d = −1/2 deci are loc


 
1 1 1 1 2
(x−1)(x+1)(x2 +1) = 4 x−1 − x+1 − x2 +1

adică
x4
Z Z Z Z  
1 1 1 1 2
dx = dx + dx = x + − − dx
x4 − 1 (x − 1) (x + 1) (x2 + 1) 4 x − 1 x + 1 x2 + 1
1
=x+ 4 (ln (x − 1) − ln (x + 1) − 2arctgx) + C

b)
1 1 a bx+c
x3 +1 = (x+1)(x2 −x+1) = x+1 + x2 −x+1

⇔ 1 = a x2 − x + 1 + (x + 1) (bx + c)
⇔ 1 = (a + b) x2 + (−a + b + c) x + (a + c)

 a+b=0
⇔ −a + b + c = 0
a+c=1

Rezolvând sistemul obţin a = 1/3, b = −1/3, c = 2/3 deci are loc


1 1/3 −1/3x + 2/3
= + 2
(x + 1) (x2 − x + 1) x+1 x −x+1
adică
 
x−2 x−2
Z Z Z
1 1 1 1 1
dx = − dx = ln (x + 1) − dx
x3 + 1 3 x + 1 x2 − x + 1 3 3 x2 − x + 1

Acum avem x2x−2 x 2


R R R
−x+1 dx = x2 −x+1 dx − x2 −x+1 dx. Pentru acestea două se vor face calcule
standard. Mai ı̂ntâi, pentru prima, se formează la numărător derivata numitorului adică
1 2x − 1 + 1
Z Z Z
x 1 2x
2
dx = 2
dx = dx
x −x+1 2 x −x+1 2 x2 − x + 1
0
2x − 1 x2 − x + 1
Z Z Z Z
1 1 1 1 1 1
= dx + dx = dx + dx
2 x2 − x + 1 2 x2 − x + 1 2 x2 − x + 1 2 x2 − x + 1
Z Z
1  1 1 1  1 1
= ln x2 − x + 1 + 2 dx = ln x 2
− x + 1 + √ 2 dx
2 2 (x − 1/2) + 3/4 2 2 2
(x − 1/2) + 3/2
= 12 ln x2 − x + 1 + 12 √3/2 1
arctg x−1/2


3/2
+ C.

9
1 1 1 a b c
c) x3 −2x2 +x = x(x2 −2x+1) = x(x−1)2
= x + x−1 + (x−1)2
unde a, b, c trebuie determinaţi.

d) Rădăcinile ı̂ntregi ale lui x3 + 6x2 + 11x + 6 = 0 se găsesc printre divizorii termenului liber.

14. Să se calculeze următoarele integrale din funcţii raţionale:


Z Z Z Z
1 1 xdx dx
a) 2 dx , b) 2 3 dx , c) 2 , d) 2,
(x3 − 1) (x − 1) (x + 1) (x − 1) (x + 1) x (x + 1)

5x2 + 6x + 9
Z Z Z
dx dx
e) , f) 2 2 dx , g) .
(x2 − 4x + 3) (x2 + 4x + 5) (x − 3) (x + 1) (x + 1) (x + 2) (x + 3)

15. Să se calculeze următoarele integrale din funcţii iraţionale:


√ Z √
x−1
Z Z Z
x+1+2 1 1
a) 2 √ dx , b) √ dx , c) √ √
3
dx , d) √ dx
(x + 1) − x + 1 3
x + 1 x + 1 + x + 1 (2 − x) 1−x

Rezolvare:
   pq1   p2 
ax+b 1 ax+b q2
R
Fie integralele de forma R x, cx+d , cx+d , ... dx unde R este o expresie raţională.
Aceste integrale se reduc la integrale raţionale cu ajutorul substituţiei

ax + b
= ts
cx + d
unde s este cel mai mic multiplu comun al numitorilor q1 , q2 , ...

√ 1/2
a) Apare termenul x + 1 = (x + 1) deci este utilă substituţia

x + 1 = t2 ⇔ x = t2 − 1 ⇒ dx = 2tdt

deci integrala devine


Z √ Z
x+1+2 t+2
2 √ dx = 2 2tdt
(x + 1) − x + 1 (t2 ) − t
Z 2 Z Z
t + 2t t+2 t+2
=2 4
dt = 2 3
dt = 2 dt
t −t t −1 (t − 1) (t2 + t + 1)

şi am ajuns la integrala dintr-o funcţie raţională. Descompunem ı̂n fracţii simple

t+2 a bt + c
2
= + 2
(t − 1) (t + t + 1) t−1 t +t+1

cu a, b, c determinaţi aducând la acelaşi numitor şi identificând coeficienţii. Obţin a = 1.b = −1, c =
−1 şi integrala se reduce la integrale simple.
R t+2
R 1 t+1

I = 2 (t−1)(t 2 +t+1) dt = 2 t−1 − t +t+1 dt
2

 
= 2 ln (t − 1) − t2t+1
R
+t+1 dt

Mai ı̂ntâi Z Z Z
t+1 t 1
dt = dt + dt
t2 + t + 1 t2 + t + 1 t2 + t + 1
1/2
iar acestea se fac prin calcule standard. La sfârşit se va ı̂nlocui t = (x + 1) .
√ √
b) Apare x = x1/2 şi 3 x = x1/3 deci se va face substituţia x = t6 unde 6 este cel mai mic multiplu
comun al numitorilor 2 şi 3.

10
16. Să se calculeze următoarele integrale din funcţii iraţionale (integrale binome):
√ 3 Z p √
√ √ 3
Z Z Z
2 (1 + 3 x) 1+ 4x 1
a) 3
x 1 + x dx , b) √
dx , c) √ dx , d) √
3
dx ,
4 5
x
x x 1 + x5
Z Z Z
x dx dx
e) p √ dx , f ) √
4
, g) 5/3
1+ 3x 1 + x4 x (2 + x3 )
2

Rezolvare:
p
Fie integralele de forma xm (a + bxn ) dx unde m, n, p ∈ Q. Aceste integrale se reduc la integrale
R

raţionale doar ı̂n următoarele trei situaţii (cu ajutorul substituţiilor respective):

i) Dacă p este număr ı̂ntreg.


m+1
ii) Dacă este număr ı̂ntreg şi ı̂n acest caz este utilă substituţia a + bxn = ts unde s este
n
numitorul lui p.
m+1 a + bxn
iii) Dacă + p este număr ı̂ntreg şi ı̂n acest caz este utilă substituţia = ts unde s
n xn
este numitorul lui p.

√ √ 2 2
a) x (1 + 3 x) = x1/2 1 + x1/3 deci m = 1/2, n = 1/3, p = 2 deci suntem ı̂n prima situaţie şi,
evident, merge substituţia x = t6 ⇒ dx = 6t5 dt deci integrala devine
√ √
Z Z
2 1/2  1/3 2 5
3
x 1 + x dx = t6 1 + t6 6t dt
Z   2
Z
1/2 1/3 2
t6 1 + t6 6t5 dt = t3 1 + t2 6t5 dt
  
=
Z
2
= t3 1 + t2 6t5 dt

şi obţin integrala dintr-o funcţie polinomială.


√ 3
(1+ 3 x) 3
b) √
4
= x−5/4 1 + x1/3 deci m = −5/4, n = 1/3, p = 3 deci suntem ı̂n primul caz.
x5

3 √
4 1/3
1+ x
c) √
x
= x−1/2 1 + x1/4 deci m = −1/2, n = 1/4, p = 1/3 şi

m+1 −1/2 + 1
= =2∈Z
n 1/4
deci suntem ı̂n a doua situaţie şi merge substituţia
4 3
1 + x1/4 = t3 ⇔ x1/4 = t3 − 1 ⇔ x = t3 − 1 ⇒ dx = 4 t3 − 1 3t2 dt

deci integrala devine


Z p
3
√ Z 1/3
1+ 4x 
√ dx = x−1/2 1 + x1/4 dx
Z  x
4 −1/2 3 1/3
 3
= t3 − 1 t 12t2 t3 − 1 dt
Z Z
3
−2 2 3 3
t3 − 1 t3 dt

= 12 t −1 tt t − 1 dt = 12
Z
t6 − t3 dt = 12 t7 /7 − t4 /4 + C
 
= 12

1/3
şi acum se ı̂nlocuieşte t cu 1 + x1/4 .
d), e) Suntem ı̂n situaţia ii).

f ), g) Suntem ı̂n situaţia iii).

11
17. Să se calculeze următoarele integrale din funcţii trigonometrice:

sin3 x
Z Z Z Z
1 1
a) sin2 x cos3 xdx , b) dx , c) dx , d) dx ,
4
cos x sin x + tgx 1 + sin2 x
Z Z
1 1
e) 4
dx , f ) dx
cos x 1 + sin x + cos x

Rezolvare:
R
Fie integralele de forma R (sin x, cos x) dx unde R (a, b) este o expresie raţională ı̂n a şi b. Aceste
integrale se reduc la integrale raţionale cu ajutorul următoarelor substiţituţii:
i) Dacă R (− sin x, cos x) = −R (sin x, cos x) atunci este utilă substituţia cos x = t.
ii) Dacă R (sin x, − cos x) = −R (sin x, cos x) atunci este utilă substituţia sin x = t.
iii) Dacă R (− sin x, − cos x) = R (sin x, cos x) atunci este utilă substituţia tgx = t.
iv) Substituţia universală tg x2 = t.

În cazul integralelor din funcţii trigonometrice sunt utile următoarele formule trigonometrice
sin2 x + cos2 x = 1,
sin x cos x = sin 2x
2 , sin2 x = 1−cos 2x
2 , cos2 x = 1+cos 2x
2 ,
2
2t 1−t
sin x = 1+t2 , cos x = 1+t2 , unde t = tg x2 ,
sin x = √ t , cos x = √ 1 , unde t = tgx .
1+t2 1+t2

a) Avem că R (sin x, cos x) = sin2 x cos3 x deci


3
R (sin x, − cos x) = sin2 x (− cos x) = − sin2 x cos3 x = −R (sin x, cos x)
adică suntem ı̂n cazul ii). Este utilă substituţia sin x = t
sin2 x cos3 xdx = sin2 x cos2 x cos xdx
R R

0
= sin2 x cos2 x (sin x) dx = sin2 x 1 − sin2 x d (sin x)
R R 

0
deci sin x = t ⇒ dt = d (sin x) = (sin x) dx adică
Z Z
2 2
t2 − t4 dt = t3 /3 − t5 /5
 
I = t 1 − t dt =

unde t trebuie ı̂nlocuit cu sin x.


sin3 x
b) R (sin x, cos x) = cos4 x este impară ı̂n sin x, deci cazul i)
1
c) R (sin x, cos x) = sin x+tgx . În acest caz vom folosi substituţia universală (ı̂n cazul integralelor
trigonometrice):
x x 2
= t ⇔ = arctgt ⇔ x = 2arctgt ⇒ dx =
tg dt
2 2 1 + t2
Deci, folosind şi formulele trigonometrice respective, are loc
Z Z Z
1 1 1 2
dx = sin x
dx = 2t 2
dt
sin x + tgx sin x + cos x 2t 1+t2 1 + t
1+t2 + 1−t2
1+t2
Z Z
1 2 1 2
= 2t 2t dt =   dt
1+t2 + 1−t2
1 + t2 2t 1−t2 +1+t2 1 + t2
(1−t2 )(1+t2 )

1 − t2
Z Z Z Z
1 1 t 1 1
= 2 dt = dt = dt − dt = ln t − t2 + C
t 1−t2 2t 2t 2 2 4
1
2
ln tg x2 − 1
tg x2 + C.

= 2 4

12
d) e) Suntem ı̂n cazul iii).

f ) Suntem ı̂n cazul iv).


18. Să se calculeze următoarele integrale folosind metoda de integrare prin părţi:
Z Z Z
a) arctgx dx , b) xarctgx dx , c) x2 arctgx dx
Z Z Z
d) arcsin x dx, e) x arcsin x dx , f) arcsin2 x dx.

19. Să se calculeze următoarele integrale:


Z Z Z Z
dx xdx xdx dx
a) , b) , c) 2 , d) 2 ,
x2 + a2 x2 + a2 2
(x + a )2 (x + a2 )
2

Z Z Z
xdx 1 1
e) , f ) dx , g) dx , n ∈ {1, 2, 3, 4, ...} .
(x2 + a2 )
3
(x2 + a2 )
3 sinn x
Rezolvare:
d)

a2
Z 2
x + a2 − x2
Z Z
1 1 1
2 dx = 2 dx = 2 dx
(x2 + a2 ) a2 (x2 + a2 ) a2 (x2 + a2 )
x2 + a2 x2
Z Z Z Z
1 1 1 1 1 x
= 2 2 dx − 2 2 dx = 2 2 2
dx − 2
x 2 dx
a 2
(x + a ) 2 a 2
(x + a )2 a x +a a (x + a2 )
2
Z  0  
1 −1
Z
1 1 x 1 1 x 1 1 1
= 2 arctg − 2 x dx = arctg + x − 1 dx
a a a a 2 x2 + a2 a3 a 2a2 x2 + a2 x2 + a2
 Z   
1 x 1 1 1 1 x 1 1 1 x
= 3 arctg + 2 x 2 − 1 2 dx = 3 arctg + 2 x 2 − arctg + C.
a a 2a x + a2 x + a2 a a 2a x + a2 a a

g) Pentru n = 1 :
Z Z Z Z
1 sin x 1 0 1
dx = 2 dx = 2
(− cos x) dx = (subst. cos x = t) = 2
dt
sin x sin 1 − cos x t −1
x
1 t − 1 1 cos x − 1
= ln + C = ln + C.
2 t+1 2 cos x + 1

sau, folosind substituţia universală


x x 2
tg = t ⇔ = arctgt ⇔ x = 2arctgt ⇒ dx = dt
2 2 1 + t2
obţinem Z Z Z
1 1 2 1  x 
dx = dt = dt = ln |t| + C = ln tg + C.

sin x 2t 1 + t2 t 2
1+t2

Pentru n = 2, folosind tabelul obţinem:


Z
1
dx = −ctgx + C
sin2 x
sau, folosind substituţia universală, obţinem

1 + t2 1 t2 − 1
 
1 −1
Z Z Z
1 1 2 1
2 dx = 2 2
dt = 2
dt = + t +C = +C
sin x 1+t 2 t 2 t 2 t

2t
1+t2
2
1 tg x2 − 1
= + C = −ctgx + C.
2 tg x2

13
Pentru n = 3 :
−1
Z Z Z Z
1 sin x 1 0
dx = dx = 2 (− cos x) dx = (subst. cos x = t) = 2 dt
sin3 x sin4 x 2
(1 − cos x) (1 − t2 )
Z !
a b c d
= − + + + dt.
1 − t 1 + t (1 − t)2 (1 + t)
2

sau, folosind substituţia universală, obţinem


2
1 + t2
Z Z Z
1 1 2 1
dx = dt = dt.
sin3 x
3 2
1+t 4 t3

2t
1+t2

20. Să se calculeze următoarele integrale:


Z Z Z
2x + 1 2x + 1 1
a) 2
dx , b) 2 dx , c) 2
dx ,
x +x+1 (x2 + x + 1) x +x+1
Z Z
1 x
d) 2 dx , e) 2 dx .
2
(x + x + 1) 2
(x + x + 1)
21. Să se calculeze aria figurii plane cuprinsă ı̂ntre curbele (date explicit) y 2 = 2px şi x2 = 2py.
Să se particularizeze pentru p = 1/2.
Rezolvare: Dacă suntem ı̂n cazul ı̂n care curbele care dau domeniul sunt date explicit iar domeniul
este deci D = {(x, y) : a ≤ x ≤ b , f1 (x) ≤ y ≤ f2 (x)} atunci aria domeniului D este dată de
Z b
A (D) = [f2 (x) − f1 (x)] dx
a

22. Să se calculeze volumul sferei şi volumul elipsoidului (acestea se obţin prin rotaţia unui semicerc şi
respectiv a unei semielipse ı̂n jurul axei Ox).
Rezolvare: Dacă volumul V ⊂ R3 este obţinut prin rotaţia mulţimii F = {(x, y) : a ≤ x ≤ b , 0 ≤ y ≤ f (x)}
atunci volumul este dat de Z b
V (F ) = π f 2 (x) dx
a

În cazul nostru sfera este dată de rotaţia domeniului (semidiscului)


n p o
F = (x, y) : −r ≤ x ≤ r, 0 ≤ y ≤ r2 − x2

respectiv dat de rotaţia domeniului (semielipsei)


 
bp 2 2
F = (x, y) : −a ≤ x ≤ a, 0 ≤ y ≤ a −x .
a

23. Să se determine volumul corpului de rotaţie dat de f : [0, 1/2] → R , f (x) = arcsin x.

24. Să se determine lungimea graficului funcţiei f : [3, 8] → R , f (x) = 32 x x.
Rezolvare: Dacă suntem ı̂n cazul ı̂n care curba este dată explicit de (C) : y = f (x) , a ≤ x ≤ b atunci
lungimea curbei este dată de
Z bq
2
L (C) = 1 + (f 0 (x)) dx.
a

25. Să se determine lungimea graficului funcţiei f : [π/3, π/2] → R , f (x) = ln (cos x).

14
x = a cos3 t

26. Să se determine lungimea curbei dată parametric , t ∈ [0, π/2].
y = a sin3 t
Rezolvare:
 Dacă suntem ı̂n cazul ı̂n care curba este dată curba este ı̂n plan şi este dată parametric de
x = x (t)
(C) : , a ≤ t ≤ b atunci lungimea curbei este dată de
y = y (t)
Z b q
2 2
L (C) = (x0 (t)) + (y 0 (t)) dt
a



 x = a cos t

27. Să se determine lungimea curbei din spaţiu dată parametric y = a sin t , t ∈ [0, π] .


z = ct



 x = x (t)

Rezolvare: În cazul ı̂n care (C) : y = y (t) , a ≤ t ≤ b (adică ı̂n cazul ı̂n care curba este dată


z = z (t)

curba este ı̂n spaţiu şi este dată parametric) lungimea curbei este dată de
Z b q
2 2 2
L (C) = (x0 (t)) + (y 0 (t)) + (z 0 (t)) dt.
a

28. Să se determine aria discului.

29. Să se determine lungimea cercului.

15
Facultatea de Matematică
Calcul integral şi aplicaţii, Semestrul I
Lector dr. Lucian MATICIUC

Seminar recapitulativ. Anexă

Derivatele funcţiilor elementare


0
1. c =0
0
2. x =1
0
3. (xn ) = nxn−1 , n ∈ N
0
4. (xa ) = axa−1 , a ∈ R+
√ 0 1
5. ( x) = √ (obţinută ı̂n particular pentru a = 1/2)
2 x
 0
1 −1
6. = 2 (obţinută ı̂n particular pentru a = −1)
x x
0
7. (ax ) = ax ln a , a ∈ R+ , a 6= 1
0
8. (ex ) = ex (obţinută ı̂n particular pentru a = e)
0 1
9. (ln x) =
x
0
10. (sin x) = cos x
0
11. (cos x) = − sin x
1
0
12. (tg x) =
cos2 x
0 −1
13. (ctg x) =
sin2 x
1 0
14. (arcsin x) = √
1 − x2
0 −1
15. (arccos x) = √
1 − x2
0 1
16. (arctg x) =
1 + x2
0 −1
17. (arcctg x) =
1 + x2
0 ex − e−x
def
18. (sh x) = ch x , unde sh x = este sinusul hiperbolic
2
x −x
0 def e + e
19. (ch x) = sh x unde ch x = este cosinusul hiperbolic
2

16
Derivatele funcţiilor compuse
0 0
1. (un ) = nun−1 · u , n ∈ N
0 0
2. (ua ) = aua−1 · u , a ∈ R+
√ 0 1 0
3. ( u) = √ · u (obţinută ı̂n particular pentru a = 1/2)
2 u
 0
1 −1 0
4. = 2 · u (obţinută ı̂n particular pentru a = −1)
u u
0 0
5. (au ) = au ln a · u , a ∈ R+ , a 6= 1
0 0
6. (eu ) = eu · u (obţinută ı̂n particular pentru a = e)
0 1 0
7. (ln u) = ·u
u
0 0
8. (sin u) = cos u · u
0 0
9. (cos u) = − sin u · u
0 1 0
10. (tg u) = 2
·u
cos u
0 −1 0
11. (ctg u) = ·u
sin2 u
0 1 0
12. (arcsin u) = √ ·u
1 − u2
0 −1 0
13. (arccos u) = √ ·u
1 − u2
0 1 0
14. (arctg u) = ·u
1 + u2
0 −1 0
15. (arcctg u) = 2
·u
1+u
0
e − e−u
 u
0 0
16. (sh u) = = ch u · u
2
0
e + e−u
 u
0 0
17. (ch u) = = sh u · u
2

Operaţii cu funcţii derivabile


0
1. (f + g) (x) = f 0 (x) + g 0 (x)
0
2. (C · f ) (x) = C · f 0 (x)
0
3. (f · g) (x) = f 0 (x) · g (x) + f (x) · g 0 (x)
 0
1 −1 0
4. (x) = 2 g (x)
g g (x)
 0
f f 0 (x) · g (x) − f (x) · g 0 (x)
5. (x) =
g g 2 (x)

17
Integrale nedefinite
Z
1. dx = x + C

xα+1
Z
2. xα dx = + C , α ∈ R , α 6= −1
α+1
Z
1
3. dx = ln |x| + C
x
Z
1 1 x
4. 2 2
dx = arctg + C , a 6= 0
x +a a a

x − a + C , a 6= 0
Z
1 1
5. 2 2
dx = ln
x −a 2a x + a
Z
1 p
6. √ dx = ln x + x2 ± a2 + C , a 6= 0

x2 ± a2
Z
1 x
7. √ dx = arcsin + C , a > 0
2
a −x 2 a
Z x Z
a
8. ax dx = + C , a > 0, a 6= 1 , ex dx = ex + C
ln a
Z
9. sin xdx = − cos x + C
Z
10. cos xdx = sin x + C
Z
1
11. dx = tg x + C
cos2 x
Z
1
12. dx = − ctg x + C
sin2 x
Z
1 x
13. dx = ln tg + C

sin x 2
Z
1  x π 
14. dx = ln tg + +C

cos x 2 4
Z
15. tg xdx = − ln |cos x| + C
Z
16. ctg xdx = ln |sin x| + C

Metode de calcul
1. Formula de integrare prin părţi pentru integrala definită
Z b b Z b
0 0
f (x) g (x) dx = [f (x) g (x)] − f (x) g (x) dx.

a a a

Z b
0
2. Prima metodă de schimbare de variabilă pentru integrala definită: pentru a calcula f (u (x)) u (x) dx
a
not 0
se notează y = u (x) deci dy = u (x) dx şi are loc
Z b Z u(b) u(b)
0
f (u (x)) u (x) dx = f (y) dy = F (y) = F (u (b)) − F (u (a)) .

a u(a) u(a)

18
Facultatea de Matematică
Calcul Integral şi Aplicaţii, Semestrul I
Lector dr. Lucian MATICIUC

Seminariile 1 - 3

Capitolul I. Integrale improprii

Z ∞
1
1. Să se studieze natura integralei dx.
0 1 + x2

Rezolvare:
1
Observăm că se integrează funcţia f : [0, ∞) → R, f (x) = 1+x 2 care este continuă pe orice interval

de tipul [0, b], cu ∀ b > 0 deci este integrabilă pe orice interval compact [0, b] ⊂ [0, ∞). Atunci, prin
definiţie
Z ∞ Z b
1 1
2
dx = lim dx
0 1+x b→∞ 0 1 + x2
R 1
Dar 1+x 2 dx = arctgx deci

Z b
1 b π
lim 2
dx = lim arctg (x)|0 = lim (arctg (b) − arctg (0)) = lim arctg (b) = arctg (∞) =
b→∞ 0 1 + x b→∞ b→∞ b→∞ 2
R∞ 1
ceea ce ı̂nseamnă că integrala improprie de primul tip 0 1+x 2 dx este convergentă şi

Z ∞
1 π
dx =
0 1 + x2 2
Z 0
1
2. Să se studieze natura integralei dx.
−∞ 1 + x2

Rezolvare:
1
Observăm că se integrează funcţia f : (−∞, 0] → R, f (x) = 1+x 2 care este continuă pe orice interval

de tipul [−b, 0], cu ∀ b > 0 deci este integrabilă pe orice interval compact [−b, 0] ⊂ (−∞, 0]. Atunci,
prin definiţie
Z 0 Z 0
1 1
2
dx = lim dx
−∞ 1 + x b→∞ −b 1 + x2
Deci
Z 0
1 0 π
2
dx = lim arctg (x)|−b = lim (arctg (0) − arctg (−b)) = lim arctg (b) = arctg (∞) =
−∞ 1+x b→∞ b→∞ b→∞ 2
R0 1
ceea ce ı̂nseamnă că integrala improprie de primul tip −∞ 1+x2
dx este convergentă şi
Z 0
1 π
dx =
−∞ 1 + x2 2
Z ∞
1
3. Să se studieze natura integralei dx.
−∞ 1 + x2

Rezolvare:

1
Prin definiţie
Z ∞ Z 0 Z ∞
1 1 1 π π
dx = dx + dx = + = π
−∞ 1 + x2 −∞ 1 + x2 0 1+x 2 2 2
R∞ 1
Deci integrala improprie de primul tip −∞ 1+x2
dx este convergentă şi
Z ∞
1
dx = π
−∞ 1 + x2

4. Fie a > 0. Integrala Z ∞


1
dx
a xα
este convergentă pentru α > 1 şi divergentă pentru α ≤ 1.
Rezolvare:
1
Observăm că se integrează funcţia f : [a, ∞) → R, f (x) = care este continuă pe orice interval

de tipul [a, b], cu ∀ b > a deci este integrabilă pe orice interval compact [a, b] ⊂ [a, ∞). Atunci, prin
definiţie
Z ∞ Z b Z b
1 1
dx = lim dx = lim x−α dx
a xα b→∞ a xα b→∞ a

Dar 
−α+1
Z  x

, α 6= 1
x−α dx = −α + 1

 ln x, α=1
deci, pentru α = 1,
Z b
b
lim x−1 dx = lim ln x|a = lim (ln b − ln a) = ∞ − ln a = ∞
b→∞ a b→∞ b→∞

şi pentru α 6= 1,
b b
x−α+1
Z
−α 1
b1−α − a1−α =

lim x dx = lim = lim
b→∞ a b→∞ −α + 1 a b→∞ 1 − α
 
1 1−α 1−α 1
∞1−α − a1−α

= lim b −a =
1−α b→∞ 1−α
1 1
Dacă avem α > 1 atunci α − 1 > 0 deci ∞1−α = ∞α−1 = ∞ = 0, iar dacă avem α < 1 atunci
1 − α > 0 deci ∞1−α = ∞.
Integrala iniţială este atunci
 b
x−α+1 ( 1 1−α
, α 6= 1

∞  lim
α−1 a , α>1
Z 
1 b→∞ −α + 1 a
α
dx = =
a x ∞, α≤1
 lim ln x|b ,

α=1

a
b→∞

R∞ 1 1 1−α
deci a xα
dx = α−1 a deci este (C) pentru α > 1 şi (D) pentru α ≤ 1.

5. Să se studieze, folosind definiţia, convergenţa următoarelor integrale improprii de specia I:


Z ∞ Z ∞ x Z ∞ Z ∞
dx e dx −2x
(a) , (b) , (c) e sin 3x dx , (d) e−3x cos 4x dx,
e x (ln x)
3/2
1 ex − 1 0 0

Z ∞ Z ∞
dx 2
(e) 2 , (f ) xe−x dx ,
0 (1 + x) 0

2
Z ∞ Z ∞
(g) e−αx sin (βx) dx , α > 0, (h) e−αx cos (βx) dx , α > 0.
0 0

Rezolvare:

(a)
∞ ∞ y=∞ y=∞
y −1/2
Z Z
1 −3/2 1
3/2
dx = (subst. ln x = y) = y dy = = −2 √ = 2.
e x (ln x) 1 −1/2 y=1 y y=1

(b)
∞ ∞
ex
Z Z
1 y=∞
x
dx = (subst. ex = y) = dy = ln |y − 1| |y=e = ln (+∞) − ln |e − 1| = +∞.
1 e −1 e y−1

(c) aplicăm de două ori metoda de integrare prin părţi pentru a calcula primitiva

−1 −2x
Z
3 −2x 33
F (x) = e−2x sin 3x dx = e sin 3x + e cos 3x − F (x)
2 −4 22
 
13 −1 −2x 3 −2x
⇔ F (x) = e sin 3x + e cos 3x
4 2 −4
2 sin 3x + 3 cos 3x −2x
⇔ F (x) = − ·e .
22 + 32
Deci Z ∞
x=+∞
e−2x sin 3x dx = −F (x) |x=0 = F (+∞) − F (0) .
0
3
Dar F (0) = − 22 +3 2 iar

def 2 sin 3x + 3 cos 3x −2x


F (+∞) = lim (−) ·e =0
x→+∞ 22 + 32
deoarece

2 sin 3x + 3 cos 3x 2 sin 3x + 3 cos 3x |2 sin 3x| + |3 cos 3x| 2+3
− = ≤ ≤ 2
22 + 32 22 + 32 |22 + 32 | 2 + 32

(adică este mărginit) iar


lim e−2x = e−∞ = 0
x→+∞

Am folosit rezultatul:

Lema 1 Fie f, g : I → R unde I este un interval. Presupunem că limx→a f (x) = 0 şi |g (x)| ≤ M , ∀x ∈ I.
Atunci
lim [f (x) · g (x)] = 0
x→a

(adică produsul dintre o cantitate care tinde la zero şi o cantitate mărginită este o cantitate care tinde la zero).

Prin urmare Z ∞
3
e−2x sin 3x dx = F (+∞) − F (0) = .
0 22 + 3 2
(f )

−1 −x2 x=∞ −1 −∞ 1 0
Z
2 1
xe−x dx = e = e + e = .
2 2 2 2

0 x=0

(g) Observăm că se integrează funcţia f : [0, ∞) → R, f (x) = e−αx sin (βx) care este continuă pe
orice interval de tipul [0, b], cu ∀ b > 0 deci este integrabilă pe orice interval compact [0, b] ⊂ [0, ∞).
Atunci, prin definiţie
Z ∞ Z b
−αx
e sin (βx) dx = lim e−αx sin (βx) dx
0 b→∞ 0

3
Rb
Dar pentru calculul integralei 0
e−αx sin (βx) dx (vezi metoda de integrare prin părţi):
Z
α sin (βx) + β cos (βx) −αx
e−αx sin (βx) dx = − ·e
α2 + β 2

adică primitiva F (x) = − α sin(βx)+β cos(βx)


α2 +β 2
β
· e−αx . Obţinem F (0) = − α2 +β 2 şi deci

Z ∞
β
e−αx sin (βx) dx = F (∞) − F (0) = lim F (x) +
0 x→∞ α2 + β2

Ţinând cont de faptul că funcţiile sin, cos sunt mărginite de 1 obţinem că

α sin (βx) + β cos (βx) |α sin (βx)| + |β cos (βx)| |α| + |β|
− ≤ ≤ 2
2 2
2
α +β 2
α +β α + β2

şi pe de altă parte lim e−αx = e−∞ = 1


e∞ = 1
∞ = 0. Dar având ı̂n vedere că produsul dintre o
x→∞
funcţie mărginită şi o funcţie care tinde la 0 va tinde la 0 obţinem că
 
α sin (βx) + β cos (βx)
lim F (x) = lim − · e−αx = 0
x→∞ x→∞ α2 + β 2

deci Z ∞
β β
e−αx sin (βx) dx = F (∞) − F (0) = 0 + 2 = 2
0 α2 +β α + β2
(h) Avem Z ∞
α
e−αx cos (βx) dx = .
0 α2 + β 2

6. Să se studieze convergenţa următoarei integrale improprii


Z ∞ √ 3
x
dx
0 1 + x2

Rezolvare:

x3 x3/2
Avem f (x) = 1+x2 = 1+x2 . Trebuie să determinăm α astfel ı̂ncât să existe limita lim xα f (x).
x→∞

x3/2 xα+3/2
 
limx→∞ xα f (x) = lim xα · 2
= lim
x→∞ 1+x x→∞ 1 + x2

Având ı̂n vedere că limita este la ∞, scoatem factor forţat x la puterea cea mai mare

xα+3/2 1 1
limx→∞ xα f (x) = lim = (aleg α + 3/2 = 2) = lim = =1
x→∞ x2 (1/x2 + 1) x→∞ 1/x2 + 1 0+1
R∞ √
x3
Deci pentru α = 1/2 ≤ 1 obţin limx→∞ xα f (x) = 1 ∈ (0, ∞) deci 0 1+x2 dx este (D).
7. Să se studieze convergenţa următoarelor integrale improprii∗ :
Z ∞ Z ∞ Z ∞
arctg (x) arctg (x) arctg (x)
(a) 2
dx , (b) dx, (c) dx,
1 x 0 x 0 x (1 + x2 )
Z ∞ Z ∞
2
(d) e−x −2x+3
dx, (e) e−ax xb dx, a, b > 0.
−1 0

Rezolvare:
∗ La punctele (b) şi (c) putem considera intergala calculată pe intervalul [0, +∞) deoarece limita ı̂n 0 există şi este finită,
arctg(x) arctg(x)
limx→0 = 1 şi limx→0 = 1 . Deci integrala este improprie de specia I.
x x(1+x2 )

4
arctg(x)
(a) Avem f (x) = x2 . Trebuie să determinăm α astfel ı̂ncât să existe limita lim xα f (x).
x→∞
 
arctg (x) π
limx→∞ xα f (x) = limx→∞ xα · = (aleg α = 2) = limx→∞ arctg (x) = .
x2 2
π
R∞ arctg(x)
Deci pentru α = 2 > 1 obţin lim xα f (x) = 2 ∈ (0, ∞) deci 1 x2 dx este (C).
x→∞
arctg(x)
(b) Avem f (x) = x . Trebuie să determinăm α astfel ı̂ncât să existe limita lim xα f (x).
x→∞
 
α α arctg (x) π
limx→∞ x f (x) = limx→∞ x · = (aleg α = 1) = limx→∞ arctg (x) = .
x 2
π
R∞ arctg(x)
Deci pentru α = 1 ≤ 1 obţin lim xα f (x) = 2 ∈ (0, ∞) deci 1 x dx este (D).
x→∞
(e) Se obţine convergenţa cu α > 1, arbitrar ales.
8. Să se studieze, folosind criteriul de convergenţă ı̂n α, convergenţa următoarelor integrale improprii:
Z ∞ Z ∞
x 1
(a) √ dx , (b) √ dx ,
0
5
x +1 0 2x + 5 + 3 x2 + 1
Z ∞ Z ∞ Z ∞
1 dx dx
(c) √ dx , (d) √
3
, (e) p , a > 2.
1 x 1 + x4 0 1 + x3 + x6 a x (x − 1) (x − 2)

9. Să se studieze convergenţa următoarelor integrale improprii:


Z ∞ Z ∞ Z ∞
1 2 −|x| x
(a) 2
dx , (b) x e dx , (c) 2
dx .
−∞ x − 2x + 3 −∞ −∞ 1 + x

Rezolvare:

Se scrie fiecare integrală ca sumă de alte două


 integrale.
Z a Eventual se poate folosi şi paritatea funcţiei
Z a  2 f (x) dx , f este funcţie pară

de sub integrală şi faptul că f (x) dx = 0
−a 
0 , f este funcţie impară.

10. Să se studieze convergenţa următoarelor integrale improprii (calculându-le, eventual, ı̂n prealabil):
Z ∞ Z ∞ Z ∞ √ Z ∞
dx ln x x x3 dx
(a) , (b) dx , (c) 2 dx , (d) 2 ,
0 1 + x3 1 x3 1 (1 + x) 0 (1 + x3 )
Z ∞ Z ∞ Z ∞
x ln x dx dx
(e) 2 dx , (f ) √ , (g) .
1
2
(1 + x ) 1 x 1+x 2
0 1 + x4

Rezolvare:

(a) Mai ı̂ntâi,

1 a bx + c
= + ⇔ 1 = (a + b) x2 + (−a + b + c) x + (a + c)
1 + x3 1 + x 1 − x + x2
⇔ a = 1/3, b = −1/3, c = 2/3

5
şi deci
−x + 2 2x − 4
Z Z Z Z
dx 1 1 1 1 1 1
3
= dx + 2
dx = ln |1 + x| + dx
1+x 3 1+x 3 1−x+x 3 3 −2 1 − x + x2
2x − 1 − 3
Z
1 1 1
= ln |1 + x| + dx
3 3 −2 1 − x + x2
0
1 − x + x2 −3
Z Z
1 1 1
= ln |1 + x| − 2
dx − dx
3 6 1−x+x 6 1 − x + x2
Z
1 1 1 1
= ln |1 + x| − ln 1 − x + x2 +  √ 2 dx =
3 6 2 2
x − 21 + 23


1 1 1 1 x − 1/2
= ln |1 + x| − ln 1 − x + x2 + √ arctg √ +C
3 6 2 3/2 3/2
1 1 x − 1/2
2 ln |1 + x| − ln 1 − x + x2 + √ arctg √

= +C
6 3 3/2
2
1 |1 + x| 1 x − 1/2 1 x2 + x + 1 1 x − 1/2
= ln + arctg √ = ln 2 + √ arctg √ +C .
6 |1 − x + x2 | 2 3/2 6 x − x + 1 3 3/2

Integrala improprie este atunci


Z ∞ x=∞
dx 1 x2 + x + 1 x=∞ 1 x − 1/2
= ln 2 + √ arctg √
1 + x3 6 x − x + 1 x=0

0 3 3/2 x=0
1 x2 + x + 1 1 02 + 0 + 1 1 x − 1/2 1 0 − 1/2
= lim ln 2 − ln 2 + √ lim arctg √ − √ arctg √
6 x→∞ x − x + 1 3 0 − 0x + 1 3 x→∞ 3/2 3 3/2
1 1 1 1 −1/2 1 π 1 1
= ln 1 − ln 1 + √ arctg (∞) − √ arctg √ =√ + √ arctg √
3 3 3 3 3/2 32 3 3
π π 2π
= √ + √ = √ .
2 3 6 3 3 3
(b) Calculăm mai ı̂ntâi primitiva
0
x−2 x−2 x−2 1 x−2
Z Z Z  Z Z
ln x −3 1
dx = x ln xdx = ln xdx = ln x − dx = ln x + x−3 dx
x3 −2 −2 −2 x −2 2
1 ln x 1 x−2
= + + C.
−2 x2 2 −2

Integrala improprie este atunci


Z ∞
ln x −1 ln x x=∞ 1 1 x=∞
dx = −
x3 2 x2 x=1 4 x2 x=1

1
−1 ln x −1 ln 1 1 1 1 1 1
= lim 2 − 2
− lim 2 + 2
= .
2 x→∞ x 2 1 4 x→∞ x 41 4
deoarece
ln x 1/x
lim
= (L’Hospital) = lim = 0.
x2 x→∞ x→∞ 2x

(c) Calculăm mai ı̂ntâi primitiva făcând substituţia



x = t2 ⇔ t = x ⇒ dx = 2tdt

(vezi Integrale din funcţii iraţionale):


Z √ Z
x t
2 dx = 2 2tdt.
(1 + x) (1 + t2 )

6
Dar
0 −1 !0 0
1 1 + t2 1 + t2

t 1 2 −2
0 1 1 1
1 + t2 =

2 = 2 2 = 2 1+t =−
2
(1 + t ) 2
(1 + t ) 2 −1 2 1 + t2

deci
√ 0    Z  
−1 1 −1 1 −1 1
Z Z
x
2 dx = 2 dt
t = 2t − 2 1 dt
(1 + x) 2 1 + t2 2 1 + t2 2 1 + t2

t x √
=− 2
+ arctgt + C = − + arctg x + C.
1+t 1+x
Integrala improprie este atunci
Z ∞ √ √ x=∞
x x √ x=∞
2 dx = − + arctg x
1 + x x=1

1 (1 + x) x=1
√ √
x 1 √ √ 1 π π 1 π
= − lim + + lim arctg x − arctg 1 = + − = + .
x→∞ 1 + x 1+1 x→∞ 2 2 4 2 4
(d) Observăm mai ı̂ntâi că
0 −1 !0 0
x2 1 1 + x3 1 + x3

1 3 −2
0 1 1 1
1 + x3 =

2 = 3 2 = 3 1+x =−
3
(1 + x ) 3
(1 + x ) 3 −1 3 1 + x3

deci
0
x3 x2
Z Z Z    Z  
1 1 1 1 1 1
2 dx = x 2 dx = x − dx = x − − 1 − dx
(1 + x3 ) (1 + x3 ) 3 1 + x3 3 1 + x3 3 1 + x3
Z
1 x 1 1
=− + dx
3 1 + x3 3 1 + x3

Integrala improprie este atunci (vezi şi punctul a))



x3 dx 1 x x=∞ 1 ∞ 1
Z Z
2 = − 3 1 + x3 +
3 0 1 + x3
dx
0 (1 + x3 ) x=0

1 x 1 0 1 2π 2π
= − lim 3
+ 3
+ √ = √ .
3 x→∞ 1 + x 31+0 33 3 9 3
(e) Observăm mai ı̂ntâi că
0 −1 !0 0
1 1 + x2 1 + x2

x 1 −2 0 1 1 1
2 = 2 = 1 + x2 1 + x2 = =−
(1 + x2 ) 2 (1 + x2 ) 2 2 −1 2 1 + x2

deci
Z Z  0   Z  
x ln x 1 1 1 1 1 1 1
2 dx = ln x − dx = ln x − − − dx
(1 + x2 ) 2 1 + x2 2 1 + x2 x 2 1 + x2
Z
1 ln x 1 1
=− + dx.
2 1 + x2 x (1 + x2 )
2
1
R
Pentru a calcula integrala x(1+x 2 ) dx , trebuie sa descompunem fracţia ı̂n fracţii simple:

1 a bx + c
2
= + ⇔ a = 1, b = −1, c = 0.
x (1 + x ) x 1 + x2

7
Deci
−x
Z Z Z
x ln x 1 ln x 1 1 1
2 2 dx = − 2 1 + x2 + 2 x
dx +
2 1 + x2
dx
(1 + x )
Z
1 ln x 1 1 2x 1 ln x 1 1
+ ln x − ln 1 + x2 + C

=− 2
+ ln x − 2
dx = − 2
21+x 2 4 1+x 21+x 2 4
1 ln x 1 2 1 2
 1 ln x 1 x2
=− + ln x − ln 1 + x + C = − + ln +C
2 1 + x2 4 4 2 1 + x2 4 1 + x2
Integrala improprie este atunci

x2 x=∞
Z
x ln x 1 ln x x=∞ 1
2 dx = − + ln
2 1 + x2 x=1 4 1 + x2 x=1

1 (1 + x2 )
1 ln x 1 ln 1 1 x2 1 12 1 1 1
=− lim 2
+ 2
+ lim ln 2
− ln 2
= − ln = ln 2,
2 x→∞ 1 + x 21+1 4 x→∞ 1 + x 4 1+1 4 2 4
deoarece
ln x 1/x
lim = ( L’Hospital ) = lim = 0,
x→∞ 1 + x2 x→∞ 2x
x2 x2
lim ln = ln lim = ln 1 = 0.
x→∞ 1 + x2 x→∞ 1 + x2

(g) Deoarece
 √  √ 
x4 + 1 = x4 + 2x2 + 1 − 2x2 = x2 + x 2 + 1 x2 − x 2 + 1 ,

putem descompune ı̂n fracţii simple şi obţinem


Z ∞ √  √  √   x=∞
x2 + x 2 + 1

dx 1 1  1
= √ ln √ + √ arctg x 2 + 1 + √ arctg x 2 − 1 .
0 1 + x4 4 2 x2 − x 2 + 1 2 2 2 2 x=0

11. Arătaţi că următoarele integrale improprii sunt divergente:


Z +∞ Z +∞ Z +∞
(a) cos x dx , (b) sin x dx , (c) x sin x dx .
0 0 0

Rezolvare:

(b) Observăm că se integrează funcţia f : [0, ∞) → R, f (x) = sin x care este continuă pe orice
interval de tipul [0, b], cu b > 0 arbitrar, deci este integrabilă pe orice interval compact [0, b] ⊂ [0, ∞).
Atunci, prin definiţie
Z ∞ Z b
sin xdx = lim sin xdx = lim (− cos b + cos 0) = 1 − lim cos b = 1 − cos ∞
0 b→∞ 0 b→∞ b→∞

R∞
Dar cos ∞ nu există (funcţiile periodice nu au limită la ∞) deci 0
sin xdx nu există adică
Z ∞
sin xdx este divergentă.
0

12. Să se studieze convergenţa integralelor


Z ∞ Z ∞
sin x cos x
dx şi dx , a > 0, α > 0.
a xα a xα

Rezolvare:

8
Integralele sunt (C) deoarece putem aplica Criteriul lui Dirichlet funcţiilor f (x) = sin x (respectiv
f (x) = cos x) şi g (x) = x1α .
Într-adevăr avem f (x) = sin x integrabilă pe orice [a, b] şi
Z
b
sin xdx = |− cos b + cos a| ≤ 2, ∀ a < b < ∞.


a

1
Evident g (x) = xα este descrescătoare (α > 0) şi

1
lim g (x) = =0
x→∞ ∞

În particular se obţine convergenţa integralelor


Z ∞ Z ∞
sin x cos x
dx, dx
0 x 1 x
sin x
(am putut extinde la cazul a = 0 deoarece limita ı̂n 0 este finită, limx→0 x = 1 ).
13. Să se studieze convergenţa integralelor:
Z ∞  Z ∞
sin x5 x sin (ax)
(a) 2
dx , (b) dx .
0 x 0 k 2 + x2

Rezolvare:

Ambele integrale sunt improprii de specia I.


(a) Avem două metode de abordare.
(a1 ) Studiem absoluta convergenţă a integralei date, adică studiem convergenţa integralei
Z ∞ 
5 Z ∞ 
sin x sin x5
dx = dx
x2 x2

0 0

folosind Criteriul ı̂n α.


Obţinem, pentru orice α ∈ (1, 2) ,

sin x5
α
limx→∞ x · f (x) = limx→∞ = 0 < +∞,
x2−α
R ∞ |sin(x5 )| R∞ sin(x5 )
deci integrala 0 x2 dx este (C) , adică 0 x2 dx este (AC) ,deci şi (C) .
(a2 ) Pentru a putea aplica Criteriul lui Dirichlet schimbăm mai ı̂ntâi variabila, y := x5 , deci x = y 1/5
şi
Z ∞ Z ∞
1 ∞ sin (y)

sin x5
Z
sin (y) 1 −4/5
dx = y dy = dy.
0 x2 0 y 2/5 5 5 0 y 6/5
Deci integrala este (C) deoarece putem aplica Criteriul lui Dirichlet funcţiilor f (y) = sin y şi g (y) =
1
.
y 6/5

(b) Integrala este (C) deoarece putem aplica Criteriul lui Dirichlet funcţiilor f (x) = sin (ax) şi
x
g (x) = 2 .
k + x2
14. Să se studieze absoluta convergenţă a integralelor:
Z ∞ Z ∞
sin (ax) cos (ax)
(a) dx , (b) dx.
0 k 2 + x2 0 k 2 + x2

Rezolvare:

9
Ambele integrale sunt improprii de specia I.
(a) Integrala este (C) deoarece putem aplica Criteriul lui Dirichlet funcţiilor f (x) = sin (ax) şi
1
g (x) = k2 +x 2 .
R∞
A studia absoluta convergenţă a integralei 0 f (x) dx ı̂nseamnă a studia convergenţa integralei
R∞
0
|f (x)| dx.
Avem
sin (ax) 1
k 2 + x2 ≤ k 2 + x2 ,

iar Z ∞
1
2 + x2
dx este (C) ,
0 k
R ∞
deci, conform unui criteriu de comparaţie, 0 sin(ax)
k2 +x2 dx este (C) .

R∞
În consecinţă, 0 sin(ax)
k2 +x2 dx este (AC) .
(b) Integrala este (C) deoarece putem aplica Criteriul lui Dirichlet funcţiilor f (x) = cos (ax) şi
1
g (x) = k2 +x 2 .

Similar, integrala este (AC) .


Z 1
1
15. Să se studieze natura integralei √ dx.
0 1 − x2
Rezolvare:
1
Observăm că se integrează funcţia f : [0, 1) → R, f (x) = √1−x 2
care este continuă pe orice interval
de tipul [0, c], cu ∀ 0 < c < 1 deci este integrabilă pe orice interval compact [0, c] ⊂ [0, 1). Atunci,
prin definiţie
Z 1 Z c
1 1 c
√ dx = lim √
dx = lim arcsin x|0 = lim (arcsin c − arcsin 0)
0 1 − x2 c%1 0 1 − x2 c%1 c%1

= arcsin 1 − arcsin 0 = π/2


Z 1
1
Deci integrala improprie √
dx este (C).
1 − x2
0
Z 0
1
16. Să se studieze natura integralei √ dx.
−1 1 − x2

Rezolvare:

Observăm că se integrează funcţia f : (−1, 0] → R, f (x) = √ 1 (deci punctul singular este −1).
1−x2
Z 1
1
17. Să se studieze natura integralei √ dx.
−1 1 − x2

Rezolvare:

Observăm că se integrează funcţia f : (−1, 1) → R, f (x) = √ 1 (deci punctele singulare sunt ±1).
1−x2
Z 1 Z 0 Z 1
1 1 1
√ dx = √ dx + √ dx.
−1 1 − x2 −1 1 − x2 0 1 − x2

18. Integrala
Z b
dx
λ
a (b − x)
este convergentă pentru λ < 1 şi divergentă pentru λ ≥ 1.

10
Rezolvare:
1
Observăm că se integrează funcţia f : [a, b) → R, f (x) = λ
care este continuă pe orice
(b − x)
interval de tipul [a, c], cu ∀ a < c < b deci este integrabilă pe orice interval compact [a, c] ⊂ [a, b).
Atunci, prin definiţie
Z b Z c Z c
dx dx −λ
λ
= lim λ
= lim (b − x) dx
a (b − x) c→b a (b − x) c→b a

Dar
−λ+1

Z  − (b − x)

, λ 6= 1
−λ
(b − x) dx = −λ + 1

− ln (b − x) , λ=1

deci, pentru λ = 1,
Z c
−1 c
lim (b − x) dx = − lim ln (b − x)|a = lim (− ln (b − c) + ln (b − a)) =
c→b a c→b c→b

= − ln 0+ + ln (b − a) = − (−∞) + ln (b − a) = ∞

şi pentru λ 6= 1,

−λ+1 c
!
c −λ+1 −λ+1
(b − x) (b − c) (b − a)
Z
−λ
lim (b − x) dx = − lim = lim −

c→b a c→b −λ + 1 c→b λ−1 λ−1
a
!
−λ+1
0−λ+1
+ (b − a)
= −
λ−1 λ−1

Dacă avem λ > 1 atunci λ − 1 > 0 deci 0−λ+1


+ = 1
λ−1
0+
= 1
0+ = +∞, iar dacă avem λ < 1 atunci
1 − λ > 0 deci 0−λ+1
+ = 0.
Integrala iniţială este atunci

−λ+1 c

(b − x)

Z b  lim −

 ,

λ 6= 1  +∞ ,
 λ ≥ 1,
dx c→b −λ + 1
λ
=
a = (b − a)
−λ+1
a (b − x) 
 lim − ln (b − x)|c ,
  −
 , λ < 1.
a λ=1 λ−1
c→b

b 1−λ
(b − a)
Z
dx
deci λ
= este (C) pentru λ < 1 şi (D) pentru λ ≥ 1.
a (b − x) 1−λ

19. Integrala
Z b
dx
λ
a (x − a)
este convergentă pentru λ < 1 şi divergentă pentru λ ≥ 1.
Rezolvare:
1
Observăm că se integrează funcţia f : (a, b] → R, f (x) = λ
care este continuă deci integrabilă
(x − a)
pe orice interval compact [a, c] ⊂ (a, b]. Se obţine că

Z b  +∞ ,
 λ ≥ 1,
dx
λ
= (b − a)
−λ+1
a (x − a) 
 , λ < 1.
1−λ

11
Z 1
dx
20. Să se studieze natura integralei √
4
.
0 1 − x4
Rezolvare:
1
Observăm că se integrează funcţia f : [0, 1) → R, f (x) = √
4
care este (continuă deci) inte-
1 − x4
grabilă pe orice interval compact [0, c] ⊂ [0, 1). Trebuie să determinăm λ astfel ı̂ncât să existe limita
λ λ λ
lim |x − 1| f (x). Deoarece x ∈ [0, 1) avem că x < 1 ⇒ |x − 1| = (1 − x) , deci
x%1

λ λ
λ (1 − x) (1 − x)
lim |x − 1| f (x) = lim p = lim
x%1 (1 − x)1/4 4 (1 + x) (1 + x2 )
p
x%1 4 x%1 (1 − x) (1 + x) (1 + x2 )

Aleg λ = 1/4 şi obţin

λ 1 1
lim |x − 1| f (x) = lim p = √
4
∈ (0, ∞)
x%1 x%1 4 2
(1 + x) (1 + x ) 4

λ 1
Deci pentru λ = 1/4 < 1, lim |x − 1| f (x) = √
4
∈ (0, ∞) adică integrala este (C).
x%1 4
21. Să se studieze convergenţa următoarelor integrale improprii:
1 3 3 3
x2 dx
Z Z Z Z
dx dx
(a) ln (1 − x) dx , (b) , (c) , (d) √ ,
4 − x2
q
0 1 2 5 2
(3 − x) (x2 + x − 6) 1 4x − x2 − 3

Z b Z ∞ Z 2
xdx dx dx
(e) p , (f ) q , (g) √ ,
a (x − a) (b − x) 2 3 4
(2 − x) (x4 + x2 + 1) 1 x x−1

Z ∞ Z 1 Z ∞ Z 1
dx dx 5x + 1 dx
(h) , (i) , (j) dx , (k) √ .
1 1 − x2 0 1 − x2 1 x3 − x − 6 0
3
1 − x3

Rezolvare:

(a) Calculăm mai ı̂ntâi primitiva


Z
ln (1 − x) dx = x ln (1 − x) − x − ln (1 − x) + C

= − (1 − x) ln (1 − x) − x + C, C ∈ R.

Deci
Z 1 x=1
ln (1 − x) dx = [− (1 − x) ln (1 − x) − x]

0 x=0
= − lim (1 − x) ln (1 − x) + 1 ln 1 − 1 + 0 = (notând 1 − x = y)
x→1
x<1
1 ln z
= − lim y ln y − 1 = (notând = z) = lim − 1 = 0 − 1.
y→0+ y z→∞ z

(b)
Z 3 Z 3 Z 2 Z 3
dx dx dx dx
= = + .
1 4 − x2 1 (2 − x) (2 + x) 1 4 − x2 2 4 − x2
(c)
3 5/2 3
x2 dx x2 dx x2 dx
Z Z Z
q = q + q .
2 5 2 2 5 2 5/2 5 2
(3 − x) (x2 + x − 6) (3 − x) (x2 + x − 6) (3 − x) (x2 + x − 6)

12
(d)
Z 3 Z 2 Z 3
dx dx dx
√ = √ + √ .
1 4x − x2 − 3 1 4x − x2 − 3 2 4x − x2 − 3
(e) Facem substituţia
a+b a+b
x− =y⇔x=y+ ⇒ dx = dy
2 2
şi obţinem
b−a b−a b−a
a+b a+b
y+
Z Z Z
2
2
2 y 2
2
I = q dy = q dy + q dy
a−b b−a 2 a−b b−a 2 a−b b−a 2
  
2
2 − y2 2
2 − y2 2
2 − y2
 0
b−a 2

Z b−a
2 − y2 a+b
2
Z b−a2 1
= (−) q dy + q dy
a−b b−a
 2 2 2 a−b b−a 2

2
2
22 −y 2
2 −y
b−a b−a
x= x=
s 2
2 2
b−a a+b y
= − − y2 + arcsin b−a .

2 a−b 2 2
a−b
x= 2 x= 2

(f ) vom lua λ = 4/3 şi α = 4/3.


(g) vom lua λ = 1/2.
(h)
1 x − 1
Z Z
dx dx
=− = − ln + C.
1 − x2 x2 − 1 2 x + 1
Acum
Z ∞ Z 2 Z ∞
dx dx dx
= +
1 1 − x2 1 1 − x 2
2 1 − x2

1 2 − 1 1 a − 1 1 c − 1 1 2 − 1
= (−) ln − lim (−) ln + lim (−) ln − (−) ln
2 2 + 1 a→1 2 a + 1 c→∞ 2 c + 1 2 2 + 1
a>1

1 a − 1
= lim ln
+ lim (−) 1 ln 1.
2 a→1 a + 1 c→∞ 2
a>1

(j) Calculăm mai ı̂ntâi primitiva


Z Z
5x + 1 5x + 1
3
dx = dx
x −x−6 (x − 2) (x2 + 2x + 3)

descompunând ı̂n fracţii simple.

22. Să se studieze convergenţa următoarelor integrale improprii:


Z ∞ Z ∞ Z ∞
x + cos x ln x
(a) (π − 2arctg (x)) dx , (b) 3
dx , (c) dx ,
1 1 x + sin x 1 x

Z ∞ Z 2 Z 2
1 1 ln x
(d) β
dx , β ∈ R , (e) β
dx , β ∈ R , (f ) dx ,
2 x (ln x) 1 x (ln x) 1 x

Z 1 Z 1 Z π/2
1 dx
(g) √
5
dx, (h) p , (i) ctgx dx ,
−1 x3 0
3
x (ex − e−x ) 0

s
Z π Z 1   Z ∞
dx 1 xdx
(j) √ , (k) ln dx , (l) √ ,
0 sin x 0 x 0 e2x − 1

13
Z ∞ Z ∞ Z 1
ln x x ln x
(m) √ dx , (n) 3 dx , (o) xa ln xdx, a ∈ R.
1 x x2 − 1 0 (1 + x2 ) 0

Rezolvare:

(a) Avem f (x) = π − 2arctg (x). Trebuie să determinăm α astfel ı̂ncât să existe limita lim xα f (x).
x→∞

π − 2arctg (x)
lim xα f (x) = lim (xα · (π − 2arctg (x))) = (nedeterm. ∞ · 0) = lim 1
x→∞ x→∞ x→∞

1
0 − 2 1+x 2
2
2x
= (aleg α = 1 şi aplic L’Hospital) = lim −1 = lim = 2.
x→∞
x2
x→∞ 1 + x2
R∞
Deci pentru α = 1 ≤ 1 obţin lim xα f (x) = 2 ∈ (0, ∞) deci 1
(π − 2arctg (x)) dx este (D).
x→∞

(b)

x3 1 + cosx x

α x + cos x α x3 + x2 cos x
lim x f (x) = lim x · 3 = (aleg α = 2) = lim = lim 3
1 + sin x

x→∞ x→∞ x + sin x x→∞ x3 + sin x x→∞ x
x3
1 + cosx x 1+0
= lim = = 1.
x→∞ 1 + sin3x 1+0
x

Am folosit că
cos x sin x
lim = 0 şi lim = 0,
x→∞ x x→∞ x3
(c)
ln x
lim xα f (x) = lim xα ·
= (aleg α = 1) = lim ln x = +∞ > 0
x→∞ x x→∞ x→∞

deci limita este > 0 iar α ≤ 1, prin urmare integrala este divergentă (Conform Criteriului ı̂n α).
(d) Calculăm primitiva
Z Z
1 −β 0
β
dx = (ln x) (ln x) dx = (pp. β 6= 1 şi fac subst. ln x = t)
x (ln x)
−β+1
t−β+1
Z
(ln x)
t−β dt = +C = + C.
−β + 1 −β + 1
Dacă β = 1, atunci
Z Z Z
1 1 0 1
dx = (ln x) dx = (subst. ln x = t) = dt = ln t + C = ln |ln x| + C.
x ln x ln x t
Prin urmare, pentru β 6= 1,
Z ∞ −β+1 x=∞
1 (ln x) 1  −β+1 −β+1

dx = = lim (ln x) − (ln 2)

β −β + 1 x=2

1 − β x→∞
2 x (ln x)
1  −β+1 −β+1

= (ln (+∞)) − (ln 2)
1−β
  
 1 +∞ − (ln 2)−β+1 = +∞, dacă β < 1,
 1−β
=   ,
 1
 1
− 1
= 1 1
, dacă β > 1.
1−β (+∞)β−1 (ln 2)β−1 β−1 (ln 2)β−1

Pentru β = 1,
Z ∞ x=∞
1
dx = ln |ln x| = lim ln |ln x|−ln |ln 1| = ln ln (+∞)−ln 0+ = ln (+∞)−(−∞) = +∞.

1 x ln x x=1 x→∞

14
2
(h) Vom lua λ = şi avem
3
1 xλ−1/3 xλ−1/3 · ex/3
limx→0+ xλ p = limx→0+ p = limx→0+ q
3
x (ex − e−x ) 3
e−x (e2x − 1) 3 e2x −1
· (2x)
1/3
2x
−1/3 x/3 λ−2/3 −1/3 0
2 e ·x 2 e
= limx→0+ q = √
3
.
3 e2x −1 1
2x

1 q q
şi, notând cu x1 = y, avem limx→0+ xλ ln x1 = limy→∞ lnyy = 0 .

(k) Vom lua λ =
2
(l) Se observă, mai ı̂ntâi, că limx→0+ f (x) = 0. Se obţine convergenţa cu α > 1, arbitrar ales.
(m) Se observă, mai ı̂ntâi, că limx→1+ f (x) = 0. Se obţine convergenţa cu α ∈ (1, 2) , arbitrar ales.
(n) Avem
!0
∞ ∞
−1
Z Z
x ln x 1
2 3 dx = 4 2 ln x dx.
0 (1 + x ) 0 (1 + x2 )
(o) Dacă a > 0, atunci, notând y = 1/x,
 a
1 1 ln y
limx→0 xa ln x = limy→∞ ln = − limy→∞ a = 0,
y y y

deci integrala nu este improprie.


Dacă a ∈ (−1, 0) , atunci se obţine convergenţa luând λ ∈ (−a, 1) , arbitrar ales, deoarece
1 1
limx→0 xλ · xa ln = limx→0 xλ+a ln = 0, cu λ + a > 0
x x
R1
(pentru a putea aplica criteriul trebuie să avem integrandul pozitiv, deci vom lua 0
xa (− ln x) dx =
R1 a
x ln x1 dx).

0
Dacă a ≤ −1, atunci se obţine divergenţa luând λ = −a ≥ 1, deoarece
1 1
limx→0 xλ · xa ln = limx→0 ln = ∞.
x x
R1
(pentru a putea aplica criteriul trebuie să avem integrandul pozitiv, deci vom lua 0
xa (− ln x) dx =
R1 a
x ln x1 dx).

0

23. Să se studieze, folosind criteriul de convergenţă ı̂n λ, convergenţa următoarelor integrale improprii:
Z 1 Z 3 Z 1
1 1 1
(a) √ dx , (b) √ √ dx , (c) dx ,
0
4
1 − x4 0
3
x+2 x+x
4 3
0 x3 − 5x2

2 1 1 e
ex
Z Z Z Z
1 dx 1
(d) √ dx , (e) x
dx , (f ) ,α>0, (g) dx.
1 x x−1 0 e −1 0 xα 1 x (ln x)
3/2

24. Să se studieze convergenţa următoarelor integrale:


Z π Z π/2 Z 1
sin x dx dx
(a) dx , (b) , (c) √ ,
7 + 6 cos x − 2 sin x a2 sin2 x + b2 cos2 x (3 − x) 1 − x2
0 0 −1

b Z br Z br
x2 dx b−x x−a
Z
(d) p , (e) dx , (f ) dx , a < b,
a (x − a) (b − x) a x−a a b−x

Z 2π
dx
(g)
0 2 sin x + 3 cos x + 4

15
Rezolvare:
x 2t 1 − t2
(a) Folosim substituţia tg = t şi formulele sin x = şi cos x = . Deci
2 1 + t2 1 + t2
Z ∞
4tdt
I= .
0 (t2 + 1) (t2 − 4t + 13)
t 1
(b) Folosim substituţia tg (x) = t şi formulele sin x = √ şi cos x = √ . Deci
1 + t2 1 + t2
!
Z ∞ Z ∞
dt 1 dt 1 1 t t=∞ π
I= = 2  = 2 · b · arctg b
b 2
= .
0 a2 t2 + b2 a 0 2
t + a a a
t=0 2ab
a

(c) x = sin t
(d) λ = 1/2 şi λ = 1/2
q
b−x
(e) Folosim substituţia x−a =t

2 (b − a) t2
Z
I= 2 dt .
0 (t2 + 1)
(f ) Folosim substituţia
x = a cos2 t + b sin2 t
cu
dx = (−2a cos t sin t + 2b sin t cos t) dt = (b − a) sin (2t) dt.
Obţinem

b Z π/2 s
(b − a) sin2 t
r
x−a
Z
dx = (b − a) sin (2t) dt
a b−x 0 (b − a) cos2 t
Z π/2 Z π/2 Z π/2
sin t 2
= 2 (b − a) sin t cos tdt = 2 (b − a) sin tdt = (b − a) (1 − cos (2t)) dt
0 cos t 0 0
 
t=π/2 sin (2t) t=π/2 π

= (b − a) t − = (b − a) .
2 2

t=0 t=0

(g) Folosim substituţia tg x2 = t, dar x ∈ [0, 2π] ⇔ x2 ∈ [0, π], iar tangenta nu este definită ı̂n π/2.
Deci
Z 2π Z π Z 2π
dx dx dx
= +
0 2 sin x + 3 cos x + 4 0 2 sin x + 3 cos x + 4 π 2 sin x + 3 cos x + 4
Z ∞ Z 0
2dt 2dt
= + .
0 t2 + 4t + 7 2
−∞ t + 4t + 7

25. Să se studieze convergenţa următoarelor integrale improprii:


Z ∞ Z ∞ Z 1   Z 1
xdx 1
(a) e−x sin (x + 1) dx , (b) 2 , (c) dx , (d) [ln x] dx .
0 0 1 + [x] 0 x 0

16
Facultatea de Matematică
Calcul integral şi aplicaţii, Semestrul I
Lector dr. Lucian MATICIUC

Seminariile 1 - 3
Capitolul I. Integrale improprii

ANEXĂ

1 Integrala improprie de primul tip


Definiţia 2 Fie f : [a, ∞) → R, o funcţie integrabilă Riemann pe intervale compacte de tipul [a, b], pentru orice
Z ∞ Z b
b > a. Integrala f (x) dx se numeşte integrala improprie de primul tip. Dacă limb→∞ f (x) dx există
a Z ∞ a

şi este finită vom spune că integrala improprie f (x) dx este convergentă, notat (C) , şi vom scrie
a
Z ∞ Z b
f (x) dx = limb→∞ f (x) dx
a a

O integrală care nu este convergentă se va numi divergentă, notat (D).

Teorema 3 Fie a > 0. Integrala Z ∞


1
dx
a xα
este convergentă pentru α > 1 şi divergentă pentru α ≤ 1.
Se obţine că
 b
x−α+1 1
 
Z ∞
limb→∞ , α 6= 1 a1−α , α > 1

1  

dx = −α + 1
a = α 1
a xα
∞, α ≤ 1.
 

 lim b
ln x| ,
b→∞ α=1a

Z a
Remarca 4 Similar putem defini f (x) dx .
−∞
Z ∞
Remarca 5 Integrala f (x) dx se poate defini prin egalitatea
−∞
Z ∞ Z a Z b Z a Z ∞
f (x) dx = lim f (x) dx + lim f (x) dx = f (x) dx + f (x) dx
−∞ b→∞ −b b→∞ a −∞ a
Z ∞
Definiţia 6 Fie o integrală improprie f (x) dx. Spunem că integrala este absolut convergentă, notat (AC) ,
Z ∞ a

dacă integrala |f (x)| dx este (C) .


1
Z ∞
Proposiţia 7 Dacă integrala f (x) dx este (AC) , atunci ea este (C) .
a

Remarca 8 Reciproca rezultatului anterior nu are loc.

17
Z ∞
Remarca 9 Se poate arăta (există exemple) că dacă integrala improprie f (x) dx este (C) , atunci nu este
a
necesar ca să existe limita limx→∞ f (x) şi să aibă valoarea

limx→∞ f (x) = 0.
Z ∞
Dar, se poate arăta, că, dacă integrala improprie f (x) dx este (C) şi există limita
a

limx→∞ f (x) ,

atunci aceasta are valoarea 0.


Astfel obţinem o condiţie suficientă de divergenţă a unei integrale improprii:Z

dacă există limita limx→∞ f (x) dar aceasta nu este 0, atunci integrala improprie f (x) dx este (D) .
a

2 Convergenţa integralei ı̂n cazul funcţiilor pozitive


Teorema 10 (Criteriul de comparaţie) Fie f, g : [a, ∞) → R două funcţii astfel ı̂ncât f ≤ g pe [a, ∞).
Z ∞ Z ∞
(i) Dacă integrala improprie g (x) dx este (C), atunci şi integrala improprie f (x) dx este (C) .
a a
Z ∞ Z ∞
(ii) Dacă integrala improprie f (x) dx este (D), atunci şi integrala improprie g (x) dx este (D) .
a a

Teorema 11 (Criteriul ı̂n α) Fie f : [a, ∞) → R astfel ı̂ncât f ≥ 0 pe [a, ∞).


Z ∞
α
(i) Dacă există α > 1 astfel ı̂ncât limx→∞ x f (x) < +∞ atunci f (x) dx este (C).
a
Z ∞
(ii) Dacă există α ≤ 1 astfel ı̂ncât limx→∞ xα f (x) > 0 atunci f (x) dx este (D).
a

3 Convergenţa integralei ı̂n cazul general


Teorema 12 (Criteriul lui Abel) Fie f, g : [a, ∞) → R două funcţii astfel ı̂ncât
Z ∞
(i) Integrala improprie f (x) dx este (C).
a

(ii) Funcţia g este monotonă şi mărginită.


Z ∞
În aceste condiţii integrala improprie f (x) g (x) dx este (C).
a

Teorema 13 (Criteriul lui Dirichlet) Fie f, g : [a, ∞) → R două funcţii astfel ı̂ncât
(i) Funcţia f este integrabilă pe [a, b], pentru orice b > a şi
Z b
f (x) dx ≤ K, pentru orice a < b < ∞.


a

(ii) Funcţia g este monotonă şi limx→∞ g (x) = 0.


Z ∞
În aceste condiţii integrala improprie f (x) g (x) dx este (C).
a

18
4 Integrala improprie de al doilea tip
Definiţia 14 Fie f : [a, b) → R, o funcţie integrabilă Riemann pe intervale compacte de tipul [a, c], pentru orice
Z b
a < c < b şi care satisface lim f (x) = ∞. Integrala f (x) dx se numeşte integrala improprie de al doilea
x%b a Z b
Rc
tip. Dacă limc%b a f (x) dx există şi este finită vom spune că integrala improprie f (x) dx este (C) şi vom
a
scrie Z b Z c
f (x) dx = limc%b f (x) dx
a a

Remarca 15 Punctul b de mai sus spunem că este punct singular.

Z b
Remarca 16 Similar putem defini f (x) dx ı̂n cazul ı̂n care funcţia f : (a, b] → R este o funcţie integrabilă
a
Riemann pe intervale compacte de tipul [c, b], pentru orice a < c < b şi care satisface limx&a f (x) = ∞.

Teorema 17 Integrala
Z b
dx
λ
a (b − x)
este convergentă pentru λ < 1 şi divergentă pentru α ≥ 1.

Proposiţia 18 Analog se poate studia natura integralei


Z b
dx
λ
, λ ∈ R.
a (x − a)

Se obţine că 
Z b  +∞ ,
 λ ≥ 1,
dx
λ
= (b − a)
−λ+1
a (x − a) 
 , λ < 1.
1−λ

Teorema 19 (Criteriul ı̂n λ) Fie f : [a, b) → R astfel ı̂ncât f (x) ≥ 0, pentru orice x ∈ [a, b). Atunci
Z b
λ
(i) Dacă există λ < 1 a.ı̂. limx%b |x − b| f (x) < ∞ atunci f (x) dx este (C).
a
Z b
λ
(ii) Dacă există λ ≥ 1 a.ı̂. limx%b |x − b| f (x) > 0 atunci f (x) dx este (D).
a

5 Criteriul lui Cauchy de caracterizare a convergenţei


Teorema 20 Fie −∞ < a < b ≤ +∞. Funcţia f : [a, b) → R este integrabilă pe [a, b) dacă şi numai dacă pentru
orice  > 0, există b ∈ [a, b) astfel ı̂ncât pentru orice b0 , b00 ∈ [b , b) are loc
Z b00
f (x) dx <  .


b0

19
Facultatea de Matematică
Calcul Integral şi Aplicaţii, Semestrul I
Lector dr. Lucian MATICIUC

Seminariile 4 – 6
Capitolul II. Integrale cu parametru

1. Să se studieze convergenţa integralelor Fresnel:


Z ∞ Z ∞
sin x2 dx şi cos x2 dx.
 
0 0

Rezolvare:
√ 1
Folosind substituţia x2 = t obţinem x = t şi dx = √ dt. Deci
2 t
Z ∞
1 ∞ sin t
Z
1
I= sin t √ dt = √ dt.
0 2 t 2 0 t
Această integrală este convergentă aplicând criteriul lui Dirichlet.
R
b 1
Într-adevăr, a sin t dt = |cos a − cos b| ≤ 2, pentru orice a < b iar funcţia t 7→ √ este monoton

t

Z ∞ şi mărginită pe intervalul [1, ∞). Deci, conform criteriului lui Dirichlet, integrala
descrescătoare
sin t
improprie √ dt este convergentă.
1 t
R∞
Să observăm că 0 sin t dt nu există (deci este divergentă) şi astfel nu putem aplica criteriul lui
Abel.
Pe de altă parte, integrala
Z 1
1 sin t
I2 = √ dt
2 0 t
este convergentă deoarece funcţia t 7→ sin
√ t este
t√
bine definită pe (0, 1] iar punctul 0 nu este punct
singular, deoarece limt→0 sin
√ t = limt→0 sin t t =
t t 0. Deci funcţia precedentă poate fi prelungită prin

 sin
 t
√ , dacă t ∈ (0, 1],
continuitate la o funcţia continuă, definită pe [0, 1], f˜ (t) = t iar integralele

 0, dacă t = 0
din cele două funcţii coincid deoarece sunt modificate doar ı̂ntr-un punct, i.e.
Z 1 Z 1
sin t
√ dt = f˜ (t) dt.
0 t 0

1
R∞ sin
Deci integrala I = √t dt este (C) .
2 0 t

2. Arătaţi că integrala Z ∞


sin x
dx
0 xα
este divergentă ı̂n cazul α < 0.
Rezolvare:

Avem Z ∞ Z 1 Z ∞
sin x sin x sin x
dx = dx + dx.
0 xα 0 xα 1 xα

1
Z 1 Z 1
sin x
Evident, integrala dx = x−α sin xdx nu este improprie, dacă α < 0, deci este o integrală
0 xα 0
Riemann, deci (C) .
Z ∞
sin x
În ceea ce priveşte integrala dx vom arăta că este (D) folosind criteriul de integrabilitate
1 xα
al lui Cauchy.
π π
Astfel avem că pentru orice  > 0 arbitrar fixat, există b0 = 2nπ + >  , b00 = 2nπ + >  astfel
6 2
ı̂ncât are loc
Z b00 sin x Z b00 sin x Z 00
1 b 1 1 π  π −α
dx = dx ≥ dx ≥ 2nπ + → +∞, când n → +∞ .

xα xα 2 b0 xα 2 3 6

b0 b0

3. Să se studieze absoluta convergenţă a integralei:


Z ∞
sin x
dx
1 xα

ı̂n cazul α > 0.


Rezolvare:
R∞ sin x
Să remarcăm mai ı̂ntâi că integrala 1 xα dx este (C) conform criteriului lui Dirichlet, pentru orice
α > 0.
R∞
A studia absoluta convergenţă a integralei 1
f (x) dx ı̂nseamnă a studia convergenţa integralei
R∞
1
|f (x)| dx.
Dacă α > 1, atunci avem
sin x 1
xα ≤ xα ,

iar Z ∞
1
α
dx este (C) ,
1 x
R ∞ x
deci, conform unui criteriu de comparaţie, 1 sinxα dx este (C) .

R ∞ sin x
În consecinţă, 1 xα dx este (AC) ı̂n cazul α > 1.
În cazul celălalt să observăm mai ı̂ntâi că
sin x sin2 x

1 − cos (2x) 1 1 1 cos (2x)
xα ≥ xα = = − .

2xα 2 xα 2 xα

Dar ı̂n cazul α ∈ (0, 1], Z ∞


1
dx este (D) .
1 xα
Pe de altă parte, Z ∞
cos (2x)
dx este (C)
1 xα
conform criteriului lui Dirichlet, pentru orice α > 0.
Deci Z ∞ Z ∞
1 1 1
cos (2x)
dx − dx = (D) − (C) = (D) ,
1 2 xα
1 xα
2
R ∞ x
iar conform unui criteriu de comparaţie vom obţine că 1 sin
xα dx este (D) .

R ∞ sin x
În consecinţă, 1 xα dx nu este (AC) ı̂n cazul α ∈ (0, 1].

2
4. Să se studieze convergenţa integralei:
Z ∞
sin (αx)
e−kx dx , unde α ≥ 0, k > 0.
0 x

Rezolvare:
Z ∞
sin (αx)
Această integrală este convergentă aplicând criteriul lui Abel, deoarece dx este con-
0 x
vergentă conform criteriului lui Dirichlet iar funcţia x 7→ e−kx este monotonă şi mărginită pe [0, ∞)
cu limx→∞ e−kx = 0 pentru orice k > 0.
5. Să se calculeze următoarea integrală cu parametru
Z 1
arctg (xy)
I (y) = √ dx
0 x 1 − x2
şi apoi să se obţină valoarea integralei
Z 1
arctg (x)
√ dx .
0 x 1 − x2
Rezolvare:

Derivăm integrala cu parametru şi operatorul de derivare comută cu integrala:


Z 1  0 Z 1 Z 1
arctg (xy) 1 0 1 1
I 0 (y) = √ dx = √ (arctg (xy))y dx = √ x dx
0 x 1 − x2 y 0 x 1 − x2 0 x 1 − x2 1 + x2 y 2
Z 1
1 1
= √ dx .
0 1−x 2 1 + x2 y 2

Deoarece apare cantitatea 1 − x2 este utilă subsituţia x = sin t, deci

t = arcsin x şi dx = cos t dt.

Obţinem
Z π/2 Z π/2
0 1 1 1
I (y) = 2 cos t dt = dt.
sin2 t
p
0 1 − sin2 t 1 + y 2 sin t 0 1+ y2
Acum se face substituţia tg (t) = r, deci
1
t = arctg (r) şi dt = dr .
1 + r2
Folosim formula sin t = √ r şi obţinem
1+r 2
Z +∞ Z +∞ Z +∞
1 1 1 1 1
I 0 (y) = r2 1 + r2
dr = dr = 2 2 dr
1 + y 2 1+r 1 + (y 2 + 1) r2 y +1

0 2 0 0
√ 12 + r2
y +1
r=∞
1 1 r 1 π 1
= 2 arctg =p (π/2 − 0) = p .

y + 1 √ 12 √ 1 2

2
y +1 2
y +1
y +1 y 2 +1 r=0

Deci
π 1 π  p 
I 0 (y) = p ⇒ I (y) = ln y + y 2 + 1 + C
2 y2 + 1 2
Dar, conform definiţiei,
I (0) = 0, deci C = 0.

3
6. Să se calculeze următoarea integrală cu parametru
Z ∞
1 − e−αt
I (α) = cos t dt , α>0
0 t

şi apoi să se obţină valoarea integralei



1 − e−t
Z
cos t dt .
0 t

Rezolvare:

Derivăm integrala cu parametru şi operatorul de derivare comută cu integrala:


∞ 0 ∞ ∞
1 − e−αt e−αt · t
Z  Z Z
0 α
I (α) = cos t dt = cos t dt = e−αt cos t dt = .
0 t α 0 t 0 α2 +1

Deci
1
ln α2 + 1 + C

I (α) =
2
Dar, conform definiţiei,
I (0) = 0, deci C = 0.

7. Să se calculeze următoarea integrală cu parametru


Z ∞
1 − cos (αt) −kt
I (α) = e dt , α, k > 0.
0 t

Rezolvare:

Derivăm integrala cu parametru şi operatorul de derivare comută cu integrala:


∞  0 ∞ ∞
1 − cos (αt) −kt
Z Z Z
0 sin (αt) −kt α
I (α) = e dt = te dt = e−kt sin (αt) dt = .
0 t α 0 t 0 k2 + α2

Deci
1
ln α2 + k 2 + C

I (α) =
2
Dar, conform definiţiei,
1
deci C = − ln k 2 ,

I (0) = 0,
2
deci
α2 + k 2 α2
   
1 1
I (α) = ln = ln 1 + 2 .
2 k2 2 k

8. Să se calculeze următoarea integrală cu parametru




ln 1 + a2 x2
Z
I (a) = dx , a, b > 0.
0 b2 + x 2

Rezolvare:

Derivăm integrala cu parametru şi operatorul de derivare comută cu integrala:


Z ∞
1 1
I 0 (a) = 2 + x2 1 + a 2 x2
2ax2 dx .
0 b

Dar, pentru orice a2 b2 6= 1,

x2 b2
 
1 1
= 2 2 − .
(b2 + x2 ) (1 + a2 x2 ) a b −1 b2 + x2 1 + a2 x2

4
Deci
Z ∞
b2
  2 
0 2a 1 2a b x x=∞ 1 1 x x=∞
I (a) = 2 2 − dx = 2 2 arctg − 2 1 arctg 1
a b −1 0 b2 + x2 1 + a2 x2 a b −1 b b x=0 a a a
x=0
 
2a 1 π π
= 2 2 b− = .
a b −1 a 2 ab + 1

şi prin urmare


π
I (a) = ln (ab + 1) + C
b
(cazul a2 b2 = 1 trebuie studiat separat).
Dar, conform definiţiei,
I (0) = 0, deci C = 0 .

9. Să se calculeze următoarele integrale cu parametru:


Z ∞
arctg (ax)
(a) I (a) = dx , a ≥ 0.
0 x (1 + x2 )
Z ∞
arctg (ax) · arctg (bx)
(b) I (a) = dx , a, b > 0.
0 x2

Rezolvare:

(a) Derivăm integrala cu parametru şi operatorul de derivare comută cu integrala:


Z ∞ Z ∞
0 1 1 1 π 1
I (a) = 2 ) 1 + a2 x2
x dx = 2 ) (1 + a2 x2 )
dx = , a 6= 1.
0 x (1 + x 0 (1 + x 2 1+a

(folosim descompunerea ı̂n fracţii simple).


În cazul a = 1 obţinem (eventual prin trecerea la limită ı̂n integrala cu parametru I 0 (a)) aceeaşi
π 1
valoare, i.e. I 0 (a) = .
2 1+a
Deci
π
I (a) = ln (1 + a) .
2
(b) Derivăm integrala cu parametru şi operatorul de derivare comută cu integrala:
Z ∞ Z ∞
arctg (bx) 1 arctg (bx)
I 0 (a) = 2 2 2
x dx = dx .
0 x 1+a x 0 x (1 + a2 x2 )

Facem schimbarea de variabilă ax = t, deci, folosind (a) , obţinem


Z ∞ Z ∞
arctg ab t
  
arctg (bx) π b
I 0 (a) = dx = dt = ln 1 + .
0 x (1 + a2 x2 ) 0 t (1 + t2 ) 2 a

Deci
π  a+b

I (a) = ln (a + b) − ln aa − b .
2
10. Să se calculeze următoarea integrală cu parametru, derivând-o ı̂n prealabil:
Z ∞ −at
e cos (bt) − e−a0 t cos (b0 t)
I (a, b) = dt , a, a0 > 0.
0 t

Rezolvare:

Se obţine
1 a2 + b20
I (a, b) = ln 20 .
2 a + b2

5
11. Să se calculeze următoarele integrale cu parametru (a > 0)
Z ∞ Z ∞ Z 1
−ax2 dx
(a) e dx, (b) , (c) xa−1 dx
0 0 a + x2 0

şi apoi, prin derivarea lor succesivă ı̂n raport cu parametrul, să se obţină valoarea altor integrale
noi.

Rezolvare:

Avem, pentru n ∈ N∗ ,
(a)
∞ ∞
r r
(2n − 1)!!
Z Z
−ax2 1 π −ax2 2n π
e dx = şi apoi e x dx = n+1 n
0 2 a 0 2 a a
(b)
∞ ∞
r
1 (2n − 1)!! 1
Z Z
dx 1 π dx π
= √ şi apoi n+1 =
0 a + x2 2 a 0 (a + x2 ) 2 (2n)!! an a
(c)
Z 1 Z 1
1 n n!
x a−1
dx = şi apoi xa−1 lnn x dx = (−1) .
0 a 0 an+1

12. Să se calculeze următoarea integrală:


Z 1
arctg (x)
√ dx .
0 x 1 − x2
Rezolvare:

Observăm că Z 1
arctg (x) 1
= dy
x 0 1 + x2 y 2
şi, deoarece putem schimba ordinea de integrare, obţinem:
Z 1 Z 1   Z 1 Z 1 
1 1 1 1
I= dy √ dx = √ dx dy .
0 0 1 + x2 y 2 1 − x2 0
2 2
0 1+x y 1 − x2
Deci, conform exerciţiului anterior,

π
Z 1
1 π  p  y=1 π  √ 
I= dy = ln y + y 2 + 1 = ln 1 + 2 .

p
2 0 y2 + 1 2 y=0 2

13. Să se calculeze următoarea integrală:


Z ∞
cos (ax) − cos (bx)
dx , 0 < a < b.
0 x

Rezolvare:

Metoda I.
Observăm că Z ∞
1
= e−tx dt
x 0

şi, deoarece putem schimba ordinea de integrare, obţinem:


Z ∞ Z ∞  Z ∞ Z ∞ 
I= (cos (ax) − cos (bx)) e−tx dt dx = (cos (ax) − cos (bx)) e−tx dx dt
0 0 0 0

6
Integrând de două ori prin părţi obţinem
Z ∞
t
cos (ax) e−tx dx = , pentru orice a > 0, t > 0,
0 t2 + a2

deci Z ∞  
t t 1  t=∞
ln t2 + a2 − ln t2 + b2

I= − dt =
0 t + a2
2 t + b2
2 2 t=0
 2
t + a2
 t=∞  2  
1 1 1 a b
= ln 2 = ln 1 − ln = ln .

2 t + b2 2 2 b2 a

t=0

Metoda II.
Observăm că integrala dată este de tip Froullani.
Verificăm mai ı̂ntâi condiţiile. Funcţia f : [0, ∞) → R dată
R ∞de f (x) = − cos x este o funcţie continuă
şi mărginită. Nu există limx→∞ f (x) dar, ı̂n schimb, a cosx x dx este (C) (conform criteriului lui
Dirichlet).
Deci are loc formula

− cos (bx) + cos (ax)
Z
a b
dx = − cos (0) ln = ln .
0 x b a

14. Să se calculeze următoarele integrale:


∞ ∞
e−ax − e−bx arctg (ax) − arctg (bx)
Z Z
(a) dx , (b) dx , 0 < a < b.
0 x 0 x

Rezolvare:

Fiecărui exemplu i se pot aplica ambele metode de la Exerciţiul 13.


(a) Metoda I. Observăm că Z ∞
1
= e−tx dt
x 0

şi apoi schimbăm ordinea de integrare.


Metoda II. Observăm că integrala dată este de tip Froullani cu f (x) = −e−x .

Pentru acest exemplu avem şi


Metoda III.
e−ax
Z
Observăm că e−ax da = + C, adică
−x
b
−e−αx α=b e−ax − e−bx
Z
e−αx dα = = .
x x

a α=a

Deoarece putem schimba ordinea de integrare, obţinem:


!
∞ b b ∞ b
e−αx x=∞
Z Z Z Z  Z  
I = e−αx dα dx = e−αx dx dα = dα
−α x=0

0 a a 0 a
Z b Z b
1 1 b
e−∞ − e 0

= dα = dα = ln .
a −α a α a
π
(b) Observăm că integrala dată este de tip Froullani cu f (x) = −arctg (x) + .
2

7
15. Reduceţi următoarele integrale la integrale de tip Froullani şi stabiliţi rezultatele:
Z ∞
sin (ax) sin (bx) 1 a+b
(a) dx = ln , a, b > 0, a 6= b;
0 x 2 |a − b|
Z 1 a−1
x − xb−1 a
(b) dx = ln , a, b > 0.
0 ln x b
Rezolvare:
(a) Se foloseşte formula trigonometrică
1
sin a sin b = (cos (a − b) − cos (a + b)) .
2
(b) Se foloseşte substituţia
ln 1/x = y ⇔ x = e−y ⇒ dx = −e−y dy,
iar
0 ∞
e−(a−1)y − e−(b−1)y e−by − e−ay
Z Z
−e−y dy =

I= dy.
+∞ −y 0 y
16. Să se calculeze următoarea integrală:
Z ∞
cos (ax) − cos (bx)
dx , 0 < a < b.
0 x2
Rezolvare:
Metoda I.
Z
cos (ax)
Observăm că sin (ax) da = + C, adică
x
b
− cos (αx) α=b − cos (bx) + cos (ax)
Z
sin (αx) dα = = .
x x

a α=a

Deoarece putem schimba ordinea de integrare, obţinem:


Z ∞ Z b ! Z b Z ∞ 
1 sin (αx)
I= sin (αx) dα dx = dx dα
0 x a a 0 x

Dar, schimbând variabila y = αx şi folosind Exerciţiul 26, obţinem


Z ∞ Z ∞
sin (αx) sin y π
dx = dy = ,
0 x 0 y 2
deci
b ∞
π b
Z Z  Z
sin (αx) π
I= dx dα = dα = (b − a) .
a 0 x 2 a 2
Metoda II.
Se foloseşte metoda de integrare prin părţi şi apoi Exerciţiul 26.
Avem
∞ 0  Z ∞
−1 cos (bx) − cos (ax) x=∞ −a sin (ax) + b sin (bx)
Z
I= (cos (ax) − cos (bx)) dx = + dx
x x x

0 x=0 0
Z ∞ Z ∞
sin (ax) sin (bx) π
= (0 − 0) − a dx + b dx = (b − a) ,
0 x 0 x 2
deoarece
cos (bx) − cos (ax) 1
limx→∞ = limx→∞ (cos (bx) − cos (ax)) · = 0
x x
iar, folosind L’Hospital,
cos (bx) − cos (ax) −b sin (bx) + a sin (ax)
limx→0 = limx→0 = 0.
x 1

8
17. Reduceţi următoarele integrale la integrale de tip Froullani folosind metoda de integrare prin părţi:
Z ∞ Z ∞ −ax
b sin (ax) − a sin (bx) be − ae−bx
(a) dx ; (b) dx ;
0 x2 0 x2
Z ∞ Z ∞ −ax 2
b ln (1 + ax) − a ln (1 + bx) e − e−bx
(c) dx ; (d) dx .
0 x2 0 x2

18. Folosind metoda de integrare prin părţi, să se stabilească următoarele rezultate:
Z ∞ −a2 x2 2 2
e − e−b x √
(a) 2
dx = π (b − a) ;
0 x
Z ∞  
ln 1 + a2 x2 − ln 1 + b2 x2
(b) dx = π (b − a) .
0 x2
Rezolvare:
(a) Se foloseşte rezultatul de la Exerciţiul 28.
19. Să se calculeze următoarea integrală:
1
xb − xa
Z
dx , 0 < a < b.
0 ln x
Rezolvare:
Observăm că Z b
xb − xa
= xy dy
ln x a
şi, deoarece putem schimba ordinea de integrare, obţinem:
Z 1 Z b ! Z b Z 1  Z b y+1 x=1 Z b
y y x 1
I= x dy dx = x dx dy = dy = dy

a y + 1 x=0 a y+1

0 a a 0
y=b 1+b
= ln (1 + y) = ln .

y=a 1+a
20. Să se calculeze următoarea integrală cu parametru:
Z π/2
ln a2 − sin2 x dx , a > 1 .

I (a) =
0

Rezolvare:
Derivăm integrala cu parametru şi operatorul de derivare comută cu integrala:
Z π/2 Z π/2 Z π/2
0 2 2
0 1 dx
I (a) = ln a − sin x a dx = 2 − sin2 x
2a dx = 2a 2 − sin2 x
.
0 0 a 0 a
Se face substituţia tg (x) = t, deci
1
x = arctg (t) şi dx = dt .
1 + t2
Folosind formula sin x = √ t obţinem
1+t2
Z π/2 Z +∞
dx 1 1
I 0 (a) = 2a = 2a dt
0 a2 − sin2 x0 a2 − t2 1 + t2
1+t2
Z +∞ Z +∞
1 1 1
= 2a 2 − 1) t2 + a2
dt = 2a 2 2 dt
(a a − 1

0 0 t2 + √aa2 −1
t=∞
2a 1 t 2 π
= 2 arctg a =√ (π/2 − 0) = √ .
a − 1 √aa2 −1 √
a2 −1 t=0
2
a −1 2
a −1

9
Deci
π  p 
I 0 (a) = √ ⇒ I (a) = π ln a + a2 − 1 + C.
a2 − 1
21. Folosind valoarea integralei
Z π/2
dx π
= ,
0 a2 sin2 x + b2 cos2 x 2ab
să se calculeze integrala
π/2
sin2 x
Z
2 dx .
0 a2 sin2 x + b2 cos2 x

22. Să se calculeze integrala cu parametru:


Z π
1
I (a) = dx , unde a > b > 0.
0 a + b cos x

Apoi derivând ı̂n raport cu a integrala cu parametru I (a) să se calculeze


Z π Z π
1 1
2 dx şi 3 dx.
0 (a + b cos x) 0 (a + b cos x)

Rezolvare:
Z π Z ∞ Z ∞
1 1 2 2 1
I (a) = dx = 1−t2 1 + t2
dt = 2 dt
0 a + b cos x 0 a + b 1+t2 a−b 0
q
a+b
a−b + t2
t=∞
2 1 t 2 π
= arctg q =√ .

q
a−b a −b 2
2 2
a+b a+b
a−b a−b t=0

Derivăm şi obţinem


π  0 π π
−1 −1
Z Z Z
1
I 0 (a) = dx = 2 (1 + 0) dx = 2 dx .
0 a + b cos x a 0 (a + b cos x) 0 (a + b cos x)

Pe de altă parte
 0
0 π h −1/2 i0 −1 2 −3/2
I (a) = √ =π a2 − b2 =π a − b2 2a.
a − b2
2
a a 2

23. Să se calculeze integrala cu parametru:


Z π
1
I (a) = dx , unde 0 < a < b.
0 a + b cos x

24. Să se calculeze integrala cu parametru:


Z 2π
1
I (a) = dx , unde a > b > 0.
0 a + b cos x

Apoi derivând ı̂n raport cu a integrala cu parametru I (a) să se calculeze


Z 2π Z 2π
1 1
2 dx şi 3 dx.
0 (a + b cos x) 0 (a + b cos x)

Rezolvare:
R 2π 1
Rπ 1
R 2π 1
Observăm că =2· √ π
0 a+b cos x dx = 0 a+b cos x
dx + π a+b cos x dx a2 −b2
.

10
25. Să se calculeze integrala cu parametru
Z ∞
1
I (a) = dx, a ∈ R∗ .
0 a + x2

Apoi derivând ı̂n raport cu a integrala cu parametru I (a) să se calculeze


Z ∞ Z ∞
1 1
2 dx şi 3 dx
0 (a + x2 ) 0 (a + x2 )

26. Să se arate că Z ∞


sin x π
dx = .
0 x 2
Rezolvare:

Se arată mai ı̂ntâi, folosind un criteriu de convergenţă, că integrala dată este convergentă.
Ştim că următoarea integrală este convergentă (se aplică criteriul lui Abel, vezi Exerciţiul 4):
Z ∞
sin (αx)
I (α) := e−kx dx , unde α ≥ 0, k > 0.
0 x

Calculăm derivata şi apoi aplicăm metoda de integrare prin părţi:


Z ∞  0 Z ∞ Z ∞
0 sin (αx)
−kx −kx cos (αx)
I (α) = e dx = e x dx = e−kx cos (αx) dx
0 x α 0 x 0

e−kx x=∞ Z ∞ e−kx


= cos (αx) − (−α) sin (αx) dx

−k x=0 0 −k
e−kx x=∞ α Z ∞
= cos (αx) − e−kx sin (αx) dx (1)

−k x=0 k 0
e−kx x=∞ α  e−kx x=∞ Z ∞ e−kx 
= cos (αx) − sin (αx) − α cos (αx) dx

−k x=0 k −k x=0 0 −k
e−kx x=∞ α x=∞ α2 Z ∞
= cos (αx) + 2 e−kx sin (αx) − 2 e−kx cos (αx) dx.

−k x=0 k x=0 k 0
−kx
x=∞ x=∞
Dar e−k cos (αx) = −1 (0 − 1) = 1
iar α −kx
e sin (αx) = kα2 (0 − 0) = 0,deci

k k k2
x=0 x=0

1 α2 0 k
I 0 (α) = − 2 I (α) ⇒ I 0 (α) = .
k k α2 + k 2
Prin integrare obţinem Z
k α
I (α) = dα = arctg + C.
α2 +k 2 k
Dar, conform definiţiei,
I (0) = 0, deci C = 0.
Deci   Z ∞
1 sin (x)
I (1) = arctg = e−kx dx
k 0 x
şi, trecând la limită,
  Z ∞
π 1 sin (x)
= limk→0+ arctg = limk→0+ e−kx dx
2 k 0 x
Z ∞ Z ∞
−kx sin (x) sin (x)
= limk→0 e dx = dx.
0 x 0 x

11
27. Să se calculeze integrala Z ∞
sin x
dx
0 x
derivând integrala cu parametru
Z ∞
sin x
e−kx dx , unde k > 0.
0 x

28. Să se arate că1 : Z ∞ √


2 π
e−x dx = .
0 2
Rezolvare:

Se arată mai ı̂ntâi, folosind un criteriu de convergenţă, că integrala dată este convergentă.
Folosind substituţia x = ut, cu u > 0 fixat, obţinem dx = udt şi
Z ∞
2 2
I= e−u t udt.
0

Deci, schimbând ordinea de integrare, calculăm pătratul intergalei I :


Z ∞ Z ∞ Z ∞ Z ∞  Z ∞ Z ∞ 
2 −x2 −u2 −x2 −u2 −u2 t2 −u2
I = e dx · e du = e dx e du = e ue dt du
0 0 0 0 0 0
2 2
!
∞ ∞ ∞
e−(1+t )u u=∞
Z Z  Z
−(1+t2 )u2
= ue du dt = dt
−2 (1 + t2 ) u=0

0 0 0

−1
Z t=∞
1 π
= (0 − 1) dt = arctgt = .

2
2 (1 + t ) 2

t=0 4
0

Obţinem √
Z ∞
2 π
I= e−x dx = .
0 2

29. Să se arate că integrala lui Gauss (sau integrala Euler–Poisson) are valoarea:
Z ∞
2 √
e−x dx = π .
−∞

Rezolvare:

Evident, Z ∞ Z 0 Z ∞ Z ∞ √
−x2 −x2 −x2 2
e dx = e dx + e dx = 2 e−x dx = π.
−∞ −∞ 0 0

30. Să se calculeze următoarele integrale:


Z ∞ Z ∞
sin x2 dx cos x2 dx .
 
şi
0 0

Rezolvare:

Se arată mai ı̂ntâi, folosind un criteriu de convergenţă, că integrala dată este convergentă.
Folosind substituţia x2 = t obţinem Z ∞
1 sin t
I= √ dt
2 0 t
care este convergentă conform criteriului lui Dirichlet.
1 Există mai multe metode de a calcula această integrală. Unde dintre ele este cu ajutorul integralei duble (şi a schimbării de

variabile ı̂n cadrul integralei duble).

12
Pe de altă parte, folosind valoarea integralei Gaussiane, avem
Z ∞ Z ∞ √ 2
Z ∞ √ 2  √ 

2 1 1 π
e−tu du = e−(u t) du = √ e−(u t) d u t = √ ,
0 0 t 0 t 2

deci Z ∞
1 2 2
√ =√ e−tu du.
t π 0

Deci, schimbând ordinea de integrare, calculăm

1 ∞
Z  Z ∞  Z ∞ Z ∞ 
2 2 1 2
I= sin t √ e−tu du dt = √ e−tu sin t du dt
2 0 π 0 π 0 0
Z ∞ Z ∞ 
1 2
=√ e−tu sin t dt du.
π 0 0

Folosind metoda de integrare prin părţi avem, vezi calculul (1),


Z ∞
2 −1 2
t=∞ 1 2
x=∞ 1
J := e−tu sin t dt = 2 e−u t sin t − 4 e−u t cos t − 4J.

0 u t=0 u x=0 u

Deci
1
J= .
1 + u4
Calculând integrala2 obţinem
Z ∞ √
1 1 1 π π
I=√ du = √ · √ = √ .
π 0 1 + u4 π 2 2 2 2

31. Să se arate că



Z r
sin t π
√ dt = .
0 t 2

√ √ √
2 Avem dx 1 x2 +x√2+1 1 1
R
1+x4
= √ ln + √ arctg(x 2 + 1) + √ arctg(x 2 − 1) , C ∈ R.
4 2 x2 −x 2+1 2 2 2 2

13
Facultatea de Matematică
Calcul integral şi Aplicaţii, Semestrul I
Lector dr. Lucian MATICIUC

Seminariile 4 – 6
Capitolul II. Integrale cu parametru

ANEXĂ

1 Integrale de tip Froullani


Teorema 1 Fie 0 < a < b şi f : [0, ∞) → R o funcţie Z ∞ continuă şi mărginită. Dacă limx→∞ f (x) = 0 sau dacă

f (bx) − f (ax)
Z
f (x)
dx este (C) , atunci este (C) şi integrala dx şi are loc formula
a x 0 x
Z ∞
f (bx) − f (ax) a
dx = f (0) ln .
0 x b

Remarca 2 Un rezultat similar are loc şi dacă limx→∞ f (x) = `, deoarece considerăm g (x) = f (x) − ` ı̂n
teorema precedentă.

2 Integrale uniform convergente


Fie f : [a, +∞) × I → R. Presupunem că pentru orice y ∈ I există integrala cu parametru
Z +∞
I (y) = f (x, y) dx. (2)
a

Prin definiţie aceasta este dată de


Z b
I (y) = limb→+∞ I (y, b) = limb→+∞ f (x, y) dx.
a

Definiţia 3 Dacă limita precedentă are loc uniform ı̂n raport cu y ∈ I, atunci spunem că integrala parametrică
I (y) este uniform convergentă, notat (U C) , ı̂n raport cu y.

Teorema 4 (Condiţia suficientă 1 de convergenţă uniformă) Fie f : [a, +∞)×I → R. Presupunem că există
Z b
integrala f (x, y) dx, pentru orice b > a. Dacă există o funcţie ϕ astfel ı̂ncât:
a Z ∞
(ii) integrala improprie ϕ (x) dx este (C);
a
(ii) are loc inegalitatea
f (x, y) ≤ ϕ (x) , pentru orice x ≥ a, y ∈ I,
Z ∞
atunci integrala parametrică f (x, y) dx este (U C) .
a

Teorema 5 (CondiţiaZsuficientă 2 de convergenţă uniformă) Fie f, g : [a, +∞) × I → R. Presupunem că



integrala parametrică f (x, y) dx este (U C) ı̂n raport cu y şi, ı̂n plus, că funcţia x 7−→ g (x, y) este monotonă
a
iar
|g (x, y)| ≤ M, pentru orice x ≥ a, y ∈ I.
Z ∞
În aceste condiţii integrala parametrică f (x, y) · g (x, y) dx este (U C) . ı̂n raport cu y.
a

14
TeoremaZ6 (Condiţia suficientă 3 de convergenţă uniformă) Fie f, g : [a, +∞) × I → R. Presupunem că

integrala f (x) dx este (C) şi că funcţia x 7−→ g (x, y) este monotonă iar
a

|g (x, y)| ≤ M, pentru orice x ≥ a, y ∈ I.


Z ∞
În aceste condiţii integrala parametrică f (x) · g (x, y) dx este (U C) . ı̂n raport cu y.
a

Exemplul 7 Din acest ultim criteriu obţinem, de exemplu, convergenţa uniformă ı̂n raport cu y a integralei de
tipul
Z +∞
e−xy · f (x) dx, a ≥ 0,
a
Z +∞
ı̂n ipoteza că f (x) dx este (C) (deoarece (x, y) 7−→ e−xy este monotonă ı̂n raport cu x şi uniform mărginită
a
de 1).

3 Utilizarea convergenţei uniforme a integralelor


Teorema 8 (Continuitatea integralei parametrice) Fie f : [a, +∞) × [c, d] → R continuă. Presupunem că
Z ∞ Z +∞
integrala parametrică f (x, y) dx este (U C) ı̂n raport cu y ∈ [c, d]. Atunci y 7−→ I (y) = f (x, y) dx
a a
este o funcţie continuă.

Teorema 9 (Condiţia suficientă 4 de convergenţă uniformă) În cazul unei funcţii pozitive f are loc următoarea
Z +∞ Z ∞
“inversă” a teoremei precedente: dacă y 7−→ I (y) = f (x, y) dx este o funcţie continuă, atunci f (x, y) dx
a a
este (U C) .
Z +∞
Exemplul 10 Ştim că dacă integrala improprie f (x) dx este (C), atunci
a
Z +∞
I (k) = e−kx · f (x) dx, a ≥ 0,
a

este (U C) ı̂n raport cu k > 0. Dacă f este continuă, atunci are loc
Z +∞ Z +∞ Z +∞
limk→0+ e−kx · f (x) dx = limk→0+ e−kx · f (x) dx = f (x) dx.
a a a

Teorema 11 (Derivabilitatea integralei parametrice) Fie f : [a, +∞) × [c, d] → R continuă. Z ∞ Presupunem
0 0
că derivata fy (x, y) este continuă ı̂n raport cu ambele variabile şi că integrala parametrică fy (x, y) dx este
a
(U C) ı̂n raport cu y ∈ [c, d].
Z +∞ 0 Z +∞
0
I (y) = f (x, y) dx = fy0 (x, y) dx, pentru orice y ∈ [c, d] .
a y a

Teorema 12 (Integrabilitatea integralei parametrice) Fie f : [a, +∞) × [c, d] → R continuă. Presupunem că
Z ∞
integrala parametrică f (x, y) dx este (U C) ı̂n raport cu y ∈ [c, d]. Atunci
a
Z d Z d Z +∞  Z +∞ Z d 
I (y) dy = f (x, y) dx dy = f (x, y) dy dx.
c c a a c

15
Corolarul 13 (Integrabilitatea integralei parametrice) Fie f : [a, +∞)×[c, d] → R continuă şi pozitivă astfel
Z +∞
ı̂ncât y 7−→ I (y) = f (x, y) dx este o funcţie continuă. Atunci
a
Z d Z d Z +∞  Z +∞ Z d 
I (y) dy = f (x, y) dx dy = f (x, y) dy dx.
c c a a c

În multe cazuri este necesar să se permute integrale luate ambele pe intervale nemărginite pentru a se
obţine relaţia
Z +∞  Z +∞  Z +∞  Z +∞ 
f (x, y) dx dy = f (x, y) dy dx. (3)
c a a c

Teorema 14 (Integrabilitatea integralei parametrice) Fie f : [a, +∞)×[c, +∞) → R continuă. Presupunem
că integralele parametrice Z Z ∞ ∞
f (x, y) dx şi f (x, y) dy sunt (U C) (4)
a c

ı̂n raport cu y şi respectiv x. Dacă există cel puţin una dintre integralele
Z d Z +∞  Z +∞ Z d 
|f (x, y)| dx dy sau |f (x, y)| dy dx,
c a a c

atunci există şi integralele iterate din (3) şi are loc relaţia (3).

Corolarul 15 (Integrabilitatea integralei parametrice) Fie f : [a, +∞) × [c, +∞) → R continuă şi pozitivă.
Presupunem că integralele parametrice (4) sunt funcţii continue. Dacă există una dintre integralele iterate din (3),
atunci există şi cealaltă şi are loc relaţia (3).

16
Facultatea de Matematică
Calcul Integral şi Aplicaţii, Semestrul I
Lector dr. Lucian MATICIUC

Seminariile 7 – 9
Capitolul III. Integrale curbilinii

1. Să se calculeze integrala curbilinie de primul tip:


Z
yds ,
(γ)

unde (γ) este segmentul parabolei y 2 = 2px de la originea coordonatelor până la A (a, b), a, b > 0.

Rezolvare:
(
x = t,
Parametrizarea curbei date este (γ) : √
y = 2pt , t ∈ [0, a] .
Se foloseşte formula elementului de arc
q
2 2
ds = (x0 (t)) + (y 0 (t)) dt.

Conform teoremei de reducere, integrala curbilinie de primul tip se reduce la integrala Riemann:
Z ap r 1 2 Z ap Z ap
−1/2 1 0
I= 2pt 1 + (2pt) 2p dt = 2pt + p2 dt = 2pt + p2 2pt + p2 dt .
0 2 0 2p 0

2. Să se calculeze integrala curbilinie de primul tip:


Z
xyds ,
(γ)
(
x = a cos θ,
unde (γ) este sfertul din primul cadran al elipsei dată parametric
y = b sin θ.
Rezolvare:

Se foloseşte formula elementului de arc


q
2 2
ds = (x0 (θ)) + (y 0 (θ)) dθ.

Integrala curbilinie de primul tip se reduce la integrala Riemann:


Z π/2 p
I = ab sin θ cos θ a2 sin2 θ + b2 cos2 θ dθ .
0

Sunt două cazuri: a = b şi a 6= b.


În cazul a = b se foloseşte formula

sin (2θ) = 2 sin θ cos θ.

În cazul a 6= b, se poate folosi substituţia

tg (θ) = t

1
şi sunt utile formulele trigonometrice
t 1
sin θ = √ şi cos θ = √ .
1 + t2 1 + t2
Sau se observă că Z π/2
ab
q 0
I= (a2 − b2 ) sin2 θ + b2 sin2 θ dθ ,
2 0
2
deci merge substituţia sin θ = t.
3. Să se calculeze integrala curbilinie de primul tip:
Z
xyz ds ,
(γ)

x = t,



1√ 3



unde (γ) este curba din spaţiu y= 8t ,
 3
 z = 1 t2 , t ∈ [0, 1] .



2
Rezolvare:

Se foloseşte formula elementului de arc


q
2 2 2
ds = (x0 (t)) + (y 0 (t)) + (z 0 (t)) dt
şi se obţine
ds = (t + 1) dt.
4. Să se calculeze următoarea integrală curbilinie de primul tip
Z
ye−x ds
(γ)

2

unde (γ) : x = ln 1 + t , y = 2 arctg t − t + 3, t ∈ [0, 1].

Rezolvare:

Se obţine
ds = dt.
5. Să se calculeze integrala curbilinie de primul tip:
Z
xyds ,
(γ)

b√ 2
unde (γ) este sfertul din primul cadran al elipsei dată explicit y = a − x2 .
a
Rezolvare:

 x = t,
Parametrizarea curbei date este (γ) :
 y = b √a2 − t2 , t ∈ [0, a] .
a
Sunt două cazuri: a = b şi a 6= b.
Integrala curbilinie de primul tip se reduce la integrala Riemann:
s 2
b a p 2 b a p 2
Z  Z
b 1
I= 2
t a −t 1+ √ (−2t) dt = t (b − a2 ) t2 + a4 dt
a 0 a 2 a2 − t2 a 0
Z ap
b 1 0
(b2 − a2 ) t2 + a4 b2 − a2 t2 + a4 dt .
 
= 2 2
a 2 (b − a ) 0

2
6. Să se calculeze integrala curbilinie de primul tip:
Z
xds ,
(γ)

unde (γ) : y = x2 , x ∈ [0, 2] .

Rezolvare:

Integrala curbilinie de primul tip se reduce la integrala Riemann:


Z 2 q
2
I= x 1 + (2x) dx.
0

Deci Z 2
1 p 0
I= 1 + 4x2 1 + 4x2 dx
8 0

sau putem vedea integrala iraţională ca pe o integrală binomă, care se rezolvă cu substituţia (este
cazul al doilea)
1 2 1/2 1 2 −1/2
1 + 4x2 = t2 ⇔ x= t −1 ⇒ dx = t −1 · (2t) .
2 4

7. Să se calculeze integrala curbilinie de primul tip:


Z
xyds ,
(γ)

unde (γ) : y = x2 , x ∈ [−1, 1] .

Rezolvare:

Integrala curbilinie de primul tip se reduce la integrala Riemann:


Z 1 q Z 1 p
2
I= x · x2 1 + (2x) dx = x3 1 + 4x2 dx.
−1 −1

Am obţinut o integrală binomă, care se rezolvă cu substituţia (este cazul al doilea)

1 2 1/2 1 2 −1/2
1 + 4x2 = t2 ⇔ x= t −1 ⇒ dx = t −1 · (2t) .
2 4

8. Să se calculeze integrala curbilinie de primul tip:


Z
y 5 ds ,
(γ)

y4
unde (γ) : x = 4 , y ∈ [0, 2] .

Rezolvare:
t4
(
x= 4 ,
Parametrizarea curbei date este (γ) :
y = t, t ∈ [0, 2] .
Integrala curbilinie de primul tip se reduce la integrala Riemann:
s 2
Z 2  Z 2 p
1
I= t5 1 + · 4t3 dt = t5 1 + t6 dt .
0 4 0

3
Deci Z 2
1 p 0
I= 1 + t6 1 + t6 dt
6 0

sau putem vedea integrala iraţională ca pe o integrală binomă, care se rezolvă cu substituţia (este
cazul al doilea)
1/6 1 2 −5/6
1 + t6 = s2 ⇔ t = s2 − 1 ⇒ dt = s −1 · (2s) .
6

9. Să se calculeze integrala curbilinie de primul tip:


Z
z x2 + y 2 ds ,

(γ)


 x = t cos t,

unde (γ) : y = t sin t,


z = t, t ∈ [0, 1] .

Rezolvare:

Se obţine
Z 1 q
Z 1 p
2 2
I= t t2 cos2 t + t2 sin2 t (cos t − t sin t) + (sin t + t cos t) + 1 dt = t3 2 + t2 dt .
0 0

Am obţinut o integrală binomă, care se rezolvă cu substituţia (este cazul al doilea)


1/2 1 2 −1/2
2 + t2 = s2 ⇔ t = s2 − 2 ⇒ dt = s −2 · (2s) .
2

10. Să se calculeze integrala curbilinie de primul tip:


Z
x2 ds ,
(γ)

unde (γ) : x2 + y 2 = 2, x, y ≥ 0.

Rezolvare:
( √
x= 2 cos θ ,
Curba dată este sfertul din primul cadran al unui cerc dat parametric de (γ) : √
y= 2 sin θ , θ ∈ [0, π/2] .
Se obţine
Z π/2
2
p
2
√ Z π/2
I= 2 cos θ 2 sin θ + 2 cos2 θ dθ = 2 2 cos2 θ dθ
0 0
√ Z π/2 √
 
1 + cos (2θ) sin (2θ) θ=π/2
=2 2 dθ = 2 θ + .
2 2

0 θ=0

11. Să se calculeze integrala curbilinie de primul tip:


Z
xy ds ,
(γ)

a2
unde (γ) este dată de intersecţia sferei x2 + y 2 + z 2 = a2 cu cilindrul x2 + y 2 = 4 situată ı̂n primul
octant (x, y, z ≥ 0).

Rezolvare:

4
2 2
Deoarece se obţine z 2 = a2 − a4 = 3a4 , curba dată este un cerc ı̂n spaţiu cu ecuaţiile parametrice
 a
 x = cos θ ,


 2

 a
(γ) : y = sin θ ,
2


 √
 z = a 3 , θ ∈ [0, π/2] .


2
Se obţine
Z π/2 2 r
a3 π/2
2  a 2 Z
a a
I= sin θ cos θ − sin θ + cos θ + 0 dθ = sin θ cos θ dθ
0 4 2 2 8 0
a3 π/2 sin (2θ) a3
Z  
cos (2θ) θ=π/2
= dθ = − .
8 0 2 16 2

θ=0

12. Să se calculeze integrala curbilinie de primul tip:


Z
(x + y + z) ds ,
(γ)


 x = a cos t,

unde (γ) : y = a sin t,


z = bt, t ∈ [0, π/2] .

Rezolvare:

Se obţine p
ds = a2 + b2 dt.

13. Să se calculeze integrala curbilinie de primul tip:


Z
x2 + y 2 ln z ds ,

(γ)


 x = et cos t,

unde (γ) : y = et sin t,


z = et , t ∈ [0, 1] .

Rezolvare:

Se obţine √
ds = 3 et dt.

14. Să se calculeze lungimea următoarelor curbe:


1

(a) Intersecţia sferei x2 + y 2 + z 2 = 1 cu paraboloidul z = 2 x2 + y 2 .
1
(b) Intersecţia sferei x2 + y 2 + z 2 = 1 cu cilindrul x2 + y 2 = 4 , z ∈ [0, 2] .


 x = r cos t,


(c) Elicea circulară y = r sin t,

 z = ht, t ∈ [0, 2π] .

(d) x = t5 − 1 , y = t4 , t ∈ [0, 1] .

(e) Porţiunea din cercul x2 + y 2 = 4 pentru care y ≤ 0.

5
(f ) Pătratul cu vârfurile A (−1, 0) , B (0, 1) , C (1, 0) , D (0, −1) .

Rezolvare:

Trebuie calculată integrala curbilinie care dă lungimea unei curbe


Z
L (γ) = ds.
(γ)

(a) Deoarece se obţine



z 2 = 1 − 2z ⇔ z 2 + 2z − 1 = 0 ⇒ z1,2 = −1 ± 2,
avem  √ 2
x2 + y 2 = 1 − z1,2
2
= 1 − −1 ± 2 .

Nu putem alege z2 (deoarece x2 + y 2 = 1 − z22 = −2 − 2 2 ), deci
h  i1/2
x2 + y 2 = 1 − z12 = R2 , unde R = 2 21/2 − 1 .


 x = R cos θ ,

Curba dată este un cerc ı̂n spaţiu cu ecuaţiile parametrice (γ) : y = R sin θ ,

 √
z = 2 − 1, θ ∈ [0, 2π] .

(b) Deoarece se obţine √


1 3 3
z2 = 1 − = ⇔ z1,2 = ±
4 4 2
(alegem doar pe z1 ≥ 0), avem
3 1
x2 + y 2 = 1 − = .
4 4
1


 x = cos θ ,



 2
Curba dată este un cerc ı̂n spaţiu cu ecuaţiile parametrice (γ) : 1
y = sin θ ,

 2




z = 23 , θ ∈ [0, 2π] .

x2 y2
15. Să se determine integrala care dă lungimea elipsei + = 1.
a2 b2

16. Să se calculeze integrala curbilinie de al doilea tip1 :


Z
x2 − y 2 dx ,

I= _
(OA)

_
unde (OA) este segmentul parabolei y = x2 cuprins ı̂ntre x = 0 şi x = 2.

Rezolvare:
(
_ x = t,
Parametrizarea curbei date este (OA) :
y = t2 , t ∈ [0, 2] .
Conform teoremei de reducere, integrala curbilinie de al doilea tip se reduce la integrala Riemann:
Z 2
t2 − t4 d (t) .

I=
0
1 La integrala curbilinie de al doilea tip contează sensul de parcurgere al curbei, mai precis, are loc:
Z Z
_ P (x, y) dx + Q (x, y) dy = − _ P (x, y) dx + Q (x, y) dy..
(AB) (BA)

6
17. Să se calculeze integrala curbilinie de al doilea tip:
Z
_
2xydx + x2 dy ,
(OA)

_
unde (OA) este

(a) parabola y 2 = x care uneşte O (0, 0) cu A (1, 1) ;

(b) dreapta y = x care uneşte O (0, 0) cu A (1, 1) ;

(c) curba y = x3 care uneşte O (0, 0) cu A (1, 1) .

Rezolvare:
(
_ x = t,
(a) Parametrizarea curbei date este (OA) : √
y = t, t ∈ [0, 1] .
Conform teoremei de reducere, integrala curbilinie de al doilea tip se reduce la integrala Riemann:
Z 1h √ √ i Z 1  √ 
2 2 1
I= 2t t d (t) + t d t = 2t t + t √ dt .
0 0 2 t

18. Să se calculeze integrala curbilinie de al doilea tip:


Z
_
xydx − y 2 dy ,
(AB)

x = t2 ,
(
_
unde (AB) :
y = t3 , t ∈ [0, 1] .
19. Să se calculeze integrala curbilinie de al doilea tip:
Z
_
x2 dx + 2xydy ,
(AB)

_
unde (AB) este jumătatea superioară a unei elipse parcursă ı̂n sens trigonometric.

Rezolvare:
(
x = a cos θ ,
Parametrizarea curbei date este (γ) :
y = b sin θ , θ ∈ [0, π] .
Integrala curbilinie de al doilea tip se reduce la integrala Riemann:
Z πh i Z π
2
(a cos θ) d (a cos θ) + 2a cos θ · b sin θ d (b sin θ) = 2ab2 − a3 sin θ cos2 θ dθ

I=
0 0
Z π
0
= − 2ab2 − a3 cos2 θ (cos θ) dθ.

0

20. Să se calculeze integrala curbilinie de al doilea tip:


Z p
_
1 − x2 dx + xdy ,
(AB)

_
y2
unde (AB) este curba x2 + 4 = 1 parcursă ı̂n sens trigonometric.

Rezolvare:

7
(
x = cos θ ,
Parametrizarea curbei date este (γ) :
y = 2 sin θ , θ ∈ [0, 2π] .
Integrala curbilinie de al doilea tip se reduce la integrala Riemann:
Z 2π hp i Z 2π
|sin θ| sin θ + 2 cos2 θ dθ
 
I= 1− cos2 θ d (cos θ) + cos θ d (2 sin θ) =
0 0
Z π Z 2π Z 2π
= sin2 θ dθ − sin2 θ dθ + 2 cos2 θdθ.
0 π 0

Sunt utile formulele trigonometrice

sin (2θ) = 2 sin θ cos θ şi cos (2θ) = 2 cos2 θ − 1 = 1 − 2 sin2 θ.

21. Să se calculeze integrala curbilinie de al doilea tip:


Z
dx
,
(γ) x3 + y 3
(
x = a cos θ ,
unde (γ) :
y = a sin θ , θ ∈ [0, π/2] .

Rezolvare:

Obţinem integrala Riemann:


π/2 π/2 π/2
−a −1
Z Z Z
d (a cos θ) sin θ sin θ
I= 3 3 = a3 dθ = 2 dθ.
0 (a cos θ) + (a sin θ) 0 cos3 θ + sin3 θ a 0 cos3 θ + sin3 θ

Având ı̂n vedere că integrala este pară ı̂n sin şi cos se poate folosi substituţia

tg (θ) = t.

Sunt utile formulele trigonometrice

t 1
sin θ = √ şi cos θ = √ .
1 + t2 1 + t2

22. Să se calculeze integrala curbilinie de al doilea tip:


Z
dx dy
− ,
(γ) 2a + y a + x

unde (γ) este curba dată de x2 + y 2 + 2ay = 0, a > 0, cu x + y ≥ 0, cu extremitatea iniţială ı̂n
A (a, −a) .

Rezolvare:
2
Observăm că avem un cerc x2 + (y + a) = a2 cu centrul ı̂n C (0, −a) şi de rază a. Curba (γ) este
arcul din cercul anterior parcurs de la A (a, −a) până la originea O (0, 0) .
(
x = a cos θ ,
2
Parametrizarea curbei date este (γ) :
y + a = a sin θ , θ ∈ [0, π/2] .
2 Ecuaţiile parametrice ale unui cerc
(x − a)2 + (y − b)2 = R2
( (
x − a = R cos θ , x = a + R cos θ ,
sunt sau, echivalent,
y − b = R sin θ , θ ∈ [0, 2π] y = b + R sin θ , θ ∈ [0, 2π] .

8
Obţinem integrala Riemann:
π/2   Z π/2  
d (a sin θ − a) −a sin θ
Z
d (a cos θ) a cos θ
I= − = − dθ
0 2a + (a sin θ − a) a + a cos θ 0 a + a sin θ a + a cos θ
Z π/2  
sin θ cos θ
=− + dθ.
0 1 + sin θ 1 + cos θ

Calculăm
1 + sin θ − 1
Z Z Z Z
sin θ 1
dθ = dθ = dθ − dθ
1 + sin θ 1 + sin θ 1 + sin θ
1 + cos θ − 1
Z Z Z Z
cos θ 1
dθ = dθ = dθ − dθ.
1 + cos θ 1 + cos θ 1 + cos θ
Ambele integrale se pot rezolva folosind substituţia trigonometrică universală
 
θ
tg = t.
2

Sunt utile formulele trigonometrice

2t 1 − t2
sin θ = şi cos θ = .
1 + t2 1 + t2

23. Să se calculeze integrala curbilinie de al doilea tip:


Z
ydx − (x − a) dy ,
(γ)

2
(x − a) y2
unde (γ) : + = 1.
a2 b2
Rezolvare:
(
x − a = a cos θ ,
Parametrizarea elipsei date este (γ) :
y = b sin θ , θ ∈ [0, 2π] .
24. Să se calculeze integrala curbilinie de al doilea tip:
Z p
a2 − x2 dx + xzdy + x2 + y 2 dz ,

_
z
(AB)


 x = a cos t,
_ 
unde (AB) : y = a sin t,


z = bt, t ∈ [0, π/2] .

Rezolvare:

Obţinem integrala Riemann:


Z π/2  q   
2 2 2 2
I= bt a − (a cos t) d (a cos t) + a cos t · bt d (a sin t) + (a cos t) + (a sin t) d (bt)
0
Z π/2
−a2 b t sin2 t + a2 b t cos2 t + a2 b t sin2 t dt.

=
0

Sunt utile formulele trigonometrice

sin (2θ) = 2 sin θ cos θ şi cos (2θ) = 2 cos2 θ − 1 = 1 − 2 sin2 θ.

Apoi se foloseşte metoda de integrare prin părţi.

9
25. Să se calculeze integrala curbilinie de al doilea tip:
Z
_
xdx + xydy + xyzdz ,
(AB)

_ √
unde (AB) : x = et , y = e−t , z = 2t, t ∈ [0, 1] .
26. Să se calculeze integrala curbilinie de al doilea tip:

√ √ √
Z
_
yzdx + xzdy + xydz ,
(AB)

_
unde (AB) : x = t, y = t2 , z = t3 , t ∈ [0, 1] .
27. Să se calculeze următoarea integrală curbilinie constatând ı̂n prealabil că este independentă de
drum:
xdy − ydx
Z
_ 2 ,
(AB) (x − y)
_
unde (AB) este arcul de curbă ce uneşte punctul A (0, −1) cu B (1, 0) .

Rezolvare:

Avem de calculat integrala Z


_
P (x, y) dx + Q (x, y) dy.
(AB)
−y x
În cazul nostru P (x, y) = (x−y)2
şi Q (x, y) = (x−y)2
.
Se verifică mai ı̂ntâi condiţiile suficiente

∂P −x − y ∂Q
(x, y) = 3 = (x, y) , pentru orice (x, y)
∂y (x − y) ∂x

care asigură existenţa unei primitive3 F a formei diferenţiale P (x, y) dx + Q (x, y) dy care se inte-
grează.
Apoi se rezolvă sistemul
∂F


 (x, y) = P (x, y) ,
∂x

 ∂F

 (x, y) = Q (x, y) ,
∂y
care defineşte primitiva F (integrându-se una din ecuaţii)4 .
3 Fie D ⊆ R2 un domeniu simplu conex (domeniu conex cu proprietatea că orice curbă ı̂nchisă situată ı̂n D mărgineşte un

domeniu care este de asemenea ı̂n D; adică domeniul D nu are “găuri”). Fie două funcţii P, Q : D → R. Dacă avem forma
diferenţială de ordinul 1
P (x, y) dx + Q (x, y) dy,
atunci aceasta este o diferenţială totală exactă dacă şi numai dacă are loc relaţia
∂P ∂Q
(x, y) = (x, y) , pentru orice (x, y) ∈ D.
∂y ∂x

4 Dacă forma diferenţială P (x, y) dx + Q (x, y) dy este exactă, atunci există F ∈ C 1 (D) astfel ı̂ncât au loc
 ∂F
 ∂x (x, y) = P (x, y) ,

 ∂F

 (x, y) = Q (x, y) ,
∂y
pentru orice (x, y) ∈ D.

10
Obţinem, dacă luăm, de exemplu, prima ecuaţie,
−2+1
−y (x − y)
Z Z
F (x, y) = P (x, y) dx = 2 dx = −y + C (y) .
(x − y) −2 + 1

Folosim a doua ecuaţie şi obţinem


 0
y x
+ C (y) = 2 ⇔ C 0 (y) = 0 ⇔ C (y) = C,
x−y y (x − y)

deci
y
F (x, y) = + C, C ∈ R.
x−y
Se aplică formula lui Leibniz-Newton şi obţine valoarea integralei
Z (x,y)=B
I = _ dF (x, y) = F (x, y) = F (B) − F (A) = F (1, 0) − F (0, −1) .

(AB) (x,y)=A

28. Să se calculeze următoarea integrală curbilinie constatând ı̂n prealabil că este independentă de
drum:
ydx − xdy
Z
,
_
(AB) y2
_
unde (AB) este arcul de curbă ce uneşte punctul A (1, 2) cu B (2, 1) .

Rezolvare:

Avem de calculat integrala Z


_
P (x, y) dx + Q (x, y) dy.
(AB)
1 −x
În cazul nostru P (x, y) = y şi Q (x, y) = y2 .
Se verifică mai ı̂ntâi condiţiile suficiente

∂P −1 ∂Q
(x, y) = 2 = (x, y) , pentru orice (x, y) ,
∂y y ∂x

care asigură existenţa unei primitive F a formei diferenţiale P (x, y) dx + Q (x, y) dy care se inte-
grează.
Apoi se rezolvă sistemul care defineşte primitiva F (integrându-se una din ecuaţii).
Obţinem, dacă luăm, de exemplu, prima ecuaţie,
Z Z
1 x
F (x, y) = P (x, y) dx = dx = + C (y) .
y y

Folosim a doua ecuaţie şi obţinem


 0
x −x
+ C (y) = ⇔ C 0 (y) = 0 ⇔ C (y) = C,
y y y2

deci
x
F (x, y) = + C, C ∈ R.
y
Se aplică formula lui Leibniz-Newton şi obţine că
Z (x,y)=B
I = _ dF (x, y) = F (x, y) = F (B) − F (A) = F (2, 1) − F (1, 2) .

(AB) (x,y)=A

11
29. Să se calculeze următoarea integrală curbilinie constatând ı̂n prealabil că este independentă de
drum:
Z
xdx + ydy + zdz
_
p ,
(AB) x2 + y 2 + z 2
_
unde (AB) este arcul de curbă ce uneşte punctul A (1, 1, 1) cu B (3, 4, 5).
Rezolvare:

Avem de calculat integrala


Z
_
P (x, y, z) dx + Q (x, y, z) dy + R (x, y, z) dz.
(AB)

x y z
În cazul nostru P (x, y, z) = √ , Q (x, y, z) = √ , R (x, y, z) = √ .
x2 +y 2 +z 2 x2 +y 2 +z 2 x2 +y 2 +z 2

Se verifică mai ı̂ntâi condiţiile suficiente



 ∂P (x, y, z) = −xy x2 + y 2 + z 2 −3/2 = ∂Q

 (x, y, z) ,
∂y ∂x





∂Q ∂R
 −3/2
(x, y, z) = −yz x2 + y 2 + z 2 = (x, y, z) ,
 ∂z
 ∂y


 ∂R (x, y, z) = −xz x2 + y 2 + z 2 −3/2 = ∂P

 
 (x, y, z) , pentru orice (x, y, z)
∂x ∂z
care asigură existenţa unei primitive5 F a formei diferenţiale P (x, y, z) dx+Q (x, y, z) dy+R (x, y, z) dz
care se integrează.
Apoi se rezolvă sistemul
∂F


 (x, y, z) = P (x, y, z) ,


 ∂x

 ∂F
(x, y, z) = Q (x, y, z)

 ∂y

 ∂F (x, y, z) = R (x, y, z) ,



∂z
care defineşte primitiva F (integrându-se una din ecuaţii)6 .
5 Fie D ⊆ R3 un domeniu simplu conex (domeniu conex cu proprietatea că orice curbă ı̂nchisă situată ı̂n D mărgineşte un
domeniu care este de asemenea ı̂n D; adică domeniul D nu are “găuri”). Fie două funcţii P, Q, R : D → R. Dacă avem forma
diferenţială de ordinul 1
P (x, y, z) dx + Q (x, y, z) dy + R (x, y, z) dz,
atunci aceasta este o diferenţială totală exactă dacă şi numai dacă au loc relaţiile
 ∂P ∂Q

 (x, y, z) = (x, y, z) ,
 ∂y

 ∂x


∂Q ∂R

(x, y, z) = (x, y, z) ,


 ∂z ∂y

 ∂R (x, y, z) = ∂P (x, y, z) ,



∂x ∂z
pentru orice (x, y, z) ∈ D.
6 Dacă forma diferenţială P (x, y, z) dx + Q (x, y, z) dy + R (x, y, z) dz este exactă, atunci există F ∈ C 1 (D) astfel ı̂ncât au loc
∂F


 (x, y, z) = P (x, y, z) ,



 ∂x

 ∂F
(x, y, z) = Q (x, y, z)

 ∂y


 ∂F (x, y, z) = R (x, y, z) ,



∂z
pentru orice (x, y, z) ∈ D.

12
Obţinem, dacă luăm, de exemplu, prima ecuaţie,
Z Z Z
x 1 −1/2 0
F (x, y, z) = P (x, y, z) dx = p dx = x2 + y 2 + z 2 x2 + y 2 + z 2 x
dx
x2 + y 2 + z 2 2
1/2
= x2 + y 2 + z 2

+ C (y, z) .

Folosim a doua ecuaţie şi obţinem


p 0 y
x2 + y 2 + z 2 + C (y, z) = p ⇔ Cy0 (y, z) = 0 ⇔ C (y, z) = C (z) ,
y x2 + y2 + z2

deci p
F (x, y, z) = x2 + y 2 + z 2 + C (z) .
Folosim a treia ecuaţie şi obţinem
p 0 z
x2 + y 2 + z 2 + C (z) = p ⇔ C 0 (z) = 0 ⇔ C (z) = C,
z x2 + y2 + z2

deci p
F (x, y, z) = x2 + y 2 + z 2 + C, C ∈ R.
Se aplică formula lui Leibniz-Newton şi obţine valoarea integralei
Z (x,y,z)=B
I = _ dF (x, y, z) = F (x, y, z) = F (B) − F (A) = F (3, 4, 5) − F (1, 1, 1) .

(AB) (x,y,z)=A

30. Să se studieze dacă următoarele forme diferenţiale sunt exacte şi ı̂n caz afirmativ să se calculeze o
primitivă a lor:

4x3 y 3 − 3y 2 + 5 dx + 3x4 y 2 − 6xy − 4 dy.


 
(a)
   
1 1 z x 1
(b) z − dx + 2 dy + − dz.
x2 y x2 + z 2 xy x2 + z 2 xy

Rezolvare:

(a) În cazul nostru P (x, y) = 4x3 y 3 − 3y 2 + 5 şi Q (x, y) = 3x4 y 2 − 6xy − 4 .
Se verifică mai ı̂ntâi condiţiile suficiente
∂P ∂Q
(x, y) = 12x3 y 2 − 6y = (x, y) , pentru orice (x, y)
∂y ∂x
care asigură existenţa unei primitive F a formei diferenţiale P (x, y) dx + Q (x, y) dy.
Apoi se rezolvă sistemul
∂F


 (x, y) = P (x, y) ,
∂x

 ∂F

 (x, y) = Q (x, y) .
∂y
Obţinem, dacă luăm, de exemplu, prima ecuaţie,
Z Z
4x3 y 3 − 3y 2 + 5 dx = x4 y 3 − 3xy 2 + 5x + C (y) .

F (x, y) = P (x, y) dx =

Folosim a doua ecuaţie şi obţinem


0
x4 y 3 − 3xy 2 + 5x + C (y) y = 3x4 y 2 − 6xy − 4 ⇔ C 0 (y) = −4 ⇔ C (y) = −4y + C,

deci
F (x, y) = x4 y 3 − 3xy 2 + 5x − 4y + C, C ∈ R.

13
z z z x 1
(b) În cazul nostru P (x, y, z) = x2 y − x2 +z 2 , Q (x, y, z) = xy 2 , R (x, y, z) = x2 +z 2 − xy .
Se verifică mai ı̂ntâi condiţiile suficiente
 ∂P −z ∂Q
 (x, y, z) = 2 2 = (x, y, z) ,
∂y x y ∂x





 ∂Q

1 ∂R
(x, y, z) = 2 = (x, y, z) ,

 ∂z xy ∂y

∂R z 2 − x2 1 ∂P


(x, y, z) = 2 + x2 y = ∂z (x, y, z) , pentru orice (x, y, z)



∂x 2
(x + z )2

care asigură existenţa unei primitive F a formei diferenţiale P (x, y, z) dx+Q (x, y, z) dy+R (x, y, z) dz.
Apoi se rezolvă sistemul
∂F


 (x, y, z) = P (x, y, z) ,


 ∂x

 ∂F
(x, y, z) = Q (x, y, z) ,

 ∂y

 ∂F (x, y, z) = R (x, y, z) .



∂z
Obţinem, dacă luăm, de exemplu, a doua ecuaţie,
−z
Z Z
z
F (x, y, z) = Q (x, y, z) dy = dx = + C (x, z) .
xy 2 xy
Folosim prima ecuaţie şi obţinem
 0
−z z z z
+ C (x, z) = 2 − 2 ⇔ Cx0 (x, z) = −
xy x x y x + z2 x2 + z 2
x
⇔ C (x, z) = −arctg + C (z) ,
z
deci
z x
F (x, y, z) = − − arctg + C (z) .
xy z
Folosim a treia ecuaţie şi obţinem
 0
z x x 1
− − arctg + C (z) = 2 2
− ⇔ C 0 (z) = 0 ⇔ C (z) = C,
xy z z x +z xy

deci
z x
F (x, y, z) = − − arctg + C, C ∈ R.
xy z
31. Determinaţi aria domeniului mărginit de curba

x = a cos3 t,
(
(γ) : , t ∈ [0, 2π]
y = a sin3 t,

Rezolvare:

Trebuie calculată integrala curbilinie care dă aria unui domeniu mărginit de o curbă ı̂nchisă
Z
1
A (γ) = xdy − ydx.
2 (γ)
(
x = a cos t,
32. Să se calculeze aria elipsei , t ∈ [0, 2π] .
y = b sin t,

14
Facultatea de Matematică
Calcul Integral şi Aplicaţii, Semestrul I
Lector dr. Lucian MATICIUC

Seminariile 10 – 12
Capitolul IV. Integrala dublă

1. Să se calculeze următoarea integrală dublă pe un domeniu dreptunghiular


ZZ
5xy 2 − 2x3 dxdy ,

D

unde D este dreptunghiul D = [1, 3] × [2, 5].

Rezolvare:

Avem explicitarea domeniului

D = (x, y) ∈ R2 : 1 ≤ x ≤ 3, 2 ≤ y ≤ 5 .


Avem următoarea reducere a integralei duble la două integrale iterate:


Z 3 Z 5  Z 5 Z 3 
2 3 2 3
 
I= 5xy − 2x dy dx = 5xy − 2x dx dy.
1 2 2 1
Z 5
5xy 2 − 2x3 dy şi vom ţine cont de faptul că atunci când integrăm ı̂n

Vom calcula integrala
2
raport cu o variabilă vom considera cealaltă variabilă drept constantă. Astfel
5  y=5 
y3 53 23
Z    
5xy 2 − 2x3 dy = − 2x3 y − 2x3 · 5 − 5x · − 2x3 · 2 =

5x = 5x ·
2 3 y=2 3 3
 4   
5 40 585
= x − 10x3 − x − 4x3 = x − 6x3 .
3 3 3
Deci  x=3
3
585 x2 x4
Z   
585
I= x − 6x3 dx = −6 .
1 3 3 2 4 x=1

2. Să se calculeze integrala precedentă folosind reducerea


Z 5 Z 3 
5xy 2 − 2x3 dx dy

2 1

(s-a schimbat ordinea de integrare).


3. Să se calculeze următoarea integrală dublă pe un domeniu dreptunghiular

x2
ZZ
2
dxdy ,
D1+y

unde D este dreptunghiul D = [2, 5] × [0, 1].

Rezolvare:

Avem explicitarea domeniului

D = (x, y) ∈ R2 : 2 ≤ x ≤ 5, 0 ≤ y ≤ 1 .


1
4. Să se calculeze următoarea integrală dublă:
ZZ p
|y − x2 | dxdy ,
D

unde D este domeniul dat de x ∈ [−1, 1] , y ∈ [0, 1] .


Rezolvare:

Aplicăm teorema de reducere şi obţinem


Z 1 Z 1 p

I= 2
|y − x | dy dx.
−1 0

Deoarece x ∈ [−1, 1] ,
Z 1 p Z x2 p Z 1 p
|y − x2 | dy = 2
x − y dy + y − x2 dy
0 0 x2
2
y=x y=1
x − y 3/2 y − x2 3/2
2
= +
−3/2 3/2

2
y=0 y=x

2  3/2 3/2 
= 1 − x2 − −x2
3
2  3/2 3/2  2  3/2 3

= 1 − x2 − −x2 = 1 − x2 − |x| .
3 3

Deci
Z 1 Z 1 Z 0 Z 1
2  3/2 3
 2 3/2
1 − x2 1 − x2 −x3 dx − x3 dx.

I= − |x| dx = dx −
3 −1 3 −1 −1 0

3/2
1 − x2
R
Dacă privim integrala dx ca integrală binomă, atunci suntem ı̂n cazul al treilea. Deci
este utilă substituţia

1 − x2 −1/2 −1 2 −3/2 −3/2


= t2 ⇔ x = t2 + 1 ⇒ dx = t +1 · 2tdt = −t t2 + 1 dt.
x2 2
Deci
−1/2 2 3/2
Z Z   
2 3/2 2
−3/2
(−t) t2 + 1

1−x dx = 1− t +1 dt
3/2
t2
Z  Z
2
−3/2 −3
=− 2
t t + 1 dt = − t4 t2 + 1 dt .
t +1
3/2
1 − x2
R
Sau putem calcula integrala dx folosind substituţia trigonometrică

x = sin t ⇒ dx = cos tdt.

Deci Z Z Z
2 3/2 2
3/2
cos4 t dt,

1−x dx = 1 − sin t cos tdt =

pentru care folosim, de două ori, formula

1 + cos (2t)
cos2 t = .
2

2
5. Să se calculeze următoarea integrală dublă schimbând convenabil ordinea de integrare
ZZ
y
3/2
dxdy ,
D (1 + x + y 2 )
2

unde D este dreptunghiul [0, 1] × [0, 1].

Rezolvare:

Avem următoarea reducere a integralei duble la două integrale simple:


Z 1 Z 1 ! Z 1 Z 1 !
y y
I= 3/2
dy dx = 3/2
dx dy.
0 0 (1 + x2 + y 2 ) 0 0 (1 + x2 + y 2 )

Z 1
y
Vom calcula integrala 3/2
dy şi vom ţine cont de faptul că atunci când integrăm
0 (1 + x + y 2 )
2
ı̂n raport cu o variabilă vom considera cealaltă variabilă drept constantă. Astfel, folosind prima
metodă de schimbare de variabilă,
Z 1
1 1
Z
y −3/2 0
3/2
dy = 1 + x2 + y 2 · 1 + x2 + y 2 y dy =
2
0 (1 + x + y ) 2 2 0

− 3 +1 y=1
1 1 + x2 + y 2 2 h  1
2 −2
 1i
2 −2 1 1
= 3 = − 2 + x − 1 + x =√ −√ .
2 −2 + 1
2
x +1 2
x +2
y=0

Deci !
Z 1 Z 1 Z 1  
y 1 1
I= 3/2
dy dx = √ −√ dx
0 0 (1 + x2 + y 2 ) 0 x2 + 1 x2 + 2
√ x=1
h  p   p i x=1 x + x 2 + 1
= ln x + x2 + 1 − ln x + x2 + 2 = ln √

2

x=0 x + x + 2 x=0
√ √
1+ 2 1 √ 1+ 2
= ln √ − ln √ = ln 2 √ .
1+ 3 2 1+ 3
Z 1 Z 1
y −3/2
Observăm că dacă vom calcula mai ı̂ntâi 3/2
dx = y 1 + x2 + y 2 dx atunci
2
0 (1 + x + y ) 2 0
calculul integralei ı̂n raport cu dx este mai dificil.
6. Să se calculeze următoarea integrală dublă:
ZZ
x2 + y dxdy ,

D

unde D este domeniul mărginit de parabolele y = x2 şi y 2 = x.

Rezolvare:

Desenăm mai ı̂ntâi domeniul şi observăm că este simplu ı̂n raport cu ambele axe.
Explicităm domeniul ı̂n cazul ı̂n care privim domeniul D ca simplu ı̂n raport cu axa Oy.
Să luăm x ∈ [0, 1] arbitrar fixat şi prin
√ x ducem o paralelă la axa Oy care va intersecta domeniul
D ı̂n curbele ϕ1 (x) = x2 şi ϕ2 (x) = x.
Obţinem explicitarea: √
D = (x, y) ∈ R2 : 0 ≤ x ≤ 1, x2 ≤ y ≤ x .


Aplicăm teorema de reducere şi obţinem


√ !
Z 1 Z x
2

I= x + y dy dx
0 x2

3
şi calculăm mai ı̂ntâi

x  y=√x √ 2! 2 !
y 2 √ x2
Z 
2 2
2 x
− x2 x2 +

x + y dy = x y + = x x+
x2 2 y=x2
2 2
√ 2! 2 !
2
√ x 2 2 x2 √ x 3x4 1 3
= x x+ − x ·x + = x2 x + − = x5/2 + x − x4 ,
2 2 2 2 2 2

deci  x=1
1  5/2+1
1 x2
Z  
5/2 1 3 4 x 3 x5
I= x + x − x dx = + −
0 2 2 5/2 + 1 2 2 2 5 x=0
2 1 3 33
= + − = .
7 4 10 140
7. Să se calculeze integrala precedentă folosind faptul că domeniul D este simplu ı̂n raport cu axa Ox.

Rezolvare:
 √
Avem explicitarea D = (x, y) ∈ R2 : 0 ≤ y ≤ 1, y 2 ≤ x ≤ y .
8. Să se calculeze următoarea integrală dublă:

x2
ZZ
p dxdy ,
D x2 + y 2

unde D este domeniul mărginit de x = 0, y = 1, y = 2 şi y = x.


Rezolvare:

Desenăm mai ı̂ntâi domeniul şi observăm că este simplu ı̂n raport cu axa Ox.
Explicităm domeniul. Să luăm y ∈ [1, 2] arbitrar fixat şi prin y ducem o paralelă la axa Ox care va
intersecta domeniul D ı̂n curbele ϕ1 (y) = 0 şi ϕ2 (y) = y. Deci obţinem explicitarea

D = (x, y) ∈ R2 : 1 ≤ y ≤ 2, 0 ≤ x ≤ y .


Aplicăm teorema de reducere şi obţinem


!
2 y
x2
Z Z
I= p dx dy.
1 0 x2 + y 2

Calculăm mai ı̂ntâi


Z y Z y x=y Z y p
x2 p 0 p
dx = x· x2 + y 2 dx = x · x2 + y 2 − x2 + y 2 dx

p
0 x2 + y 2 0 x x=0 0
Z y 2 Z y Z y
p x + y2 p x2 y2
= y 2y 2 − p dx = y 2y 2 − p dx − p dx ,
0 x2 + y 2 0 x2 + y 2 0 x2 + y 2

deci
!
y √
p
x2 y + 2y 2
Z p  p  x=y
2
2 dx = y 2y 2 − y ln x + x2 + y 2 = y 2 2 − y 2 ln

p p
x2 + y 2
0 x=0 y2
√  √ 
= y 2 2 − y 2 ln 1 + 2 .

Obţinem
√ √  y=2 √ 
2
1 2√ √ 
√
2 − ln 1 + 2 y 3 7 2 − ln 1 + 2
Z  
1 2 
I= y 2 − y ln 1 + 2 dy = = .
1 2 2 2 3 y=1 6

4
9. Să se calculeze integrala precedentă folosind faptul că, deşi domeniul D nu este simplu ı̂n raport cu
axa Oy, se poate totuşi descompune ı̂n două subdomenii care sunt simple ı̂n raport cu axa Oy.

Rezolvare:

Obţinem:
x2 x2
ZZ ZZ
I= p dxdy + p dxdy ,
D1 x2 + y 2 D2 x2 + y 2
 
unde D1 = (x, y) ∈ R2 : 0 ≤ x ≤ 1, 1 ≤ y ≤ 2 şi D2 = (x, y) ∈ R2 : 1 ≤ x ≤ 2, x ≤ y ≤ 2 .
10. Să se calculeze următoarea integrală dublă
ZZ p
y 2 R2 − x2 dxdy ,
D

2 2 2
unde D este discul x + y ≤ R .

Rezolvare:

Obţinem explicitarea:
n p p o
D = (x, y) ∈ R2 : −R ≤ x ≤ R, − R2 − x2 ≤ y ≤ R2 − x2 .

11. Să se calculeze următoarea integrală dublă:


ZZ
(1 − y) dxdy ,
D

unde D este domeniul dat de x2 + y 2 − 2y ≤ 0, y ≤ x2 şi x ≥ 0.

Rezolvare:

Să observăm că


2
x2 + y 2 − 2y = 0 ⇔ x2 + (y − 1) = 1
reprezintă cercul cu centrul C (0, 1) şi raza 1.
Obţinem explicitarea
n p o
D = (x, y) ∈ R2 : 0 ≤ x ≤ 1, 1 − 1 − x2 ≤ y ≤ x2 .

Aplicăm teorema de reducere şi obţinem

x2
!
Z 1 Z
I= √ (1 − y) dy dx.
0 1− 1−x2

Calculăm mai ı̂ntâi


x2  y=x2
y 2
Z 
√ (1 − y) dy = y− .
1− 1−x2 2 y=1−√1−x2

12. Să se calculeze următoarea integrală dublă:


ZZ
xy dxdy ,
D

5
unde D este domeniul mărginit de xy = 1 şi x + y = 2 .

Rezolvare:

5
Obţinem explicitarea
 
1 1 5
D= (x, y) ∈ R2 : ≤ x ≤ 2, ≤y ≤ −x .
2 x 2

Aplicăm teorema de reducere şi obţinem


!
Z 2 Z −x+5/2
I= xy dy dx.
1/2 1/x

Calculăm mai ı̂ntâi


y=−x+5/2 " 2 #
−x+5/2
y 2
Z
1 5 1
xy dy = x = x −x + − 2 .
1/x 2 y=1/x 2 2 x

13. Să se calculeze următoarea integrală dublă:


ZZ
(|x| + |y|) dxdy ,
D

unde D este domeniul dat de |x| + |y| ≤ 1.

Rezolvare:

În cadranul I domeniul devine D1 : x + y ≤ 1 şi x, y ≥ 0 (deci D1 este mărginit de dreptele


y = −x + 1, x = 0, y = 0).
În cadranul II domeniul devine D2 : −x + y ≤ 1 şi x ≤ 0, y ≥ 0 (deci D2 este mărginit de dreptele
y = x + 1, x = 0, y = 0).
În cadranul III domeniul devine D3 : −x − y ≤ 1 şi x, y ≤ 0 (deci D3 este mărginit de dreptele
y = −x − 1, x = 0, y = 0).
În cadranul IV domeniul devine D4 : x − y ≤ 1 şi x ≥ 0, y ≤ 0 (deci D4 este mărginit de dreptele
y = x − 1, x = 0, y = 0).
Atunci D = D1 ∪ D2 ∪ D3 ∪ D4 şi
ZZ ZZ ZZ ZZ
I= (|x| + |y|) dxdy + (|x| + |y|) dxdy + (|x| + |y|) dxdy + (|x| + |y|) dxdy,
D1 D2 D3 D4

unde
D1 = (x, y) ∈ R2

: 0 ≤ x ≤ 1, 0 ≤ y ≤ −x + 1 ,
D2 = (x, y) ∈ R2

: −1 ≤ x ≤ 0, 0 ≤ y ≤ x + 1 ,
D3 = (x, y) ∈ R2

: −1 ≤ x ≤ 0, −x − 1 ≤ y ≤ 0 ,
D4 = (x, y) ∈ R2

: 0 ≤ x ≤ 1, x − 1 ≤ y ≤ 0 .

14. Să se calculeze următoarea integrală dublă:


ZZ
1
√ dxdy ,
D x

unde D este domeniul dat de y 2 ≤ 8x, y ≤ 2x şi y + 4x ≤ 24.

Rezolvare:

Domeniul D nu este simplu ı̂n raport cu axa Oy, dar se poate descompune ı̂n trei subdomenii care
sunt simple ı̂n raport cu axa Oy.
Deci ZZ ZZ ZZ
1 1 1
I= √ dxdy + √ dxdy + √ dxdy ,
D1 x D2 x D3 x

6
unde n √ o
D1 = (x, y) ∈ R2 : 0 ≤ x ≤ 2, − 8x ≤ y ≤ 2x ,
n √ √ o
D2 = (x, y) ∈ R2 : 2 ≤ x ≤ 9/2, − 8x ≤ y ≤ 8x ,
n √ o
D3 = (x, y) ∈ R2 : 9/2 ≤ x ≤ 8, − 8x ≤ y ≤ −4x + 24 .

15. Să se calculeze următoarea integrală dublă:


ZZ
y dxdy ,
D

unde D este domeniul dat de x2 + y 2 ≥ 2x, y 2 ≤ 8x şi 0 ≤ x ≤ 2.

Rezolvare:

Domeniul D nu este simplu ı̂n raport cu axa Oy, dar se poate descompune ı̂n două subdomenii care
sunt simple ı̂n raport cu axa Oy.
Deci ZZ ZZ
I= y dxdy + y dxdy ,
D1 D2

unde n √ p o
D1 = (x, y) ∈ R2 : 0 ≤ x ≤ 2, − 2x ≤ y ≤ − 2x − x2 ,
n √ p o
D2 = (x, y) ∈ R2 : 0 ≤ x ≤ 2, 2x ≤ y ≤ 2x − x2 .

16. Să se calculeze următoarea integrală dublă

x2
ZZ
2
dxdy ,
Dy

unde D este domeniul mărginit de curbele x = 2, y = x şi xy = 1.

17. Să se calculeze aria domeniului D mărginit de parabola y = x2 − 1 şi de dreapta y = x + 1.

Rezolvare:

Trebuie
 explicitat domeniul D care este simplu ı̂n raport
cu Oy (ı̂n raport cu axa Ox nu este simplu):
D = (x, y) ∈ R2 : −1 ≤ x ≤ 2, x2 − 1 ≤ y ≤ x + 1 , deci
ZZ Z 2 Z x+1  Z 2   Z 2
y=x+1
x + 1 − x2 + 1 dx

A= dxdy = dy dx = y|y=x2 −1 dy =
D −1 x2 −1 −1 −1
  x=2
2 3
x x
= 2x + − .
2 3 x=−1

18. Să se calculeze următoarea integrală dublă:


ZZ
y
p dxdy ,
D x2 + y 2

unde D este domeniul dat de: 4x2 + y 2 ≤ 4, x2 + y 2 ≥ 1 şi y ≥ 0.

Rezolvare:

Domeniul D este simplu ı̂n raport cu axa Oy :


n p p o
D = (x, y) ∈ R2 : −1 ≤ x ≤ 1, 1 − x2 ≤ y ≤ 2 1 − x2 .

7
19. Să se calculeze următoarea integrală dublă făcând o schimbare de variabilă convenabilă:
ZZ
x2 + y 2 dxdy ,

D

unde D este domeniul: x2 + y 2 ≥ x şi x2 + y 2 ≤ 2x.

Rezolvare:

Pentru a desena domeniul trebuie mai ı̂ntâi să luam curbele date de egalităţi:
2
x2 + y 2 = x ⇔ (x − 1/2) + y 2 = 1/4,
2
x2 + y 2 = 2x ⇔ (x − 1) + y 2 = 1.
2
Deci domeniul este dat de exteriorul cercului (x − 1/2) + y 2 = 1/4 de centrul C1 (1/2, 0) şi rază
2
1/2, şi de interiorul cercului (x − 1) + y 2 = 1 de centrul C2 (1, 0) şi rază 1.
Observăm că domeniul nu este simplu ı̂n raport cu nici o axă. Putem descompune domeniul ı̂n
două domenii simple ı̂n raport cu axa Oy sau putem folosi coordonatele polare (date de coordo-
natele parametrice ale cercului)
(
x = ρ cos θ,
y = ρ sin θ, ρ ∈ [0, R] , θ ∈ [0, 2π] .

Jacobianul ı̂n cazul trecerii la coordonate polare este dat de:

∂x ∂x


D (x, y) def ∂ρ ∂θ cos θ −ρ sin θ

J (ρ, θ) = = = =ρ
D (ρ, θ) ∂y ∂y sin θ ρ cos θ
∂ρ ∂θ

Trebuie să determinăm domeniile exacte pentru noile variabile polare (ρ, θ). Folosim inegalităţiile
domeniului dat D
2 2
x2 + y 2 ≥ x ⇒ (ρ cos θ) + (ρ sin θ) ≥ ρ cos θ ⇔ ρ2 ≥ ρ cos θ ⇔ ρ ≥ cos θ

respectiv
2 2
x2 + y 2 ≤ 2x ⇒ (ρ cos θ) + (ρ sin θ) ≤ 2ρ cos θ ⇔ ρ2 ≤ 2ρ cos θ ⇔ ρ ≤ 2 cos θ.

În ceea ce priveşte unghiul θ deducem din graficul domeniului D că θ ∈ [−π/2, π/2]. Deci

∆ = (ρ, θ) ∈ R2 : θ ∈ [−π/2, π/2] , ρ ∈ [cos θ, 2 cos θ]




Integrala dublă devine


ZZ   ZZ ZZ
2 2
I= (ρ cos θ) + (ρ sin θ) |J (ρ, θ)| dρdθ = ρ2 ρ dρdθ = ρ3 dρdθ
∆ ∆ ∆
!
π/2 2 cos θ π/2 4 ρ=2 cos θ π/2
Z Z Z Z
ρ 15
= ρ3 dρ dθ = dθ = cos4 θ dθ,
−π/2 cos θ −π/2 4 ρ=cos θ 4 −π/2

pentru care folosim, de două ori, formula

1 + cos (2θ)
cos2 θ = .
2

20. Să se calculeze următoarea integrală dublă făcând o schimbare de variabilă convenabilă:
ZZ
x2 + y 2 dxdy,

D

8
unde D este domeniul mărginit de curba x2 + y 2 = 2.

Rezolvare:

Observ mai ı̂ntâi că D este interiorul cercului (discul) dat de


√ 2
x2 + y 2 ≤ 2 .

Se trece la coordonate polare (ρ, θ) date de


(
x = ρ cos θ,
y = ρ sin θ.

Jacobianul transformării este J = ρ.


Din inegalitatea care dă pe D vom obţine domeniul noii variabile ρ :
h √ i
ρ2 cos2 θ + ρ2 sin2 θ ≤ 2 ⇔ ρ2 ≤ 2 ⇔ ρ ∈ 0, 2 .

Din desen avem că θ ∈ [0, 2π] .


(
0 ≤ θ ≤ 2π,
Deci noul domeniu ∆ : √ .
0 ≤ ρ ≤ 2.
Conform schimbării de variabilă şi a teoremei de reducere vom obţine că
Z 2π "Z √2 # Z 2π Z √
2
2 2 2 2
ρ3 dρ .

I= ρ cos θ + ρ sin θ ρ dρ dθ = dθ ·
0 0 0 0

21. Să se calculeze următoarea integrală dublă făcând o schimbare de variabilă convenabilă:
ZZ
x2 + y 2 dxdy ,

D
√ √
unde D este domeniul din primul cadran limitat de x2 + y 2 = a2 , y = x 3 şi x = y 3.

Rezolvare:

Observ mai ı̂ntâi că D este interiorul cercului (discul) dat de

x2 + y 2 ≤ a2
1 √
astfel ı̂ncât √ x ≤ y ≤ x 3 .
3
Se trece la coordonate polare (ρ, θ) şi, din inegalitatea care dă pe D, vom obţine domeniul pentru ρ :

ρ2 cos2 θ + ρ2 sin2 θ ≤ a2 ⇔ ρ2 ≤ a 2 ⇔ ρ ∈ [0, a] .

Din desen avem că θ ∈ [θ1 , θ2 ] , unde θ1 , θ2 sunt unghiurile făcute de dreptele y = √1 x şi respectiv
√ √ 3
y = x 3 cu axa Ox. Deci tg (θ1 ) = √13 şi tg (θ2 ) = 3 .

 0 ≤ ρ ≤ a,
Obţinem noul domeniu ∆ :
 π ≤θ≤ π.
6 3
Conform schimbării de variabilă şi a teoremei de reducere vom obţine că
Z π/3 Z a  Z π/3 Z a
ρ2 cos2 θ + ρ2 sin2 θ ρ dρ dθ = ρ3 dρ .

I= dθ ·
π/6 0 π/6 0

9
22. Să se calculeze următoarea integrală dublă făcând o schimbare de variabilă convenabilă:
ZZ p
sin x2 + y 2 dxdy ,
D

x2 + y 2 ≤ 4π 2 ,
(
unde D este domeniul dat de
x2 + y 2 ≥ π 2 .
Rezolvare:

Se trece la coordonate polare (ρ, θ) şi, din inegalitatea care dă pe D, vom obţine domeniul pentru ρ :

π 2 ≤ ρ2 cos2 θ + ρ2 sin2 θ ≤ 4π 2 ⇔ π 2 ≤ ρ2 ≤ 4π 2 ⇔ ρ ∈ [π, 2π] .

Din desen avem că θ ∈ [0, 2π] .


(
π ≤ ρ ≤ 2π,
Obţinem noul domeniu ∆ : .
0 ≤ θ ≤ 2π .
Conform schimbării de variabilă şi a teoremei de reducere vom obţine că
Z 2π Z 2π  Z π/3 Z a
sin ρ2 cos2 θ + ρ2 sin2 θ ρ dρ dθ = ρ sin ρ2 dρ .
 
I= dθ ·
0 π π/6 0

23. Să se calculeze următoarea integrală dublă făcând o schimbare de variabilă convenabilă:

y2
ZZ
2
dxdy,
D x

unde D : 1 ≤ x2 + y 2 ≤ 2x.

Rezolvare:

Observ mai ı̂ntâi că D este exteriorul cercului

x2 + y 2 ≥ 1

intersectat cu interiorul cercului


2
(x − 1) + y 2 ≤ 1.
Se trece la coordonate polare (ρ, θ) şi, din inegalitatea care dă pe D, vom obţine domeniul noii
variabile ρ :

1 ≤ ρ2 cos2 θ + ρ2 sin2 θ ≤ 2ρ cos θ ⇔ 1 ≤ ρ2 ≤ 2ρ cos θ ⇔ ρ ∈ [1, 2 cos θ] .

Observăm că trebuie impusă condiţia


1 h π πi
2 cos θ ≥ 1 ⇔ cos θ ≥ ⇔ θ∈ − , .
2 3 3
Pe de √
altă parte, din desen avem
√ că θ ∈ [−θ1 , θ2 ] , unde θ√1 , θ2 sunt unghiurile făcute de dreptele
y = − 3 x şi respectiv y = x 3 cu axa Ox. Deci tg (θ2 ) = 3 .
 π
 − ≤θ≤ π,
Deci noul domeniu ∆ : 3 3 .
1 ≤ ρ ≤ 2 cos θ .

Conform schimbării de variabilă şi a teoremei de reducere vom obţine că


Z π/3 "Z 2 cos θ #
1 π/3 2 1 π/3 
Z Z
2
tg (θ) 4 cos2 θ − 1 dθ = 4 sin2 θ − tg2 (θ) dθ .
  
I= tg (θ) ρ dρ dθ =
−π/3 1 2 −π/3 2 −π/3

Deci trebuie calculate primitiva Z


sin2 θ dθ

10
1+cos(2θ)
folosind formula sin2 θ = 2 , precum şi primitiva
Z
tg2 (θ) dθ

folosind subtituţia tg (θ) = t şi formulele sin θ = √ t şi cos θ = √ 1 .


t2 +1 t2 +1

24. Făcând o schimbare de variabilă convenabilă să se calculeze aria domeniului D din primul cadran
mărginit de curbele
( (
xy = p, y = ax,
şi 0 < p < q, 0 < a < b.
xy = q y = bx,
Rezolvare:

Trebuie calculată aria dată de integrala


ZZ
A= dxdy.
D

Vom face schimbarea de variabilă (


xy = u,
y/x = v,
deci echivalent obţinem
p
u/v = u1/2 v −1/2 ,
(
x=

y= uv = u1/2 v 1/2 , u ∈ [p, q] , v ∈ [a, b] .
1
Jacobianul este dat de J = iar domeniul ∆ este deci dreptunghiular
2v
∆ = (u, v) ∈ R2 : u ∈ [p, q] , v ∈ [a, b] = [p, q] × [a, b] .


25. Făcând o schimbare de variabilă convenabilă să se calculeze aria domeniului D mărginit de curbele
( 2 ( 2
y = px, x = ay,
şi 0 < p < q, 0 < a < b.
y 2 = qx x2 = by,

26. Folosind coordonatele polare generalizate să se calculeze aria domeniului D mărginit de elipsa
x2 y2
+ = 1.
a2 b2
Rezolvare:

Trebuie calculată aria dată de integrala


ZZ
A= dxdy.
D

Se trece la coordonate polare generalizate (ρ, θ) date de


(
x = aρ cos θ,
y = bρ sin θ.
Jacobianul transformării este J = abρ.
Din inegalitatea care dă pe D vom obţine domeniul noii variabile ρ :
ρ2 cos2 θ + ρ2 sin2 θ ≤ 1 ⇔ ρ2 ≤ 1 ⇔ ρ ∈ [0, 1] .
Din desen avem că θ ∈ [0, 2π] .
(
0 ≤ θ ≤ 2π,
Deci noul domeniu ∆ : .
0 ≤ ρ ≤ 1.

11
27. Făcând o schimbare de variabilă convenabilă să se calculeze integrala

ZZ
arcsin x + y dxdy ,
D

unde D este domeniul mărginit de x + y = 0, x + y = 1 şi y = −1, y = 1.

Rezolvare:

Vom face schimbarea de variabilă (


x + y = u,
y=v
deci echivalent obţinem (
x = u − v,
y = v, u ∈ [0, 1] , v ∈ [−1, 1] .
Jacobianul este dat de J = 1 iar domeniul ∆ este deci dreptunghiular

∆ = (u, v) ∈ R2 : u ∈ [0, 1] , v ∈ [−1, 1] = [0, 1] × [−1, 1] .




28. Făcând o schimbare de variabilă convenabilă să se calculeze integrala


ZZ
4 2
(x + y) (x − y) dxdy ,
D

unde D este domeniul mărginit de x + y = −1, x + y = 1 şi x − y = −3, x − y = −1.

Rezolvare:

Vom face schimbarea de variabilă (


x + y = u,
x−y =v
deci echivalent obţinem
 1
 x = (u + v) ,

2
 1
 y = (u − v) , u ∈ [−1, 1] , v ∈ [−3, −1] .
2

29. Să se transforme următoarea integrală curbilinie folosind formula lui Green:
I p   p 
I= x2 + y 2 dx + y xy + ln x + x2 + y 2 dy ,
(γ)

unde (γ) este curba ı̂nchisă x2 + y 2 = 1.

Rezolvare:

Formula lui Green: dacă curba ı̂nchisă (γ) mărgineşte domeniul D atunci are loc
Z ZZ  
∂Q ∂P
I= P (x, y) dx + Q (x, y) dy = − dxdy .
(γ) D ∂x ∂y

30. Să se aplice formula lui Green pentru calculul integralei curbilinii
I
2
2 x2 + y 2 dx + (x + y) dy ,

(γ)

unde (γ) este triunghiul dat de intersecţia dreptelor x = 1, y = x şi y = 4 − x.

12
31. Să se aplice formula lui Green pentru calculul integralei curbilinii
I
2
(x + y) dx − x2 + y 2 dy ,

(γ)

unde (γ) este triunghiul cu vârfurile A (1, 0) , B (0, 1) şi O (0, 0) .


32. Să se găsească volumul unui corp mărginit de planul xOy şi de planele x = 0, x = a şi y = 0, y = b
2 2
şi superior de suprafaţa 2z = xp + yq .

Rezolvare:

Avem ZZ
I= f (x, y) dxdy,
D

unde z = f (x, y) şi D = [0, a] × [0, b]. Deci


!
a b
x2 y2 x2 y2
ZZ   Z Z  
I= + dxdy = + dy dx
(D) 2p 2q 0 0 2p 2q
a  y=b ! Z a 2
x2 1 y 3 b3
Z  
bx
= ·y+ · dx = + dx
0 2p 2q 3 y=0 0 2p 6q
 x=a
b x3 b3 a3 b ab3


= · + · x = + .
2p 3 6q x=0 6p 6q

33. Să se calculeze următoarea integrală dublă făcând o schimbare de variabilă convenabilă:
ZZ
dxdy
,
2 2 2
D (1 + x + y )

unde D este domeniul: x2 + y 2 ≤ 2y.

Rezolvare:

Observ mai ı̂ntâi că D este interiorul cercului


2
x2 + (y − 1) = 1.

Se trece
( la coordonate polare (ρ, θ) şi, din inegalitatea care dă pe D, vom obţine noul domeniu
0≤θ≤π
∆: . Conform schimbării de variabilă şi a reducerii vom obţine că
0 ≤ ρ ≤ 2 sin θ

π 1 π
Z
1
I= − dθ
2 2 0 1 + 4 sin2 θ

care se va rezolva cu substituţia


tg (θ) = t
şi cu formulele trigonometrice
t 1
sin (θ) = √ , cos (θ) = √ .
1 + t2 1 + t2

34. Făcând o schimbare de variabilă convenabilă să se calculeze integrala


ZZ
dxdy
,
2 2 2 2 2
D (1 + b x + a y )

x2 y2
unde D este domeniul: a2 + b2 ≤ 1.

13
Rezolvare:
2 2
Observ mai ı̂ntâi că D este dat de interiorul
( elipsei xa2 + yb2 = 1. Se trece la coordonate polare
x = aρ cos θ
generalizate (ρ, θ) date de ecuaţiile şi, din inegalitatea care dă pe D, vom obţine
( y = bρ sin θ
0 ≤ θ ≤ 2π
noul domeniu ∆ : . Conform schimbării de variabilă şi a reducerii vom obţine că
0≤ρ≤1
Z 1
abρ
I = 2π 2 dρ .
0 (1 + a2 b2 ρ2 )

35. Să se calculeze


ZZ s
1 − x2 − y 2
dxdy ,
D 1 + x2 + y 2
unde D este sfertul din primul cadran al discului x2 + y 2 ≤ 1.
36. Să se calculeze ZZ
xy
p dxdy ,
D x2 + y 2
x2 y2
unde D este sfertul din primul cadran al interiorului elipsei a2 + b2 ≤ 1.

Rezolvare:

Se trece la coordonate polare generalizate (ρ, θ) date de


(
x = aρ cos θ,
y = bρ sin θ.

Jacobianul transformării este J = abρ.


Din inegalitatea care dă pe D vom obţine domeniul noii variabile ρ :

ρ2 cos2 θ + ρ2 sin2 θ ≤ 1 ⇔ ρ2 ≤ 1 ⇔ ρ ∈ [0, 1] .

Din desen avem că θ ∈ [0, π/2] .


(
0 ≤ θ ≤ π/2,
Deci noul domeniu ∆ : .
0 ≤ ρ ≤ 1.
Se obţine integrala
 
Z 1 Z π/2 Z 1 Z π/2
aρ cos θ · bρ sin θ sin θ cos θ
I=  q ρ dθ dρ = ab ρ2 dρ · p dθ .
0 0 2
(aρ cos θ) + (bρ sin θ)
2 0 0 a cos2 θ + b2 sin2 θ
2

Ultima integrală se rezolvă cu substituţia

t = sin2 θ ⇒ dt = 2 sin θ cos θdθ.

37. Să se calculeze masa şi coordonatele centrului de greutate ale plăcii D = {x + y ≤ 1, x ≥ 0, y ≥ 0}
dacă densitatea este µ (x, y) = xy.

Rezolvare:

Masa m a plăcii D este dată de ZZ


m= µ (x, y) dxdy
D

14
iar coordonatele
Z Z centrului de greutate G (xG , yG ) sunt date de

1
 xG = m

 xµ (x, y) dxdy ,
D
ZZ .
 yG = 1


m yµ (x, y) dxdy .
D

38. Să secalculeze masa şi coordonatele centrului


de greutate ale plăcii
D = x2 + y 2 ≤ a2 , x2 + y 2 ≥ ax, y ≥ 0 dacă densitatea este µ (x, y) = 1.

Rezolvare:
2 2
Observ că D este dat de interiorul cercului x2 + y 2 = a2 şi de exteriorul cercului x − a2 + y 2 = a4 .
Se trece
( la coordonate polare (ρ, θ) şi, din inegalitatea care dă pe D, vom obţine noul domeniu
0 ≤ θ ≤ π,
∆:
a cos θ ≤ ρ ≤ a.

39. Să se calculeze volumul corpului mărginit de planele x = 0, y = 0, z = 0, de cilindrul x2 + y 2 = R2


şi superior de către paraboloidul hiperbolic 5z = xy.

Rezolvare:

Volumul este dat de ZZ


V= f (x, y) dxdy ,
D
unde z = f (x, y) este ecuaţia suprafeţei ce mărgineşte superior volumul, iar (x, y) ∈ D unde D este
proiecţia suprafeţei pe planul XOY . În cazul nostru D este sfertul de disc x2 + y 2 ≤ R2 , x, y ≥ 0.
40. Să se calculeze următoarea integrală dublă pe un domeniu dreptunghiular
ZZ
cos (x + y) dxdy ,
D
h πi h πi
unde D este dreptunghiul D = 0, × 0, .
4 2
41. Să se calculeze următoarea integrală dublă pe un domeniu dreptunghiular
ZZ
x cos (xy) dxdy ,
D

unde D este dreptunghiul D = [1, 2] × [0, π] .


42. Să se calculeze următoarea integrală dublă
ZZ
(x + 2y) dxdy ,
D

unde D este domeniul mărginit de curbele y = 2x şi y = 3 − x2 .


43. Să se calculeze următoarea integrală dublă
ZZ
(2x + 5y) dxdy ,
D

unde D este domeniul mărginit de curbele y = 0, y = 4, x = 4 şi y = x2 .


44. Să se calculeze volumul cilindroidului
 
3
C = (x, y, z) ∈ R3 : x2 + y 2 ≤ 25, y ≤ x, z ≤ 25 + xy
4
(este corpul mărginit superior de către paraboloidul hiperbolic z = 25 + xy şi cu proiecţia pe planul
3
x0y dată de porţiunea de disc x2 + y 2 ≤ 25, y ≤ x ).
4

15
45. Să se calculeze următoarea integrală dublă
ZZ
(1 + x) dxdy ,
D

unde  
2 1
D= (x, y) ∈ R : y ≥ |x| , y ≤ x + 2 .
2

46. Să se calculeze următoarea integrală dublă


ZZ
y
dxdy ,
D 1 + x
25
unde D este domeniul mărginit de curbele x2 + y 2 = 25 şi x2 + y 2 − x = 0.
4
47. Să se calculeze următoarea integrală dublă
ZZ
1
2 + y2
dxdy ,
D 1 + x

unde n √ o
D = (x, y) ∈ R2 : 0 ≤ y ≤ 3x, 1 ≤ x2 + y 2 ≤ 4 .
ZZ
48. Să se transforme integrala dublă f (x, y) dxdy ı̂n integrale iterate, unde D este domeniul mărginit
D
de cercurile x2 + y 2 = 2x, x2 + y 2 = 4x.

Rezolvare:

În cazul nostru, primul cerc este


2
(C1 ) : x2 + y 2 = 2x x2 − 2x + y 2 = 0 (x − 1) + y 2 = 1,

⇔ ⇔

adică are centrul ı̂n C1 (1, 0) şi raza r1 = 1.


Al doilea cerc
2
(C2 ) : x2 + y 2 = 4x x2 − 4x + y 2 = 0 (x − 2) + y 2 = 4

⇔ ⇔

este cu centrul ı̂n C2 (2, 0) şi de rază r2 = 2.


Ducem prin x = 2 o paralelă la axa Oy şi astfel domeniul D se va ı̂mpărţi ı̂n trei subdomenii:
D = D1 ∪ D2 ∪ D3 şi fiecare subdomeniu Di este simplu ı̂n raport cu Oy. Are loc
ZZ ZZ ZZ
I= f (x, y) dxdy + f (x, y) dxdy + f (x, y) dxdy
(D1 ) (D2 ) (D3 )

Trebuie explicitat fiecare subdomeniu. În acest sens vom scrie ecuaţiile explicite ale
cercurilor: p
(C1 ) : x2 + y 2 = 2x ⇔ y 2 = 2x − x2 ⇔ y = ± 2x − x2
p
(C2 ) : x2 + y 2 = 4x ⇔ y 2 = 4x − x2 ⇔ y = ± 4x − x2
deci obţinem
 √ √
D1 = (x, y) ∈ R2 : 0 ≤ x ≤ 2, 2x − x2 ≤ y ≤ 4x − x2
 √ √
D2 = (x, y) ∈ R2 : 0 ≤ x ≤ 2, − 4x − x2 ≤ y ≤ − 2x − x2
 √ √
D3 = (x, y) ∈ R2 : 2 ≤ x ≤ 4, − 4x − x2 ≤ y ≤ 4x − x2

16
În acest caz integralele sunt date de
√ !
ZZ Z 2 Z 4x−x2
I1 = f (x, y) dxdy = √ f (x, y) dy dx,
(D1 ) 0 2x−x2
√ !
ZZ Z 2 Z − 2x−x2
I2 = f (x, y) dxdy = √ f (x, y) dy dx,
(D2 ) 0 − 4x−x2
ZZ Z 4 Z √4x−x2 !
I3 = f (x, y) dxdy = √ f (x, y) dy dx.
(D3 ) 2 − 4x−x2

17
Facultatea de Matematică
Calcul Integral şi Aplicaţii, Semestrul I
Lector dr. Lucian MATICIUC

Seminariile 10 - 12
Capitolul IV. Integrala dublă

ANEXĂ

Conicele sunt date de ecuaţii algebrice de gradul 2, cu două necunoscute. Ecuaţiile lor reduse sunt
următoarele.
1. Cercul cu centrul ı̂n C (a, b) şi de rază R are ecuaţia implicită
2 2
(x − a) + (y − b) = R2 , a, b ∈ R, R > 0.
Acesta are ecuaţiile explicite date de
q
2
y =b± R2 − (x − a)
şi ecuaţiile parametrice date de
(
x = R cos θ,
y = R sin θ, θ ∈ [0, 2π] .

2. Elipsa cu centrul ı̂n origine şi de semiaxe a şi b are ecuaţia implicită
x2 y2
+ = 1, a, b > 0.
a2 b2
Aceasta are ecuaţiile explicite date de
bp 2
y=± a − x2
a
şi ecuaţiile parametrice date de
(
x = aR cos θ,
y = bR sin θ, θ ∈ [0, 2π] .

3. Parabola cu axa Oy drept axă de simetrie are ecuaţia explicită


y = ax2 + bx + c, a, b, c ∈ R.

4. Parabola cu axa Ox drept axă de simetrie are ecuaţia implicită


y 2 = 2px, p ∈ R.
Aceasta are ecuaţiile explicite date de p
y = ± 2px .

5. Hiperbola are ecuaţia implicită


x2 y2
− = 1, a, b > 0.
a2 b2
Aceasta are ecuaţiile explicite date de
bp 2
y=± x − a2 .
a

18
6. Hiperbola echilateră cu axele de coordonate drept asimptote are ecuaţia implicită

xy = c, c ∈ R.

deci ecuaţia explicită este


c
y= .
x

19
Facultatea de Matematică
Calcul Integral şi Aplicaţii, Semestrul I
Lector dr. Lucian MATICIUC

413-14
Seminariile 13 - 15

Capitolul V. Integrala triplă

1. Să se calculeze ZZZ


dxdydz
2 ,
V (x + y + z)
unde V = [1, 3] × [0, 1] × [0, 2] .

Rezolvare:
! !
Z 3 Z 1 Z 2 Z 3 Z 1 Z 2  
1 −2
I = 2 dz dy dx = (x + y + z) dz dy dx
1 (x + y + z)
0 0 1 0 0
   
Z 3 Z 1 −1 z=2 Z 3 Z 1   
(x + y + z) −1 −1
= dy dx = − (2 + x + y) − (x + y) dy dx
   
−1

1 0 1 0
z=0

2. Să se calculeze ZZZ


dxdydz
3 ,
V (1 + x + y + z)
unde V este mărginit de planele x = 0, y = 0, z = 0 şi de planul x + y + z = 1.

Rezolvare:
(
(x, y) ∈ D
Explicitarea lui V : unde domeniul D este dat de placa triunghiulară D :
( 0≤z ≤1−x−y
0 ≤ x ≤ 1,
0 ≤ y ≤ 1 − x.
3. Să se calculeze ZZZ
ydxdydz ,
V

unde V este tetraedrul din primul octant mărginit de planele de coordonate x = 0, y = 0, z = 0 şi
de planul x + y + z = 2.

Rezolvare:
(
(x, y) ∈ D
Explicitarea lui V : unde domeniul D este proiecţia corpului V pe planul
0≤z ≤2−x−
(y
0≤x≤2
xOy, deci este placa triunghiulară D : .
0≤y ≤2−x
Z Z Z 2−x−y  Z 2 Z 2−y Z 2−x−y  
I= ydz dxdy = ydz dy dx.
D 0 0 0 0

1
4. Să se calculeze ZZZ
zdxdydz ,
V
x2 y2 z2
unde V este jumătatea superioară a elipsoidului a2 + b2 + c2 = 1.

Rezolvare:

 (x, y) ∈ D
Explicitarea lui V : q unde domeniul D este dat de interiorul de elipsă
 0≤z ≤c 1− x2 y2
a2 − b2
x2 y2
D: a2 + b2 ≤ 1.
5. Să se calculeze ZZZ
zdxdydz ,
V
h2

unde V este mărginit de suprafaţa conică z 2 = R2 x2 + y 2 , 0 ≤ z ≤ h.

Rezolvare:
(
(x, y) ∈ D
Explicitarea lui V : p unde domeniul D este discul D : x2 + y 2 ≤ R2 .
h 2 + y2 ≤ z ≤ h
R x
6. Să se calculeze ZZZ
2
(x + y + z) dxdydz ,
V

x2 + y 2 ≤ 2az,
(
a > 0,
unde V este dat de V :
x2 + y 2 + z 2 ≤ 3a2 .

Rezolvare:

Mai ı̂ntâi determin intersecţia celor două corpuri. Deci x2 + y 2 = 2az şi introduc ı̂n a doua ecuaţie:
2az + z 2 = 3a2 ⇔ (z − a) (z + 3a) = 0 şi deoarece z ≥ 0 aleg soluţia z = a. deci obţin x2 +
√ 2
y 2 = a 2 care este ecuaţia cercului ı̂n care se ı̂ntâlneşte paraboloidul cu sfera. Explicitarea lui
(x, y) ∈ D
(
√ 2
V : unde domeniul D este discul D : x2 + y 2 ≤ a 2 .
x2 +y 2
p
2a ≤ z ≤ 3a2 − x2 − y 2
7. Să se calculeze ZZZ
x2 + y 2 zdxdydz ,

V

unde V este mărginit de paraboloidul z = x2 + y 2 şi de sfera x2 + y 2 + z 2 = 6 şi conţine o parte din
porţiunea nenegativă a axei Oz.

Rezolvare:
(
(x, y) ∈ D
Explicitarea lui V : p unde domeniul D este proiecţia corpului V pe
x2 + y 2 ≤ z ≤ 6 − x2 − y 2
planul xOy (se determină mai ı̂ntâi sferei x2 + y 2 + z 2 = 6 cu paraboloidul z = x2 + y 2 ), deci este
discul D : x2 + y 2 ≤ 2.

ZZ Z √6−x2 −y2 !
2 2

I= x +y zdz dxdy.
D x2 +y 2

2
8. Să se calculeze ZZZ
x2 + y 2 + z 2 dxdydz ,

V

unde V este bila ı̂nchisă de rază R cu centrul ı̂n origine.

Rezolvare:

Pentru a calcula integrala triplă vom folosi coordonate sferice cu ρ ∈ [0, R] , ϕ ∈ [0, π] , θ ∈ [0, 2π] ,
J = −ρ2 sin ϕ. Deci
Z R Z π Z 2π    
2 2 2
V= (ρ cos θ sin ϕ) + (ρ sin θ sin ϕ) + (ρ cos ϕ) |J| dθ dϕ dρ.
0 0 0

9. Să se calculeze ZZZ


dxdydz
p ,
V x2 + y2 + z2
2 2 2 2 2 2
unde V este situat
p ı̂n semispaţiul superior şi este delimitat de sferele x +y +z = 1, x +y +z = 9
şi de conul z = x2 + y 2 .

Rezolvare:

Pentru a calcula integrala triplă vom folosi coordonate sferice cu ρ ∈ [1, 3] , ϕ ∈ [0, π/4] , θ ∈ [0, 2π],
J = −ρ2 sin ϕ. Deci
   
Z 3 Z π/4 Z 2π
1
V=   q |J| dθ dϕ dρ.
1 0 0 2 2 2
(ρ cos θ sin ϕ) + (ρ sin θ sin ϕ) + (ρ cos ϕ)

10. Să se calculeze ZZZ


x2 + y 2 dxdydz

V

unde V este coroana circulară mărginită de cilindrii circulari x2 + y 2 = 4, x2 + y 2 = 9 şi de planele


z = 0 şi de z = 1.

Rezolvare:

Pentru a calcula integrala triplă vom folosi coordonatele cilindrice unde ρ ∈ [2, 3] , θ ∈ [0, 2π] , z ∈
[0, 1] iar iacobianul este, făcând calculele, J = ρ. Deci
Z 3 Z 2π Z 1    
2 2
V= (ρ cos θ) + (ρ sin θ) |J| dz dθ dρ.
2 0 0

11. Să se calculeze


x2 y2 z2
ZZZ  
2
+ 2 + 2 dxdydz ,
V a b c
x2 y2 z2
unde V este dat de 1 ≤ 2
+ 2 + 2 ≤ 4.
a b c
Rezolvare:

Pentru a calcula integrala vom folosi coordonate sferice generalizate cu ρ ∈ [1, 2] , θ ∈ [0, π] , θ ∈
[0, 2π], J = −abcρ2 sin ϕ. Deci
! ! !
Z 2 Z π Z 2π 2 2 2
(aρ cos θ sin ϕ) (bρ sin θ sin ϕ) (cρ cos ϕ)
V= + + |J| dθ dϕ dρ.
1 0 0 a2 b2 c2

3
12. Să se calculeze cu ajutorul formulei lui Gauss-Ostrogradski următoarea integrală de suprafaţă de
specia a doua ZZ
x2 dydz + y 2 dzdx + z 2 dxdy ,
S
x2 y2 z2
unde (S) este faţa exterioară a elipsoidului a2 + b2 + c2 = 1.

Rezolvare:

Observ că P = x2 , Q = y 2 , R = z 2 ; pentru a calcula integrala triplă pe interiorul unui elipsoid


folosim coordonatele sferice generalizate.

13. Să se calculeze cu ajutorul formulei lui Gauss-Ostrogradski următoarea integrală de suprafaţă de
specia a doua ZZ
x3 y 2 dydz + x2 y 3 dzdx + 3zdxdy ,
S

unde S este faţa exterioară a domeniului V mărginit de paraboloizii z = x2 + y 2 , z = 6 − x2 − y 2 .

Rezolvare:

Conform formulei lui Gauss-Ostrogradski


ZZZ
3x2 y 2 + 3x2 y 2 + 3z dxdydz

I=
V
(
(x, y) ∈ D
unde V : iar domeniul D este proiecţia corpului V pe planul xOy (se
x2 + y 2 ≤ z ≤ 6 − x2 − y 2
determină mai ı̂ntâi intersecţia celor doi paraboloizi), deci este discul D : x2 + y 2 ≤ 3.

6−x2 −y 2
ZZ Z !
2 2 2 2

I= 3x y + 3x y + 3 dz dxdy.
D x2 +y 2

Pentru calculul integralei duble folosim coordonate polare.


z2
14. Să se determine volumul corpului dat de ≥ x2 + y 2 , 0 ≤ z ≤ h.
h2
Rezolvare:

Volumul lui V este dat de ZZZ


V= dxdydz
V
(
(x, y) ∈ D
Explicitarea lui V : p unde domeniul D este proiecţia corpului V pe planul
h x2 + y 2 ≤ z ≤ h
2
xOy (se determină mai ı̂ntâi intersecţia planului z = h > 0 cu paraboloidul hz 2 = x2 + y 2 ), deci este
discul D : x2 + y 2 ≤ 1.
!
ZZ Z h
I= √ dz dxdy.
D h x2 +y 2

Pentru calculul integralei duble folosim coordonate polare.


15. Să se determine volumul corpului situat ı̂n semispaţiul superior z ≥ 0 şi mărginit de suprafeţele
x2 + y 2 + z 2 = a2 , x2 + y 2 + z 2 = b2 , x2 + y 2 = z 2 , a < b.

Rezolvare:

4
Pentru a calcula integrala triplă vom folosi coordonate sferice cu ρ ∈ [a, b] , ϕ ∈ [0, π/4] , θ ∈ [0, 2π],
J = −ρ2 sin ϕ. Deci
Z b Z π/4 Z 2π  !
V= |J| dθ dϕ dρ.
a 0 0

16. Să se calculeze volumul unui corp mărginit de suprafeţele z = x2 + y 2 − 1 şi z = 2 − x2 − y 2 .


17. Să se calculeze volumul unui corp mărginit de suprafaţa
2
(i) x2 + y 2 + z 2 = a3 z cu x, y, z ≥ 0.
2
x2 y2 z2 x2 y

(ii) 2
+ 2 + 2 = cu x, y, z ≥ 0.
a b c h3

Rezolvare:
ZZZ
(i) Pentru a calcula volumul V = dxdydz folosesc coordonatele sferice. Suntem ı̂n primul
V
octant (x, y, z ≥ 0) deci ϕ ∈ [0, π/2] , θ ∈ [0, π/2]. Pentru a determina ρ folosim inegalitatea care-l dă
pe V : x2 + y 2 + z 2 ≤ a3 z. Deci
 2
2 2 2
(ρ cos θ sin ϕ) + (ρ sin θ sin ϕ) + (ρ cos ϕ) ≤ a3 ρ cos ϕ
2 √
⇔ ρ2 ≤ a3 ρ cos ϕ ⇔ 0 ≤ ρ ≤ a 3 cos ϕ .
 √

 0 ≤ ρ ≤ a 3 cos ϕ

Deci ∆ : 0 ≤ ϕ ≤ π/2 iar J = −ρ2 sin ϕ.


0 ≤ θ ≤ π/2

ZZZ
(ii) Pentru a calcula volumul V = dxdydz folosesc coordonatele sferice generalizate. Suntem
V
ı̂n primul octant (x, y, z ≥ 0) deci ϕ ∈ [0, π/2] , θ ∈ [0, π/2]. Pentru a determina ρ folosim inegalitatea
 2 2 2
2 2
care-l dă pe V : xa2 + yb2 + zc2 ≤ xh3y . Deci
 2
(aρ cos θ sin ϕ)2 (bρ sin θ sin ϕ)2 (cρ cos ϕ)2 (aρ cos θ sin ϕ)2 (bρ sin θ sin ϕ)
a2 + b2 + c2 ≤ h3

a2 b
⇔ 0 ≤ ρ4 ≤ h3 ρ3 cos2 θ sin3 ϕ sin θ.

a2 b


 0≤ρ≤ h3 cos2 θ sin3 ϕ sin θ

Deci ∆ : 0 ≤ ϕ ≤ π/2 iar J = −abcρ2 sin ϕ.


0 ≤ θ ≤ π/2

18. Să se calculeze ZZZ


1
3/2
dxdydz ,
V (a2 + x2 + y 2 − z)
unde V este dat de az ≤ x2 + y 2 ≤ a2 cu z ≥ 0.
19. Să se calculeze 3/2
y2 z2
ZZZ 
2
1−x − − dxdydz ,
V 9 4
y2 z2
unde V este dat de x2 + + ≤ 1 cu y ≥ 0, z ≥ 0.
9 4

5
20. Să se determine masa şi centrul de greutate al interiorului de sferă x2 +y 2 +z 2 ≤ 2az dacă densitatea
µ ı̂ntr-un punct este invers proporţională cu distanţa de la acel punct la originea coordonatelor, i.e.
µ (x, y, z) = √ 2 k 2 2 .
x +y +z

Rezolvare:

Pentru a calcula integralele triple vom trece la coordonate sferice. Observăm mai ı̂ntâi că sfera este
2
x2 + y 2 + z 2 = 2az ⇔ x2 + y 2 + z − 2az = 0 ⇔ x2 + y 2 + (z − a) = a2 ,

deci are centrul ı̂n punctul C (0, 0, a) şi raza a deci este situată deasupra planului z = 0 (planul
xOy). Deci ϕ ∈ [0, π/2] , θ ∈ [0, 2π]. Pentru a determina ρ folosim inegalitatea care-l dă pe V :
2 2 2
(ρ cos θ sin ϕ) + (ρ sin θ sin ϕ) + (ρ cos ϕ) ≤ 2aρ cos ϕ
⇔ ρ2 ≤ 2aρ cos ϕ
⇔ 0 ≤ ρ ≤ 2a cos ϕ.


 0 ≤ ρ ≤ 2a cos ϕ

Deci ∆ : 0 ≤ ϕ ≤ π/2 iar J = −ρ2 sin ϕ.


0 ≤ θ ≤ 2π

Mai ı̂ntâi trebuie calculată masa dată de formula (2):


! !
ZZZ Z 2π Z π/2 Z 2a cos ϕ
1
m= µ (x, y, z) dxdydz = k p |J| dρ dϕ dθ
V 0 0 0 ρ2
Z 2π  Z π/2 Z 2a cos ϕ ! Z 2π  Z π/2 !
ρ2 ρ=2a cos ϕ

=k dθ ρ sin ϕ dρ dϕ = k dθ sin ϕ dϕ
0 0 0 0 0 2 ρ=0
Z π/2
cos3 ϕ ϕ=π/2 4
= 4a2 kπ sin ϕ cos2 ϕ dϕ = 4a2 kπ = a2 kπ .
−3 ϕ=0 3

0

Coordonatele centrului de greutate G (xG , yG , zG ) sunt date de formulele (3).


21. Să se determine momentul de inerţie ı̂n raport cu planul yOz al solidului omogen, de densitate
unitate,
q având configuraţia domeniului V mărginit de planul z = c > 0 şi de conul eliptic z =
x2 y2
c a2 + b2 .

Rezolvare:
ZZZ
Momentul de inerţie ı̂n raport cu planul yOz este Iyz = x2 dxdydz. Explicitarea lui V :
q V
2 2
c xa2 + yb2 ≤ z ≤ c, cu (x, y) ∈ D, unde domeniul D este proiecţia corpului V pe planul xOy
(se determină mai ı̂ntâi intersecţia planului z = c > 0 cu conul eliptic), deci este discul eliptic
2 2
D : xa2 + yb2 ≤ 1. !
ZZ Z c
I= q
2 2
x2 dz dxdy
D c x
a2
+ yb2

Pentru calculul integralei duble folosim coordonate polare generalizate.


22. Să se determine coordonatele centrului de greutate al unui solid omogen mărginit de pânza unui
con circular drept, având unghiul de la vârf egal cu 2α şi de o sferă de rază R cu centrul ı̂n vârful
conului, astfel ı̂ncât z ∈ [0, R].

Rezolvare:

Dacă un solid este omogen, atunci centrul lui de greutate se găseşte pe axa lui de simetrie (ı̂n caz că
există), deci ı̂n cazul nostru este pe Oz. Astfel obţinem

xG = 0, yG = 0.

6
Prin definiţie ZZZ
1
zG = zdxdydz,
m V

unde m este masa lui V , dată de


ZZZ ZZZ
m= µ (x, y, z) dxdydz = c dxdydz = c · V,
V V

deoarece solidul este omogen, deci densitatea µ (x, y, z) este contantă.


Pentru a calcula cele 2 integrale triple vom folosi coordonate sferice cu ρ ∈ [0, R] , ϕ ∈ [0, α] , θ ∈
[0, 2π], J = −ρ2 sin ϕ. Deci
Z R Z α Z 2π  
V= |J| dθ dϕ dρ.
0 0 0

23. Să se determine momentul de inerţie ı̂n raport cu axa Oz a solidului de configuraţie bila de rază a
cu centrul ı̂n origine, şi densitate µ (x, y, z) = x2 + y 2 + z 2 .

Rezolvare:
ZZZ
x2 + y 2 µ (x, y, z) dxdydz.

Momentul de inerţie ı̂n raport cu Oz este Iz =
V
Deci ZZZ
x2 + y 2 x2 + y 2 + z 2 dxdydz
 
Iz =
V

care se va calcula folosind coordonatele sferice cu ρ ∈ [0, a] , ϕ ∈ [0, π] , θ ∈ [0, 2π], J = −ρ2 sin ϕ.
Obţinem
Z a Z π Z 2π  
2 2
Iz = (ρ cos θ sin ϕ) + (ρ sin θ sin ϕ) ·
0 0 0
   
2 2 2
· (ρ cos θ sin ϕ) + (ρ sin θ sin ϕ) + (ρ cos ϕ) |J| dθ dϕ dρ.

7
Facultatea de Matematică
Calcul Integral şi Aplicaţii, Semestrul I
Lector dr. Lucian MATICIUC

Seminariile 13 - 15
13-14

Capitolul V. Integrala triplă

ANEXĂ
(
(x, y) ∈ D
Teorema 1 (de reducere a integralei triple) Dacă, V are explicitarea V : , atunci
g1 (x, y) ≤ z ≤ g2 (x, y)
are loc reducerea !
ZZZ ZZ Z g2 (x,y)
f (x, y, z) dxdydz = f (x, y, z) dz dxdy
V D g1 (x,y)

Teorema
 2 (schimbarea de variabilă ı̂n integrala triplă) Presupunem că V este dat de ecuaţiile parametrice

 x = x (ρ, θ, ϕ)


V : y = y (ρ, θ, ϕ) unde (ρ, θ, ϕ) ∈ ∆.

 z = z (ρ, θ, ϕ)

Vom calcula iacobianul


∂x ∂x ∂x
∂ρ ∂θ ∂ϕ
not D (x, y, z) def

∂y ∂y ∂y
J == == .

∂ρ ∂θ ∂ϕ
D (ρ, θ, ϕ)
∂z ∂z ∂z
∂ρ ∂θ ∂ϕ

Atunci are loc schimbarea de variabilă ı̂n integrala triplă
ZZZ ZZZ
f (x, y, z) dxdydz = f (x (ρ, θ, ϕ) , y (ρ, θ, ϕ) , z (ρ, θ, ϕ)) · |J (ρ, θ, ϕ)| · dρdθdϕ (1)
V ∆

Corolarul 3 (folosirea coordonatelor sferice (coordonatele polare ı̂n spaţiu)) Coordonatele sferice (coordo-
natele polare ı̂n spaţiu) sunt (ρ, θ, ϕ) şi ecuaţiile de legătură cu coordonatele carteziene (x, y, z) sunt


 x = ρ cos θ sin ϕ,

y = ρ sin θ sin ϕ, unde ρ ∈ [0, ∞), ϕ ∈ [0, π] , θ ∈ [0, 2π).


z = ρ cos ϕ,

În funcţie de domeniul V trebuie determinate, mai precis, intervalele de variaţie pentru ρ, ϕ, θ, adică domeniul ∆.
cos θ sin ϕ −ρ sin θ sin ϕ ρ cos θ cos ϕ




Jacobianul este ı̂n acest caz dat de J = sin θ sin ϕ ρ cos θ sin ϕ ρ sin θ cos ϕ (se pot face calcule dez-




cos ϕ 0 −ρ sin ϕ
voltând după a treia linie) şi se va obţine valoarea

J = −ρ2 sin ϕ.

Deci (1) devine


ZZZ ZZZ
f (x, y, z) dxdydz = f (x (ρ, θ, ϕ) , y (ρ, θ, ϕ) , z (ρ, θ, ϕ)) ρ2 sin ϕ dρdθdϕ.
V ∆

8
Corolarul 4 (folosirea coordonatelor sferice generalizate (coordonatele polare generalizate ı̂n spaţiu))
Coordonatele sferice generalizate (coordonatele polare generalizate ı̂n spaţiu) sunt (ρ, θ, ϕ) şi ecuaţiile de legătură
cu coordonatele carteziene (x, y, z) sunt


 x = aρ cos θ sin ϕ,

y = bρ sin θ sin ϕ, unde ρ ∈ [0, ∞), ϕ ∈ [0, π] , θ ∈ [0, 2π).


z = cρ cos ϕ,

În funcţie de domeniul V trebuie determinate, mai precis, intervalele de variaţie pentru ρ, θ, ϕ, adică domeniul ∆.
a cos θ sin ϕ −aρ sin θ sin ϕ aρ cos θ cos ϕ


Jacobianul este ı̂n acest caz dat de J = b sin θ sin ϕ bρ cos θ sin ϕ bρ sin θ cos ϕ (se pot face calcule


c cos ϕ 0 −cρ sin ϕ
dezvoltând după a treia linie) şi se va obţine valoarea

J = −abcρ2 sin ϕ.

Deci (1) devine


ZZZ ZZZ
f (x, y, z) dxdydz = f (x (ρ, θ, ϕ) , y (ρ, θ, ϕ) , z (ρ, θ, ϕ)) abcρ2 sin ϕ dρdθdϕ.
V ∆

Corolarul 5 (folosirea coordonatelor cilindrice) Coordonatele cilindrice sunt (ρ, θ, h)




 x = ρ cos θ,

y = ρ sin θ, unde ρ ∈ [0, ∞), θ ∈ [0, 2π), h ∈ (−∞, +∞) .


z = h,

În funcţie de domeniul V trebuie determinate,


mai precis, intervalele
de variaţie pentru ρ, θ, h, adică domeniul ∆.
cos θ −ρ sin θ 0


Jacobianul este ı̂n acest caz dat de J = sin θ ρ cos θ 0 (se pot face calcule dezvoltând după a treia linie)


0 0 1
şi se va obţine valoarea
J = ρ.
Deci (1) devine
ZZZ ZZZ
f (x, y, z) dxdydz = f (x (ρ, θ, h) , y (ρ, θ, h) , z (ρ, θ, h)) ρ dρdθdh.
V ∆

Teorema 6 Volumul V al unui corp V este dat de


ZZZ
V= dxdydz
V

Teorema 7 Fie un corp V de densitate µ (x, y, z). Atunci masa este dată de
ZZZ
m= µ (x, y, z) dxdydz (2)
V

iar coordonatele centrului de greutate G (xG , yG , zG ) sunt date de


ZZZ
1


 x G = xµ (x, y, z) dxdydz



 m V
 ZZZ
 1
yG = yµ (x, y, z) dxdydz (3)

 m V
 ZZZ
1



 zG =
 zµ (x, y, z) dxdydz
m V

9
Teorema 8 (Formula lui Gauss-Ostrogradski) Fie corpul V mărginit de suprafaţa S (adică S = Fr (V ) ) la
care alegem faţa exterioară. Are loc următoarea formulă de legătură dintre intregrala triplă şi integrala de suprafaţă
de specia a doua:
ZZ ZZZ  
∂P ∂Q ∂R
P dydz + Qdzdx + Rdxdy = + + dxdydz.
S V ∂x ∂y ∂z

10

S-ar putea să vă placă și